SlideShare a Scribd company logo
1 of 67
ĐẠI HỌC THÁI NGUYÊN
TRƯỜNG ĐẠI HỌC KHOA HỌC
---------------------------
Tải tài liệu tại sividoc.com
Viết đề tài giá sinh viên – ZALO:0973.287.149-TEAMLUANVAN.COM
NGUYỄN THANH TÙNG
ĐA THỨC TRONG CÁC BÀI TOÁN
THI HỌC SINH GIỎI
LUẬN VĂN THẠC SĨ TOÁN HỌC
THÁI NGUYÊN - 2017
ĐẠI HỌC THÁI NGUYÊN
TRƯỜNG ĐẠI HỌC KHOA HỌC
---------------------------
Tải tài liệu tại sividoc.com
Viết đề tài giá sinh viên – ZALO:0973.287.149-TEAMLUANVAN.COM
Viết đề tài giá sinh viên – ZALO:0973.287.149-TEAMLUANVAN.COM
ĐẠI HỌC THÁI NGUYÊN
TRƯỜNG ĐẠI HỌC KHOA HỌC
---------------------------
Tải tài liệu tại sividoc.com
Viết đề tài giá sinh viên – ZALO:0973.287.149-TEAMLUANVAN.COM
NGUYỄN THANH TÙNG
ĐA THỨC TRONG CÁC BÀI TOÁN
THI HỌC SINH GIỎI
LUẬN VĂN THẠC SĨ TOÁN HỌC
Chuyên ngành: Phương pháp Toán sơ cấp
Mã số: 60 46 01 13
NGƯỜI HƯỚNG DẪN KHOA HỌC
GS.TSKH. Đặng Hùng Thắng
THÁI NGUYÊN - 2017
1
Viết đề tài giá sinh viên – ZALO:0973.287.149-TEAMLUANVAN.COM
Mục lục
Danh sách kí hi u 3
MƠ đau 4
Chương 1. Đa th c m t bien 7
1.1 Định nghĩa và các tính chat . . . . . . . . . . . . . . . . . . . . . 7
1.1.1 Định nghĩa . . . . . . . . . . . . . . . . . . . . . . . . . 7
1.1.2 Các phép tính trên đa thức . . . . . . . . . . . . . . . . . 8
1.1.3 Các tính chat cơ bản . . . . . . . . . . . . . . . . . . . . 9
1.2 Phép chia đa thức. Ưóc chung lón nhat và nhỏ nhat ............................ 11
1.2.1 Phép chia đa thức..................................................................... 11
1.2.2 Thu t toán Euclide ................................................................... 11
1.3 Nghi m của đa thức. Phương trình b c cao........................................... 16
1.3.1 Nghi m của đa thức ................................................................. 16
1.3.2 Phương trình b c cao ............................................................... 22
1.4 Đạo hàm của đa thức. Định lý Taylor .................................................. 32
Chương 2. Đa th c bat kha quy 36
2.1 Đa thức bat khả quy ............................................................................. 36
2.1.1 Đa thức vói h so thục và phức................................................ 37
2.1.2 Đa thức bat khả quy của vành Q[x] ......................................... 40
2
Viết đề tài giá sinh viên – ZALO:0973.287.149-TEAMLUANVAN.COM
2.2 M t so bài toán đien hình ..................................................................42
Chương 3. M t so chu đe khác 46
3.1 Đa thức nhieu bien ............................................................................... 46
3.2 Đa thức đoi xứng.................................................................................. 49
3.3 Phương trình hàm đa thức.................................................................... 53
3.4 Đa thức Chebyshev .............................................................................. 56
3.4.1 Định nghĩa - Tính chat............................................................. 57
3.4.2 M t so bài toán chon loc .......................................................58
Ket lu n 63
Tài li u tham khao 64
3
Viết đề tài giá sinh viên – ZALO:0973.287.149-TEAMLUANVAN.COM
Danh sách kí hi u
Z vành so nguyên
Q trưòng so hữu tý
R trưòng so thục
C trưòng so phức
R vành
F trưòng
R[x] vành đa thức vói h so trên vành R
degP(x) b c của đa thức P(x)
P(x) . Q(x), Q(x) | P(x) đa thức Q(x) là ưóc của đa thức P(x)
gcd(P(X),Q(X)) ưóc chung lón nhat của P(X) và Q(X)
a ≡ b (mod p) a đong dư vói b theo modulo p
m
i=1
m
i=1
ai ký hi u tong a1 +a2 +···+am
bi ký hi u tích b1b2 ···bm
∑
∏
4
Viết đề tài giá sinh viên – ZALO:0973.287.149-TEAMLUANVAN.COM
MƠ đau
Đa thức là m t đoi tưong quan trong của Toán hoc cả ve m t lý thuyet cũng như
ứng dụng. Đoi vói Toán hoc pho thông, hoc sinh làm quen vói các phép toán trên
đa thức (c ng trừ nhân chia), giải các phương trình b c nhat, b c hai và m t so
dạng phương trình b c cao. Trong các kỳ thi hoc sinh giỏi quoc gia và quoc te, chủ
đe đa thức cũng đưoc khai thác sâu hơn vói các bài toán hay và tương đoi khó ve
phương trình đại so b c cao, phương trình hàm đa thức, đathức bat khả quy, tính
chia het của đa thức ...
Các bài toán nâng cao ve đa thức xuat hi n cũng khá nhieu trong các tạp chí
toán hoc cho hoc sinh khá giỏi (như Tạp chí Toán hoc và Tuoi trẻ, Kvant, Crux,. . . ).
Tuy nhiên hi n nay có ít các tài li u ve tieng Vi t trình bày m t cách h thong cả
lý thuyet và bài t p ve đa thức, vói định hưóng boi dưõng hoc sinh giỏi Toán và
boi dưõng giáo viên dạy chuyên Toán.
Mục tiêu của lu n văn là tìm hieu m t cách đay đủ những ket quả quan trong
của đa thức có nhieu ứng dụng trong Toán pho thông. Trên cơ sỏ đó, phân loại và
h thong hoá (theo dạng cũng như phương pháp giải) các bài t p nâng cao ve đa
thức đã có cũng như sáng tác, bo sung thêm những bài toán mói.
Chúng tôi rat co gang đe lu n văn này trỏ thành m t tài li u tham khảo tot, thiet
thục phục vụ cho vi c giảng dạy hoc sinh giỏi và boi dưõng giáo viên. Thông qua
vi c viet lu n văn hoc viên sẽ mỏ r ng nâng cao hieu biet ve đa thức, hình thành
các kỹ năng giải các bài toán khó ve đa thức, kỹ năng tìm kiem thu th p chon loc
các thông tin.
N i dung của lu n văn đưoc trình bày trong ba chương như sau:
5
Viết đề tài giá sinh viên – ZALO:0973.287.149-TEAMLUANVAN.COM
• Chương 1. Đa thŕc m t bien. Trong chương này chúng tôi sẽ trình bày ngan
gon ve định nghĩa và các tính chat của đa thức. Các van đe nen tảng ve phép
chia đa thức, ưóc - b i, nghi m và phương trình b c cao, đạo hàm và khai
trien Taylor sẽ đưoc trình bày.
• Chương 2. Đa thŕc bat khá quy. Đa thức bat khả quy là m t trong những
chủ đe trong tâm của lý thuyet các đa thức. Nó vừa mang tính chat lý thuyet,
vừa mang tính ứng dụng, đ c bi t là các bài t p nâng cao trong các đe thi
có tính chat tuyen chon. Chương này chúng tôi t p trung nghiên cứu các đa
thức bat khả quy trên các vành (trưòng) so quen biet của toán hoc sơ cap.
• Chương 3. M t so chủ đe khác. Chương này dành đe nghiên cứu m t so van
đe nâng cao của lý thuyet đa thức, mà mục đích của nó là đe hieu biet sâu sac
hơn lý thuyet, đong thòi là nen tảng cho các ứng dụng. Các van đe đưoc quan
tâm trong chương này là các đa thức nhieu bien, đa thức đoi xứng, phương
trình hàm đa thức và đa thức Chebyshev.
Lu n văn này đưoc thục hi n tại Trưòng Đại hoc Khoa hoc - Đại hoc Thái
Nguyên và hoàn thành vói sụ hưóng dȁn của GS.TSKH. Đ ng Hùng Thang (Trưòng
ĐHKHTN - ĐHQG Hà N i). Tác giả xin đưoc bày tỏ lòng biet ơn chân thành và
sâu sac tói ngưòi hưóng dȁn khoa hoc của mình, ngưòi đã đ t van đe nghiên cứu,
dành nhieu thòi gian hưóng dȁn và t n tình giải đáp những thac mac của tác giả
trong suot quá trình làm lu n văn.
Tác giả xin trân trong cảm ơn Ban Giám hi u Trưòng Đại hoc Khoa hoc - Đại
hoc Thái Nguyên, Ban Chủ nhi m Khoa Toán–Tin, cùng các giảng viên đã tham
gia giảng dạy, đã tạo moi đieu ki n tot nhat đe tác giả hoc t p và nghiên cứu.
Tác giả muon gửi những lòi cảm ơn tot đep nhat tói t p the lóp Cao hoc Toán
khóa 9 (2015-2017) đã đ ng viên và giúp đõ tác giả rat nhieu trong suot quá trình
hoc t p.
Nhân dịp này, tác giả cũng xin chân thành cảm ơn Sỏ Giáo dục và Đào tạo Hải
6
Viết đề tài giá sinh viên – ZALO:0973.287.149-TEAMLUANVAN.COM
Phòng, Ban Giám hi u và các đong nghi p ỏ Trưòng THPT Hùng Vương đã tạo
đieu ki n cho tác giả hoàn thành tot nhi m vụ hoc t p và công tác của mình.
Cuoi cùng, tác giả muon dành những lòi cảm ơn đ c bi t nhat đen bo me và
đại gia đình đã luôn đ ng viên và chia sẻ những khó khăn đe tác giả hoàn thành
tot lu n văn này.
Thái Nguyên, ngày 02 tháng 11 năm 2017
Tác giả
Nguyen Thanh Tùng
7
Viết đề tài giá sinh viên – ZALO:0973.287.149-TEAMLUANVAN.COM
Chương 1
Đa th c m t bien
1.1 Định nghĩa và các tính chat
Phan này chúng tôi sẽ trình bày ngan gon nhat có the lý thuyet các đa thức m t
bien. Những chi tiet hơn có the tham khảo Lê Thị Thanh Nhàn [5] ho c Nguyen
Văn M u [3].
1.1.1 Định nghĩa
Giả sử R là m t vành giao hoán có đơn vị.
Định nghĩa 1.1.1. Bieu thức có dạng
anxn
+an−1xn−1
+ ...+ a1x +a0 vói an 0
trong đó an, an−1,...,a1, a0 là những phan tử thu c vành R, đưoc goi là m t đa
thŕc trên vành R.
Trong định nghĩa này, ai đưoc goi là các h so của đa thức, h so an đưoc goi
là h so b c cao nhat của đa thức, so tụ nhiên n đưoc goi là b c của đa thức, ký
hi u là deg P(x), x đưoc goi là ȁn, hay bien hay đoi so của đa thức, an đưoc goi là
h so cao nhat, a0 đưoc goi là h so tự do của đa thức.
Neu ai = 0 vói i = 1,2,...,n−1 và a0 0 thì ta có b c của đa thức là không.
Neu ai = 0 vói i = 1,2,...,n thì f (x) = 0, ta goi đa thức này là đa thŕc không.
8
Viết đề tài giá sinh viên – ZALO:0973.287.149-TEAMLUANVAN.COM
Nói chung ngưòi ta không định nghĩa b c của đa thức không nhưng ta coi b c của
nó là −∞.
Hai đa thức f và g đưoc goi là bang nhau, và viet f = g, neu chúng cùng là đa
thức không, ho c cả hai khác đa thức không, đong thòi deg f = deg g và các h so
tương ứng bang nhau.
T p hop tat cả các đa thức lay h so trong vành R đưoc ký hi u là R[x], và đưoc
goi là vành đa thŕc trên R. Khi R là m t trưòng, thì vành R[x] là m t vành giao
hoán có đơn vị.
Vói lý do là ứng dụng lý thuyet đa thức trong các bài thi hoc sinh giỏi, hay nói
chung là các kỳ thi có tính chat tuyen chon, lu n văn này thưòng xét R là Z, Q, R,
ho c C, khi đó các đa thức thu c Z[x], Q[x], R[x], ho c C[x] đưoc goi tên lan lưot
là các đa thŕc nguyên, đa thŕc hru tý, đa thŕc thực, ho c đa thŕc phŕc.
1.1.2 Các phép tính trên đa th c
Cho hai đa thức
f (x) = anxn
+ an−1xn−1
+...+ a1x + a0,
g(x) = bnxn
+ bn−1xn−1
+... +b1x + b0.
Ta định nghĩa các phép tính so hoc như sau
• Phép c ng
f (x)+ g(x) = (an +bn)xn
+(an−1 +bn−1)xn−1
+. . . +(a1 +b1)x+(a0 + b0).
• Phép trừ
f (x)− g(x) = (an −bn)xn
+(an−1 −bn−1)xn−1
+. . .+(a1 −b1)x −(a0 − b0).
• Phép nhân
f (x)g(x) = c2nx2n
+ c2n−1x2n−1
+ ... + c1x + c0
9
Viết đề tài giá sinh viên – ZALO:0973.287.149-TEAMLUANVAN.COM
trong đó
ck = a0bk +a1bk−1 +...+akb0 vói k = 0,1,...,n.
1.1.3 Các tính chat cơ ban
Định lí 1.1.2. Giá sr F m t trường. Với hai đa thŕc f (x) và g(x) thu c vành đa
thŕc F[x], luôn ton tại c p đa thŕc q(x) và r(x) duy nhat thu c vành đa thŕc F[x]
đe
với degr(x) < degg(x).
f(x) = g(x)·q(x)+ r(x),
Ta goi q(x) và r(x) lan lưot là đa thŕc thương (ho c thương) và đa thŕc dư (hay
dư) trong phép chia f (x) cho g(x).
Neu r(x) = 0 thì ta nói f(x) chia het cho g(x), hay g(x) chia het f(x) hay f(x)
là b i của g(x) hay g(x) là ước của f(x). Ta sẽ kí hi u là f . g hay g | f .
Giả sử f (x) = anxn
+ an−1xn−1
+ ... + a1x + a0 là m t đa thức thu c vành đa
thức R[x]. Xét phan tử α ∈ R bat kỳ. Khi đó
f (α) = anαn
+ an−1αn−1
+ .. . + a1α + a0
đưoc goi là giá trị của đa thŕc f (x) tại α.
Neu f (α) = 0 thì α đưoc goi là m t nghi m của đa thức f (x). Neu ton tại
k ∈ N, k > 1 sao cho f (x) . (x−α)k
nhưng f (x) không chia het cho (x−α)k+1
thì
α đưoc goi là nghi m b i k của đa thức f (x). Đ c bi t, k = 1 thì α đưoc goi là
nghi m đơn, k = 2 thì α đưoc goi là nghi m kép.
Bài toán tìm nghi m của đa thức
f(x) = anxn
+ an−1xn−1
+...+a1x+a0 vói an =
/ 0.
trong vành R đưoc goi là giái phương trình đại so b c n trong R.
10
Viết đề tài giá sinh viên – ZALO:0973.287.149-TEAMLUANVAN.COM
k
Định lí 1.1.3. Giá sr F m t trường, α ∈ F và f (x) ∈ F[x]. Dư của phép chia f (x)
cho (x −α) chính là f (α).
Định lí 1.1.4 (Định lý Bézout). Phan tr α ∈ F là nghi m của đa thŕc f (x) ∈ F[x]
khi và chỉ khi f (x) chia het cho (x − α).
Định lý sau đây cho ta m t đánh giá ve so nghi m của m t đa thức thục.
Định lí 1.1.5. M t đa thŕc thực b c n đeu có không quá n nghi m thực.
Định lý này có m t so h quả sau đây.
H qua 1.1.6. Đa thŕc có vô so nghi m là đa thŕc không.
H qua 1.1.7. Neu m t đa thŕc có b c không vượt quá n mà nh n cùng m t giá trị
tại n+1 điem khác nhau của ȁn thì đa thŕc đó là đa thŕc hang.
H qua 1.1.8. Hai đa thŕc có b c không vượt quá n mà nh n cùng m t giá trị thóa
mãn bang nhau tại n + 1 giá trị khác nhau của ȁn thì hai đa thŕc đó đong nhat
bang nhau.
Trong trưòng hop đa thức phức, ta có ket quả sau đây ve so lưong nghi m của
nó.
Định lí 1.1.9. M t đa thŕc phŕc b c n có đúng n nghi m tính cá b i.
Cuoi cùng của mục này, ta sẽ trình bày không chứng minh m t ket quả ve dạng
bieu dien của các đa thức thục.
Định lí 1.1.10. Bat kỳ đa thŕc thực f (x) ∈ R[x] nào có b c n và h so b c cao nhat
an /= 0 có the phân tích m t cách duy nhat (không tính thŕ tự) thành các nhân tr
m s
f (x) = an ∏(x − di) ∏(x2
+ bkx + ck)
i=1 k=1
với di, bk, sk ∈ R, 2s+ m = n và b2
− 4ck < 0 với m, n ∈ N.
11
Viết đề tài giá sinh viên – ZALO:0973.287.149-TEAMLUANVAN.COM
q
Trong lý thuyet đa thức, m t phan rat quan trong đó là khảo sát các nghi m
hữu tý và nguyên của m t đa thức. Ta có định lý sau đây.
Định lí 1.1.11. Xét đa thŕc nguyên f (x) ∈ Z[x] có dạng
f (x) = a0xn
+a1xn−1
+...+ an−1x+an, a0 0.
Nghi m hru tý neu có x = p
với gcd(p, q) = 1 thì p là ước của h so tự do và q là
ước của h so b c cao nhat, tŕc là p | an, q | a0.
H qua 1.1.12. Xét đa thŕc nguyên f (x) ∈ Z[x] có dạng chuȁn tac, tŕc là h so b c
cao nhat là 1. Khi đó mői nghi m hru tý neu có của đa thŕc f (x) đeu là nghi m
nguyên.
1.2 Phép chia đa th c. ƯỚc chung lỚn nhat và nho nhat
1.2.1 Phép chia đa th c
Định nghĩa 1.2.1. M t đa thức d(x) chia het hai đa thức f (x) và g(x) goi là ước
chung của f (x) và g(x). Neu d(x) là m t ưóc chung chia het cho moi ưóc chung
khác của hai đa thức f (x) và g(x) đúng thì ta goi d(x) là ước chung lớn nhat của
f(x) và g(x) .
Rõ ràng các ưóc chung lón nhat sai khác hang so, đe bảo đảm tính duy nhat ta
có the quy ưóc chon ưóc chung lón nhat dạng chuan tac (h so cao nhat bang 1).
Ta kí hi u là d(x) = (f (x),g(x)) = gcd(( f(x),g(x)).
1.2.2 Thu t toán Euclide
Ta chia liên tiep
f(x) = g(x)·q(x)+ r(x)
g(x) = r(x)·q1(x)+r1(x)
12
Viết đề tài giá sinh viên – ZALO:0973.287.149-TEAMLUANVAN.COM
r(x) = r1(x) · q2(x) + r2(x)
...
rk−2(x) = rk−1(x) · qk(x) + rk(x)
rk−1(x) = rk(x ·qk+1(x).
Khi đó (f(x),g(x)) = rk
∗(x) vói rk
∗(x) = c·rk(x) đa thức có h so b c cao nhat là
1 (ngưòi ta thưòng goi những đa thức này là đa thŕc chuȁn tac hay đa thŕc monic).
Ket quả neu d(x) = ( f (x),g(x)) thì ton tại hai đa thức u(x),v(x) ∈ R[x] đe có
bieu dien
f(x)·u(x)+ g(x)·v(x) = d(x).
Hơn nữa ta có the chon degu < degg và degv < deg f.
Bây giò ta xét m t so ví dụ đe hieu sâu sac hơn lý thuyet. Các ví dụ này chúng
tôi tham khảo trong Lê Hoành Phò [6].
Bài toán 1.2.2. Cho P(x) = x + x3
+x9
+x27
+x81
+x243
. Tìm dư của phép chia
đa thŕc P(x) cho
(a) x−1,
(b) x2
−1.
Lời giái. (a) Ta có P(x) = (x − 1)Q(x) + r(x) vói degr(x) < deg(x − 1) = 1. Suy
ra degr(x) = 0 nên dư r(x) = c. Do đó P(x) = (x − 1) · Q(x) + c. Chon x = 1 suy
ra P(1) = c hay c = P(1) = 6. V y dư r(x) = 6.
(b) Ta có P(x) = (x2
− 1)· H(x)+s(x) vói degs(x) ≤ 1. V y
P(x) = (x2
− 1)·H(x)+ax+b.
Chon x = 1 ta có P(1) = a + b = 6; x = −1 ta có P(−1) = −a + b = −6. Do đó
a = 6, b = 0. V y ta có đa thức dư là r(x) = 6x.
13
Viết đề tài giá sinh viên – ZALO:0973.287.149-TEAMLUANVAN.COM
Bài toán 1.2.3 (Trung Quoc 1981). Tìm dư của phép chia
(a) x12
+x8
+x4
+1 cho x3
+x2
+x+1;
(b) f(x100
) cho f(x) với f(x) = x99
+x98
+...+x+1.
Lời giái. (a) Ta có
x12
+x8
+x4
+1 = (x3
+x2
+x+1)(x9
−x8
+2x5
+ 2x4
+3x−3)+4.
Suy ra dư là 4.
(b) Ta có
nên
f (x) = x99
+ x98
+ ... + x + 1
f (x100
) = x9900
+ x9800
+ ... + x100
+ 1
= f (x) x9800
+ 2x9701
−2x9700
+3x9601
−3x9600
+ ... + 99x − 99) + 100
nên dư là 100.
Bài toán 1.2.4. Xác định a và b đe đa thŕc f (x) = 6x4
− 7x3
+ ax2
+ 3x + 2 chia
het cho x2
−x+b.
Lời giái. Lay đa thức f (x) = 6x4
− 7x3
+ax2
+3x +2 chia cho g(x) = x2
− x +b
thì đưoc thương q(x) = 6x2
− x + (a − 5b − 1) và phan dư r(x) = (a − 5b + 2)x +
(−ab−6b2
+a+b+2).
Vì f (x) . g(x) nên r(x) = 0 suy ra
a− 5b +2 = 0, (1.1)
—ab +6b2
+b+2 = 0. (1.2)
Từ (1.1) suy ra a = 5b − 2. Thay vào (1.2) ta có b2
+ 3b + 2 = 0 nên b = −1 ho c
b = −2. Vói b = −1 thì a = −7. Vói b = −2 thì a = −12.
V y f (x) = 6x4
− 7x3
− 7x2
+ 2 hay f (x) = 6x4
− 7x3
− 12x2
+ 3x + 2.
14
Viết đề tài giá sinh viên – ZALO:0973.287.149-TEAMLUANVAN.COM
3
2 2 2 2
2 2
Bài toán 1.2.5. Tìm ước chung lớn nhat của hai đa thŕc f(x) = x2
+ x3
− 3x2
−
4x−1 và g(x) = x3
+x2
−x−1.
Lời giái. Ta thục hi n các phép chia liên tiep và hő tro vói phép nhân thêm hang
so
thì
f(x) = q(x)·g(x)+r(x)
q(x) = x, r(x) = −2x2
− 3x − 1
và g(x) = q1(x) · r(x) + r1(x) thì
q1(x) = −
1
x−
1
, r1(x) = −
3
x −
3
,
và tiep tục r(x) = q2(x) · r1(x) + r2(x) thì
q2(x) = −
2
(2x + 1), r2(x) = 0
Do đó ( f(x),g(x)) = x + 1 vói quy ưóc lay h so cao nhat bang 1 từ r1(x) =
−3
x− 3
.
Bài toán 1.2.6 (New York 1973, Bỉ 1981). Chŕng minh rang với moi giá trị n ∈ N,
đa thŕc (x+1)2n+1
+xn+2
chia het cho đa thŕc x2
+x+1.
Lời giái. Ta chứng minh bang quy nạp theo n ∈ N.
Vói n = 0 khȁng định đúng vì khi đó
(x + 1)2n+1
+ xn+2
= x2
+ x.
Giả sử khȁng định đúng vói n − 1, nghĩa là (x + 1)2n−1
+ xn+1
chia het cho x2
+
x+1. Khi đó đa thức
(x+1)2n+1
+xn+2
= (x +1)2
·(x + 1)2n−1
+x·xn+1
= (x2
+2x+1)(x+1)2n−1
+x ·xn+1
= (x2
+ x + 1)(x + 1)2n−1
+ x[x + 1)2n−1
+ xn+1
]
chia het cho x2
+ x + 1. Suy ra đieu chứng minh vói n.
15
Viết đề tài giá sinh viên – ZALO:0973.287.149-TEAMLUANVAN.COM
Bài toán 1.2.7. Cho hai so nguyên dương n và k. Chŕng minh rang xn
− 1 . xk
− 1
khi và chỉ khi n là b i so của k.
Lời giái. Ta có the phát bieu bài toán dưói dạng: Đe xn
− 1 . xk
− 1 đieu ki n can
và đủ là n là b i so của k.
Đieu ki n đủ. Giả sử n là b i so của k tức là n = km vói m nguyên dương. Khi đó
xn
−1 = xkm
−1
= (xk
)m
−1
= (xk
−1)[x(k−1) +xk(m−1) +xk(m−2) +...+xk
+1].
Đȁng thức này chứng tỏ rang xn
− 1 . xk
− 1.
Đieu ki n can. Ta lay so nguyên dương n chia cho so nguyên dương k.
Giả sử q và r là thương và so dư trong phép chia, tức là có n = kq + r vói
0 ≤ r < k. Khi đó
xn
− 1 = xkq+r
− 1 = xkq+r
− xr
+ xr
− 1 = xr
(xkq
− 1) + xr
− 1. (1.3)
Ő trên ta đã chứng minh xkq
−1 . xk
−1. Vì v y neu xn
−1 . xk
−1 thì từ (1.3) suy ra
xr
−1 . xk
−1. Nhưng r < k nên xr
−1 . xk
−1 khi r = 0. Thành ra neu xn
−1 . xk
−1
thì r = 0, tức là n = kq, hay n là b i so của k.
Bài toán 1.2.8 (Rumani 1962). Tìm đieu ki n của các so nguyên p và q sao cho
đa thŕc
(a) P(x) = x2
+ px + q nh n cùng giá trị chȁn (lé) với moi x ∈ Z.
(b) Q(x) = x3
+ px + q nh n các giá trị chia het cho 3 với moi x ∈ Z.
Lời giái. (a) P(x) nh n giá trị cùng chȁn (ho c lẻ) vói moi x ∈ Z khi và chỉ khi mői
so P(x + 1)− P(x) = 2x + 1+ p chia het cho 2 nghĩa là p lẻ.
16
Viết đề tài giá sinh viên – ZALO:0973.287.149-TEAMLUANVAN.COM
Khi đó tính chȁn lẻ của P(x) phụ thu c vào tính chȁn lẻ của q = P(0). Như v y
tat cả giá trị của P(x) là chȁn (lẻ) khi p lẻ và q chȁn (tương ứng q lẻ).
(b) Vì Q(x) = x(x2
+ p) + q nên Q(3x) = 3x(9x2
+ p)+ q chia het cho 3. Vói giá
trị đó thì
Q(3x± 1) = (3x ±1)(9x2
±6x +1+ p)+q ≡ ±(1+ p) (mod 3)
chia het cho 3 khi và chỉ khi 1 + p chia het cho 3.
V y Q(x) chia het cho 3 (vói moi x ∈ Z) khi q = 0, q ≡ 2 (mod 3).
Bài toán 1.2.9 (Hong Kong 2008). Xét đa thŕc
f(x) = cmxm
+cm−1xm−1
+...+c1x +c0
với các h so ci là nhrng so nguyên khác không. Xây dựng dãy so (an) như sau:
a1 = 0 và an+1 = f (an) với n = 1,2,... Giá sr i, j là nhrng so nguyên dương vài
< j. Chŕng minh rang (aj+1 −aj) là b i của (ai+1 −ai).
Chŕng minh. Do f(x) là đa thức h so nguyên nên [f(a) − f(b)] chia het cho
(a−b), vói a và b là hai so nguyên phân bi t.
Do đó ai+2 − ai+1 = f (ai+1) − f (ai) chia het cho (ai+1 − ai), vói moi i > 0.
Như v y neu i, j là những so nguyên dương và i < j thì (aj+1 − a j) chia het cho
(ai+1 −ai). Ta có đieu phải chứng minh.
1.3 Nghi m cua đa th c. Phương trình b c cao
1.3.1 Nghi m cua đa th c
Bài toán 1.3.1. Cho đa thŕc b c chȁn và tat cá h so đeu lé. Chŕng minh đa thŕc
không có nghi m hru tý.
Chŕng minh. Xét P(x) = a0xn
+a1xn−1
+...+an−1x+an vói a0 /= 0.
17
Viết đề tài giá sinh viên – ZALO:0973.287.149-TEAMLUANVAN.COM
q
q
p p2 ps
Vói n chȁn, các h so ai lẻ. Giả sử đa thức có nghi m hữu tý x = p
thì p | a0,
q | an. Suy ra p, q lẻ.
The x = p
vào đa thức ta có
an pn
+ an−1qpn−1
+ ... + a0qn
= 0.
Đieu này vô lý vì ve trái là tong của m t so lẻ các so hạng lẻ nên không the bang
0. V y đa thức không có nghi m hữu tý. Ta có đieu phải chứng minh.
Bài toán 1.3.2. Cho so tự nhiên n ≥ 2, chŕng minh phương trình
xn xn−1 x2
x
n!
+
(n−1)!
+...+
2!
+
1!
+1 = 0. (1.4)
không có nghi m hru tý.
Chŕng minh. Ta chứng minh bài toán bang phương pháp phản chứng. Giả sử
phương trình đã cho có nghi m hữu tý α. Khi đó α sẽ là nghi m hữu tý của
đa thức
P(x) = xn
+ nxn−1
+...+
xk
k!
... + n!
x2
2!
n!
x
+
1!
n!.
Nhưng do P(x) là m t đa thức b c n vói h so nguyên, và h so b c cao nhat, của
xn
, là 1 nên α phải là so nguyên và ta có
αn
+nαn−1
+...+
αk
k!
+...+ n!
α2
1!
n! = 0. (1.5)
Goi p là m t ưóc nguyên to của n. Vói moi k = 1, 2, . . . , n, kí hi u rk là so mũ
cao nhat của p thỏa mãn k! . prk, ta có
rk =
k
+
k
+...+
k
. (1.6)
Vói s là so nguyên không âm thỏa mãn ps
≤ k < ps+1
, từ (1.6) ta suy ra
r
k k k p2
k ≤
p
+
p2 + ... +
ps = k
p − 1
< k.
Do đó rn −rk > rn − k. Suy ra rn − rk ≥ rn − k +1. Vì v y ta đưoc
n!
: pn−k+1
vói moi k = 1,2,...,n. (1.7)
k!
n! +
n! +
18
Viết đề tài giá sinh viên – ZALO:0973.287.149-TEAMLUANVAN.COM
n
2
2
— −
2
Nhưng mà n . p nên từ (1.5) ta có αn . p và do đó α . p. Suy ra αk . pk
vói moi
k = 1,2,...,n.
Ket hop đieu này vói (1.7) ta đưoc n!αk
. pr+1
vói moi k = 1,2,...,n. Từ đây
và (1.5) ta suy ra n! . pr+1
.
k! n
Mâu thuȁn vừa nh n đưoc chứng tỏ giả sử ban đau là sai và vì v y ta có đieu
phải chứng minh.
Bài toán 1.3.3 (Vi t Nam 1992). Cho đa thŕc
P(x) = 1+x2
+x9
+xn1 +...+xns
+x1992
.
Với n1,...,ns là các so tự nhiên cho trước thóa mãn
9 < n1 < ... < ns < 1992.
Chŕng minh rang nghi m của đa thŕc P(x) (neu có) không the lớn hơn 1−
√
5
.
Lời giái. Ta có
P(x) = 1+x2
+x9
+xn1 +...+xns
+x1992
.
Vói x ≥ 0 thì P(x) ≥ 1 > 0. Ta sẽ chứng minh
P(x) > 0 vói moi x ∈
1−
√
5
;0
!
.
Th t v y vói x < 0 và x /= −1 ta có
P(x) ≥ 1 + x + x3
+ x5
+ ... + x2k+1
+ ... + x1991
(x1990
+x1998
+...+1).(1−x2
)
= 1+x
= 1+x 1 − x996
1 − x2
(1 − x2)
1 x2
+x x997
=
1−x2
.
Mà vói x ∈ 1−
√
5
;0 thì
1 − x2
> 0, −x997
> 0, 1 − x2
+ x > 0.
19
Viết đề tài giá sinh viên – ZALO:0973.287.149-TEAMLUANVAN.COM
2
2
nên
P(x) > 0 vói moi x ∈
1−
√
5
;0
!
.
V y P(x) > 0 vói x ∈ 1−
√
5
;+∞ . Ta có đieu phải chứng minh.
Bài toán 1.3.4 (IMO 1976). Cho các đa thŕc Pk(x), với k = 1,2,3... xác định bới
P1(x) = x2
− 2, Pi+1 = P1(Pi(x)), với i = 1,2,3,...
Chŕng minh rang Pn(x) = x có 2n
nghi m thực phân bi t.
Lời giái. Ta thu hep vi c xét nghi m của phương trình trên đoạn −2 ≤ x ≤ 2.
Đ t x = 2cos(t). Khi đó, bang quy nạp ta chứng minh đưoc
Pn(x) = 2cos(2nt
).
Thêm nữa, phương trình Pn(x) = x trỏ thành cos(2nt
) = cos(t). Từ đó ta đưoc 2n
nghi m
t =
2kπ
, t =
2kπ
, k = 1,2,...,n
2n −1 2n + 1
V y phương trình Pn(x) = x có 2n
nghi m thục phân bi t.
Bài toán 1.3.5. Cho đa thŕc f(x) = a0 +a1x+...+anxn
có n nghi m thực. Chŕng
minh với moi p > n−1 thì đa thŕc
g(x) = a0 + a1.p.x + a2.p(p − 1)x2
+ ... + an p(p − 1)...(p − n + 1)xn
.
cũng có n nghi m thực.
Lời giái. Đe giải bài toán ta xét hai trưòng hop
Trường hợp 1. Đa thức f (x) không nh n x = 0 làm nghi m.
Ta chứng minh bang quy nạp.
Vói n = 1 bài toán hien nhiên đúng .
20
Viết đề tài giá sinh viên – ZALO:0973.287.149-TEAMLUANVAN.COM
Giả sử đúng vói n = k, ta chứng minh đúng vói n = k + 1, tức là neu đa thức
f (x) = a0 +a1x +... + ak+1xk+1
có k +1 nghi m thục khác 0 thì đa thức
g(x) = a0 + pa1x + ... + p(p − 1)...(p − k)ak+1xk+1
cũng có k +1 nghi m thục khác 0 vói moi p > k.
Goi c là m t nghi m của f (x) thì
f(x) = (x−c)q(x), (1.8)
vói q(x) là m t đa thức nào đó là đa thức b c k của x,
q(x) = b0 + b1x + ... + bkxk
. (1.9)
Thay (1.9) vào (1.8) roi đong nhat h so ta đưoc
a0 = cb0, a1 = cb1 + b0, ... ak = cbk + bk−1, ak+1 = bk
Do đó
trong đó
g(x) = a0 + pa1x + ... + p(p − 1)...(p − k)ak+1xk+1
= cb0 + p(cb1 + b0)x + .. . + p(p − 1)(p − k)bkxk+1
= cQ(x) + pxQ(x) − x2
Q(x)
Q(x) = b0 + b1 px + ... + p(p − 1)...(p − k + 1)bkxk
.
Do f (x) có k +1 nghi m thục khác 0 nên q(x) có k nghi m thục khác 0. M t khác
p > k nên p > k −1.
Cho nên theo giả thiet quy nạp ta có đa thức Q(x) có k nghi m thục. Do đó
g(x) có k +1 nghi m thục. V y theo nguyên lý quy nạp, bài toán đúng.
Trường hợp 2. Đa thức f (x) nh n x = 0 làm nghi m.
21
Viết đề tài giá sinh viên – ZALO:0973.287.149-TEAMLUANVAN.COM
d
Giả sử x = 0 là nghi m b i k của f(x), vói k ∈ Z+, k ≤ n. Khi đó ta có
f (x) = akxk
+ ... + anxn
= (anxn−k
+ ... + ak)xk
và
g(x) = p(p − 1)...(p − k + 1)ak.xk
+... + p(p − 1)...(p −n + 1)an.xn
= p(p− 1)...(p −k + 1)xk
[ak +...+(p −k)...(p − n+ 1)anxn−k
].
Vì f (x) có n nghi m thục nên H(x) = ak + ... + anxn−k
có (n − k) nghi m thục
khác 0.
Do đó áp dụng ket quả của Trưòng hop 1 cho H(x) và p′ = p − k > n − k − 1
(do p > n − 1), ta đưoc đa thức
R(x) = ak + ... +(p − k)...(p − n + 1)anxn−k
có n − k nghi m thục.
V y g(x) có n nghi m thục.
Bài toán 1.3.6 (Trung Quoc 1996). Cho đa thŕc p(x) b c 5 có 5 nghi m thực phân
bi t. Tìm so bé nhat của các h so khác 0.
Lời giái. Xét
p(x) = ax5
+bx4
+cx3
+dx+e, a 0.
Neu có 4 h so bang 0 thì b = c = d = e = 0 nên p(x) = ax5
có nghi m b i (loại)
tức là p(x) không the có m t h so khác 0. Do đó p(x) có ít nhat hai h so khác 0.
Xét p(x) = ax5
+bxn
, n ≥ 2 thì p(x) có nghi m b i. Ta tiep tục loại trưòng hop
này.
Xét p(x) = ax5
+dx = ax ax4
+ a
có toi đa ba nghi m. Ta cũng loại.
Xét p(x) = ax5
+e có m t nghi m. Ta cũng loại.
Do đó p(x) có ít nhat ba h so khác 0. Chon p(x) = x5
−5x3
+4x = x(x2
−4)
thì p(x) có đúng 5 nghi m phân bi t và đúng ba h so khác 0.
V y so bé nhat của h so khác 0 là 3.
22
Viết đề tài giá sinh viên – ZALO:0973.287.149-TEAMLUANVAN.COM
3
3
27
3
p
2p
√
p
1.3.2 Phương trình b c cao
Lý thuyet giai phương trình b c 3 tong quát
Xét phương trình đa thức b c ba
ax3
+ bx2
+ cx + d = 0, a /= 0. (1.10)
Ngoài vi c tách nhóm so hạng ho c tìm m t nghi m roi phân tích nhân tử, dùng
hang đȁng thức, ta có cách giải tong quát như sau:
Trưóc het, chia 2 ve cho a /= 0 đưa ve phương trình: x3
+ Bx2
+Cx + D = 0.
Tiep theo đ t x = y − B
đưa tiep ve phương trình: y3
− py = q, trong đó
p
b2 2B3
BC
=
3
−C, q = −
27
+
3
− D.
Có hai hưóng đe giải phương trình
y3
− py = q. (1.11)
Hướng thŕ nhat. Đ t y = u + v và chon u, v = p
thì từ y3
= u3
+ v3
+ 3uv(u + v) ta
có u3
+v3
= q và u3
.v3
= P3
. V y u3
,v3
là nghi m phương trình
3
Z3
−qZ +
27
= 0.
Neu ∆ < 0 sau này ta dùng so phức đe giải quyet.
Hướng thŕ hai. Neu p = 0 thì (1.11) tương đương vói y3
= q, tức là y =
√
q.
Neu p > 0. Đ t y = 2
q
p
t thì (1.11) tương đương vói
4t3
− 3t = m (1.12)
3
√
3q
trong đó m = .
Xét |m| ≤ 1, đ t m = cosα thì (1.12) có ba nghi m
t1 = cos
α
, t2 = cos
α +2π
, t3 = cos
α − 2π
.
3 3 3
23
Viết đề tài giá sinh viên – ZALO:0973.287.149-TEAMLUANVAN.COM
3
2
2
Xét |m| > 1, đ t m = 1
d3
+ 1
suy ra d3
= m ±
√
m2 − 1. Phương trình
2 d 2
2 k3
2 k 2
2 d3
(1.12) có m t nghi m
t =
1
d +
1
=
1
q
3
m+
√
m2 −1+
q
3
m−
√
m2 −1
Neu p < 0. Đ t y = 2.
q
− p
t thì (1.11) tương đương vói
4t3
+ 3t = m. (1.13)
Ta đ t tiep
m =
1
k3
−
1
suy ra k3
= m ±
√
m2 +1.
Phương trình (1.13) có m t nghi m
t =
1
k −
1
=
1
q
3
m+
√
m2 + 1+
q
3
m −
√
m2 +1 .
Ta goi các phương trình b c ba 4x3
+ 3x + −m = 0, 4x3
−3x − m = 0 là các
dạng phương trình b c 3 chuȁn tac. Ý nghĩa cơ bản là moi phương trình b c 3 đeu
đưa ve đưoc dạng chuan tac đó.
Chú ý thêm khi |m| ≥ 1,
4x3
+3x−m = (x−α)(4x2
+4αx+4α2
+3).
Vói
α =
1
q
3
m+
√
m2 + 1+
q
3
m−
√
m2 + 1
có ∆′ = −12(α2 + 1) < 0 và
4x3
− 3x − m = (x − β)(4x2
+ 4βx + 4β 2
+ 3).
Vói
β =
1
q
3
m+
√
m2 −1+
q
3
m−
√
m2 −1
có ∆′ = −12(1 − β 2) < 0.
24
Viết đề tài giá sinh viên – ZALO:0973.287.149-TEAMLUANVAN.COM
2
ax
Các phương trình b c 4 đ c bi t
1. Phương trình có dạng
ax4
+ bx2
+ c = 0, a 0. (1.14)
Đ t t = x2
, t ≤ 0 thì đưa ve phương trình b c hai at2
+ bt + c = 0
2. Phương trình có dạng
(x +a)4
+(x+ b)4
= c. (1.15)
Đ t t = x+ a+b
thì đưa ve phương trình trùng phương At4
+Bt2
+C = 0 và
giải như trên.
3. Phương trình có dạng
(ax2
+bx +c)(ax2
+bx +d) = m. (1.16)
Đ t t = ax2
+bx thì đưa ve phương trình b c hai (t +c)(t +d) = m.
4. Phương trình có dạng
(x +a)(x + b)(x +c)(x +d) = m. (1.17)
Neu có a + d = b + c thì ghép c p (x + a)(x + d) và (x + b)(x + c) roi đ t
t = x2
+(a+d)x = x2
+(b+c)x đe đưa ve dạng trên.
5. Phương trình có dạng
ax4
+ bx3
+ cx2
+ dx + e = 0. (1.18)
Neu ad2
= eb2
/= 0 thì chia hai ve cho x2
/= 0 roi đ t t = x+ e
(Phương t
r
ì
n
h
quy hoi mỏ r ng b c bon).
25
Viết đề tài giá sinh viên – ZALO:0973.287.149-TEAMLUANVAN.COM
x
2
p
x x
Phương trình quy hoi (đoi x ng h so)
Xét phương trình
a0xn
+ a1xn−1
+ a2xn−2
+ ··· + an−2x2
+ an−1x + an = 0 (1.19)
trong đó a0 = an; a1 = an−1; a2 = an−2;...
Xét n chȁn, n = 2m. Chia hai ve cho xm
/= 0.. Đ t t = x + 1
, |t| ≥ 2 đưa ve
phương trình b c m = n
.
Xét n lẻ, n = 2m + 1. Phương trình có nghi m x = −1 nên phân tích ra thừa so
(x+1) và thừa so b c 2m lại là phương trình quy hoi b c chȁn như trên.
Đôi khi ta mỏ r ng dạng quy hoi (quy hoi kèm tỉ l ) vói cách đ t
t = x −
1
, t = x +
a
Phương trình b c cao
Xét phương trình
a0xn
+ a1xn−1
+ ··· + an−1x + an = 0, a0 0. (1.20)
Nguyên tac chung là bien đoi ve dạng tích, đ t an phụ đe đưa ve phương trình b c
thap hơn. Đ c bi t
• Neu tong các h so bang 0 thì có nghi m x = 1.
• Neu tong đan dau các h so bang 0 thì có nghi m x = −1 .
• Nhac lại, nghi m hữu tỉ neu có thì có dạng x = q vói p | an và q | a0. The
trục tiep ho c dúng sơ đo Hoocne đe thử nghi m.
Đôi khi phương trình b c cao đoi vói bien x mà lại b c thap đoi vói tham so thì
ta chuyen ve phương trình theo an là tham so đó.
26
Viết đề tài giá sinh viên – ZALO:0973.287.149-TEAMLUANVAN.COM
2
2
2
q
Bài toán 1.3.7. Giái các phương trình sau
(a) 4x3
− 10x2
+ 6x − 1 = 0.
(b) 8x3
− 36x + 27 = 0.
Lời giái. (a) Ta có
4x3
−10x2
+6x −1 = 0 ⇔ 4x3
−2x2
−8x2
+4x+ 2x −1 = 0
⇔ 2x (2x −1)−4x(2x − 1)+(2x −1) = 0
⇔ (2x − 1)(2x −4x +1) = 0
⇔ 2x −1 = 0 ho c 2x − 4x + 1 = 0.
V y nghi m của phương trình là
1
x =
2
, x =
2 ±
√
2
.
2
Bài toán 1.3.8. Tìm quan h gira p và q đe phương trình x3
+ px + q = 0 có
the viet dưới dạng x4
= (x2
− ax + b)2
. Áp dnng ket quá đó đe giái phương trình:
x3
−18x+27 = 0.
Lời giái. Ta có
Suy ra
x4
− (x2
− ax + b)2
= −m(x3
+ px + q).
a2
+ 2b = 0, 2ab = pm, 2a = m,b2
= qm.
Từ đó b = p, kéo theo p2
= mq. V y m = p
2
.
M t khác a2
= −2b ⇒ a = m
⇒ m2
= −2b = −2p ⇒ m2
= −8p. Từ đó p
4
=
2 4 q2
−8 ⇒ p4
+ 8pq2
= 0.
V y quan h giữa p và q: p3
+8q2
= 0.
Ta có
x3
− 18x + 27 = 0 ⇔ x4
= (x2
+ 6x − 18)2
27
Viết đề tài giá sinh viên – ZALO:0973.287.149-TEAMLUANVAN.COM
3 2 2
3 2 2 3
⇔ (6x − 18)(2x2
+ 6x − 18) = 0
x = 3 ho c x =
−3 ±
√
35
.
2
Bài toán 1.3.9. Giái và bi n lu n phương trình
x3
− 3x2
+ 3(a + 1)x − (a + 1)2
= 0. (1.21)
Lời giái. Ta có x3
− 3x2
+ 3(a + 1)x − (a + 1)2
= 0 tương đương vói
−x = −3x +3(a +1)x −(a +1) .
Neu a =
/ −1 thì nhân hai ve vói (a+1), ta đưoc
−x (a + 1) = −3x (a + 1) + 3(a + 1) x − (a + 1)
C ng hai ve vói x3
ta đưoc −ax3
= (x−a−1)3
. Từ đó ta đưoc x−a−1 = −
√
3
ax,
suy ra
a + 1
x = √
3
a+ 1
= a2 −
√
3
a.
Neu a = −1 thì de thay phương trình có hai nghi m x1 = 0 và x2 = 3.
Bài toán 1.3.10. Giái các phương trình sau
(a) x4
− 2x3
− 6x2
+ 16x −8 = 0;
(b) x4
+ x2
+ 4x − 3 = 0;
Lời giái. (a) Ta có
x4
− 2x3
−6x2
+16x − 8 = 0 ⇔ (x− 2)(x3
− 6x +4) = 0
⇔ (x − 2)(x − 2)(x2
+ 2x − 2) = 0
⇔ (x − 2)2
(x2
+2x − 2) = 0.
V y nghi m của phương trình là x = 2, x = −1 ±
√
3.
⇔
√
3
28
Viết đề tài giá sinh viên – ZALO:0973.287.149-TEAMLUANVAN.COM
2 2
2 2
2 2 2
0
x
x
x0 x0
0 2
0 x0
0
(b) Ta có
x4
+x2
+4x−3 = 0 ⇔ x4
= −x2
−4x +3
⇔ (x + 1) = (x −2)
⇔ (x +1) −(x −2) = 0
⇔ (x −x+3)(x + x −1) = 0
V y nghi m của phương trình là x = −1±
2
√
5
.
Bài toán 1.3.11 (IMO 1973). Giá sr phương trình x4
+ ax3
+ bx2
+ ax + 1 = 0 có
nghi m. Tìm giá trị bé nhat của a2
+b2
.
Lời giái. Goi x0 là nghi m của phương trình đã cho, tức là
x4
+ax3
+bx2
+ax0 +1 = 0
0 0 0
Từ đây ta có ket lu n x0 /= 0
.
Chia hai ve cho x2
, ta có
x2
+ ax0 + b +
a
+
1
= 0.
0
Phương trình này tương đương vói
x0
2
x2
+
1
+ a x0 +
1
+b = 0.
Đ t y = x0 + 1
vói đieu ki n |y| = |x0|+| 1
| ≥ 2. Ta có (y2
− 2)+ay +b = 0 Suy
ra
|2− y | = |ay + b| ≤
√
a2
+ b2
√
y2 +1 ⇒ a2
+b2
≥
(2 − y2
)2
1 + y2
Đ t t = y2
, t ≥ 4. Ta chứng minh
(2 −t)2
4
1+t
≥
5
(1.22)
Th t v y, ta có (1.22) tương đương vói 5(2 −t)2
≥ 4(1+t), tức là 5t2
− 24t + 16 ≥
0. Nhưng đieu này đúng vì t ≥ 4. Như v y giá trị bé nhat của a2
+ b2
là 4/5.
2
29
Viết đề tài giá sinh viên – ZALO:0973.287.149-TEAMLUANVAN.COM
√ 2
3 1
0
−3x = 4x −x
x ≥ 2, 4x−x2
≥ 0
x4
− 8x3
+ 16x2
+ 27x − 90 = 0.
Bài toán 1.3.12 (Vi t Nam 2002). Giái phương trình
q
4 − 3
√
10 − 3x = x − 2 (1.23)
Lời giái. Ta có bien đoi tương đương như sau
q
4− 3
√
10 −3x = x −2 ⇔
x ≥ 2
4−3
√
10−3x = (x−2)2
⇔
x ≥ 2
⇔
9(10 − 3x) = (4x − x2
)2
⇔
x ≥ 2, 0 ≤ x ≤ 4
⇔
2 ≤ x ≤ 4
Bây giò ta sẽ giải phương trình
x4
− 8x3
+ 16x2
+ 27x − 90 = 0.
x4
− 8x3
+ 16x2
+ 27x − 90 = 0. (1.24)
Bang cách thử trục tiep ta thay x = 3 là m t nghi m nên phương trình (1.24) đưoc
viet lại thành
(x − 3)(x + 2)(x2
− 7x + 15) = 0.
V y phương trình có nghi m duy nhat x = 3.
Bài toán 1.3.13 (Vi t Nam 1991). Giái phương trình
x3
−3x2
− 8x+ 40 = 8
√
4
4x+4. (1.25)
30
Viết đề tài giá sinh viên – ZALO:0973.287.149-TEAMLUANVAN.COM
2
2
2
8 8 8
8
8
3 3
3 3 8
3 3 3 3
Lời giái. Từ phương trình x3
− 3x2
− 8x+ 40 = 8
√
4
4x+ 4 ta có đieu ki n x ≥ −1.
Áp dụng bat đȁng thức AM-GM ta có
Do đó
8
√
4
4x+4 = 4
√
4
4.4.4.(x + 1) ≤ x + 13.
x3
−3x2
−8x+40 ≤ x +13 ⇔ x3
−3x2
−9x +27 ≤ 0
⇔ (x + 3)(x − 6x + 9) ≤ 0
⇔ (x +3)(x− 3) ≤ 0.
Vì x ≥ −1 nên (x − 3)2
≥ 0, suy ra x = 3. Thử lại ta thay đúng. V y phương trình
có nghi m duy nhat x = 3.
Bài toán 1.3.14. Chŕng minh rang
(a) x =
r
3
a+ a+1
q
8a−1 +
r
3
a− a+1
q
8a−1
với a ≥ 1
là so tự nhiên
(b)
√
3
2+
√
3
4 là so vô tỉ.
Lời giái. (a) Áp dụng hang đȁng thức (u+v)3
= u3
+v3
+3uv(u+v) ta có
x3
= 2a +(1 −2a)x ⇔ x3
+(2a − 1)x − 2a = 0
⇔ (x − 1)(x + x + 2a) = 0.
Xét đa thức b c hai x2
+ x + 2a có ∆ = 1 − 8a ≥ 0
Khi a = 1
, ta có x =
q
3 1 +
q
3 1
= 1
Khi a > 1
, ta có 1 − 8a âm nên đa thức x2
+ x + 2a có nghi m thục duy nhat
x = 1.
V y vói a ≥ 1
thì
x =
s
3
a+
a+ 1
r
8a−1
+
s
3
a−
a+ 1
r
8a−1
31
Viết đề tài giá sinh viên – ZALO:0973.287.149-TEAMLUANVAN.COM
là m t so tụ nhiên.
(b) Đ t x =
√
3
2+
√
3
4. Ta có
x3
= 2+ 4+3
√
3
8(
√
3
2+
√
3
4)
Tức là x3
− 6x − 6.
Giả sử x ∈ Q. Do h so b c cao nhat của đa thức là 1 nên x là so nguyên. Ta có
2 <
√
3
2+
√
3
4 < 4 nên x = 3. Do đó x3
−6x−6 = 3, vô lý. V y x là so vô tý.
Bài toán 1.3.15 (Vi t Nam 1984). Tìm đa thŕc theo x có b c bé nhat với h so
nguyên, biet m t nghi m là
√
2+
√
3
3.
Lời giái. Đ t a =
√
2+
√
3
3 ta có
a2
= 2+ 2
√
2.
√
3
3+
√
3
9
và
Ta rút ra
a3
= 2
√
2+ 6
√
3
3+ 3
√
2.
√
3
9+ 3 (1.26)
√
3
3 = a−
√
2,
√
3
9 = a2
−2−2
√
2(a−
√
2) = a2
+ 2− 2
√
2a.
Thay vào (1.26) ta có
a3
= 2
√
2 + 6(a −
√
2) + 3
√
2(a2
+ 2 − 2
√
2a) + 3,
a3
+ 6a − 3 =
√
2(3a2
+ 2).
Bình phương hai ve ta thay a là nghi m của đa thức
x6
− 6x4
− 6x3
+ 12x2
− 36x + 1
Bang phép đong nhat h so ta chứng minh đa thức trên không phân tích đưoc thành
hai đa thức b c thap hơn có h nguyên nên đa thức trên chính là đa thức có b c bé
nhat thỏa đe bài.
32
Viết đề tài giá sinh viên – ZALO:0973.287.149-TEAMLUANVAN.COM
1.4 Đạo hàm cua đa th c. Định lý Taylor
Đạo hàm của m t hàm so là m t khải ni m thu c nhánh Giải tích toán hoc. Trong
mục này chúng tôi sẽ trình bày ngan gon ve đạo hàm của hàm so đa thức.
Định nghĩa 1.4.1. Cho f(x) ∈ R[x] và deg f(x) = n,
anxn
+ an−1xn−1
+ ... + a1x + a0 vói an 0
• Đa thức
nanxn−1
+ (n − 1)an−1xn−2
+ ... + 2a2x + a1
đưoc goi là đạo hàm cap m t của đa thŕc f (x) (ho c ngan gon là đạo hàm
của đa thŕc f (x)), và đưoc ký hi u là f′(x). Bang quy nạp, ta định nghĩa đạo
hàm cap k của đa thức f (x) là đạo hàm của đạo hàm cap k − 1. Ta ký hi u
đạo hàm cap 2, 3 và cap k tong quát lan lưot là f ′′(x), f ′′′(x), và f (k)(x).
• Vói moi x0 ∈ R, bieu thức sau đây đưoc goi là khai trien Taylor của đa thức
thục f(x)
f
f ′(x0) f (k)(x0) k f (n)(x0) n
(x) = f (x0)+
1!
(x −x0)+...+
k!
(x − x0) +...+
n!
(x − x0) .
Từ định nghĩa này ta có neu deg f = n thì
deg f ′ = n − 1, deg f ′′ = n − 2,...,
deg f (k) = n − k vói 1 ≤ k ≤ n, deg f (x) = 0.
Bây giò ta xét m t so bài toán ve đạo hàm hàm và khai trien Taylor. Chúng tôi
tham khảo [6] trong trình bày.
Bài toán 1.4.2 (Ba Lan 1979). Cho đa thŕc P(x) có b c n > 1 và có n nghi m thực
x1,x2,x3,...,xn phân bi t. Chŕng minh rang
1 1 1
P′(x1)
+
P′(x2)
+ ... +
P′(xn)
= 0.
33
Viết đề tài giá sinh viên – ZALO:0973.287.149-TEAMLUANVAN.COM
−
i=1
P′(x )
i
Lời giái. Đ t
ta suy ra
P(x) = a(x−x1)(x−x2)...(x−xn),a =
/ 0
n
P′(x) = P1(x)+P2(x)+...+Pn(x) vói Pi(x) =
Ta thay
∏
j=1,
j
=
/
(x x j).
i
Pi(xj) = 0 vói moi j /= i kéo theo P′(xj) = Pj(xj)
Xét đa thức
0 vói moi j = 1,2,...,n
F
n
Pi(x)
(x) = ∑ P′(xi)
− 1
có b c không vưot quá n − 1. Vói i = 1,2,...,n ta có F(xi) = Pi(xi) − 1 = 0 suy ra
F(x) có n nghi m phân bi t. V y F(x) = 0. M t khác h so của F(x) đoi vói xn−1
bang 0 nên
a a a
Như v y
P′(x1)
+
P′(x2)
+ .. . +
P′(xn)
= 0.
1 1 1
P′(x1)
+
P′(x2)
+ ... +
P′(xn)
= 0.
Bài toán 1.4.3 (CHDCLB Đức 1974).
(a) Chŕng minh rang không ton tại đa thŕc P(x) đe với moi x ∈ R ta có các bat
đȁng thŕc
P′(x) > P′′(x), (1.27)
P(x) > P′′(x). (1.28)
(b) Khȁng định trên còn đúng không neu thay đői bat đȁng thŕc (1.27) bang
bat đȁng thŕc
P(x) > P′(x). (1.29)
34
Viết đề tài giá sinh viên – ZALO:0973.287.149-TEAMLUANVAN.COM
k
n+1
Lời giái. (a) Neu P(x) là hang so thì P′(x) = P′′(x) = 0, và bat đȁng thức (1.27)
không thỏa mãn.
Giả sử degP(x) = n ≥ 1 khi đó neu n lẻ thì deg(P(x)−P′′(x)) = n là so lẻ, từ
đó P(x)−P′′(x) ≤ 0 vói ít nhat m t điem x ∈ R.
Như v y đoi vói đa thức P(x) không thỏa mãn ho c bat đȁng thức (1.28) ho c
bat đȁng thức (1.27).
V y (a) đưoc chứng minh xong.
(b) Chon P(x) = x2
+ 3. Khi đó vói x ∈ R ta có P(x) − P′(x) ≡ x2
− 2x + 3 > 0 và
P(x) − P′′(x) ≡ x2
+ 1 > 0 nghĩa là khȁng định trên không còn đúng nữa.
Bài toán 1.4.4 (Vi t Nam 1986). Cho f ∈ R[x] có deg f = n và f(k) = 2k
, với
k = 0,1,2,...,n. Tính f(n+1).
Lời giái. Xét đa thức
g(x) = 1+
x
+
x(x − 1)
+ ... +
x(x − 1)(x − 2)...(x − n + 1)
.
1! 2! n!
Khi đó degg = n và
n
g(k) = ∑Ci
= 2k
= f (k)
vói n + 1 giá trị nên f ≡ g.
Do đó
i=0
n
f(n+1) = g(n+1) = ∑Ci
= 2n+1
− 1.
i=0
Bài toán 1.4.5 (Singapore 1978). Cho đa thŕc P(x) b c n và hai so a < b thóa
mãn
P(a) < 0,−P′(a) ≤ 0, P(a) ≤ 0,...,(−1)n
P(n)(a) ≤ 0,
P(b) > 0, P′(b) ≥ 0, P(b) ≥ 0,...,P(n)(b) ≥ 0.
Chŕng minh các nghi m thực của P(x) thu c (a,b).
35
Viết đề tài giá sinh viên – ZALO:0973.287.149-TEAMLUANVAN.COM
Lời giái. Khai trien Taylor ta có
P
P′(b) P(b) 2 P(n)(b) n
(x) = P(b) +
1!
(x − b) +
2!
(x − b) +... +
n!
(x − b) .
Neu x ≥ b thì P(x) > 0 suy ra P(x) không có nghi m x ≥ b.
Tương tụ
P
P′(a) P(a) 2 P(n)(a) n
(x) = P(a) +
1!
(x − a) +
2!
(x − a)
−P′(a) P(a)
+...+
2
n!
(x − a)
(−1)n
(a) n
= P(a)+
1!
(a − x) +
2!
(a − x) + ... +
n!
(a − x)
Neu x < a thì P(x) < 0 suy ra P(x) không có nghi m x ≤ a
V y các nghi m phải thu c (a,b). Bài toán đưoc chứng minh xong. Ta goi ưóc
lưong ve nghi m ỏ trên là ước lượng Newton.
36
Viết đề tài giá sinh viên – ZALO:0973.287.149-TEAMLUANVAN.COM
Chương 2
Đa th c bat kha quy
2.1 Đa th c bat kha quy
Định nghĩa 2.1.1. Cho đa thức f ∈ Z[x]. Ta goi đa thức f là bat khá quy trên Z[x]
neu f không phân tích đưoc thành tích hai đa thức thu c Z[x] vói b c lón hơn hay
bang 1.
Trưòng hop f là m t đa thức hữu tý, ta định nghĩa hoàn toàn tương tụ.
Moi quan h ve tính bat khả quy trên Z[x] và Q[x] đưoc phát bieu như sau:
Định lí 2.1.2. Neu đa thŕc f ∈ Z[x] bat khá quy trên Z[x] thì fcũng bat khá quy
trên Q[x].
Đe chứng minh định lý này, ta can bo đe sau đây.
Trưóc het, ta goi đa thức f ∈ Z[x] là nguyên bán neu các h so nguyên to cùng
nhau.
Bo đe 2.1.3 (Bo đe Gauss). Tích của hai đa thŕc nguyên bán là m t đa thŕc nguyên
bán.
Chŕng minh. Cho hai đa thức nguyên bản
f (x) = a0xn
+ a1xn
− 1 + ... + an,
g(x) = b0xm
+ b1xm−1
+ ... + bm.
37
Viết đề tài giá sinh viên – ZALO:0973.287.149-TEAMLUANVAN.COM
bi
Khi đó
f (x)g(x) = c0xn+m
+ c1xn+m−1
+ ... + cn+m.
Giả sử đa thức tích f (x)g(x) không nguyên bản. Khi đó ton tại m t so nguyên to
p là ưóc chung của các h so c0,...,cn+m
Vì f nguyên bản nên goi ai là so đau tiên mà ai . p.
Vì g nguyên bản nên goi bj là so đau tiên mà bj . p.
Bang cách xét h so theo lũy thừa xi+ j
ta có h so tương ứng không chia het
cho p. Đieu này vô lý. V y f(x)g(x) là đa thức nguyên bản.
Chŕng minh Định lý 2.1.2. Cho f ∈ Z[x] là m t đa thức bat khả quy. Giả sử f khả
quy trên Q[x], tức là ta có bieu dien f (x) = f1(x) f2(x) vói f1, f2 ∈ Q[x], có b c lón
hơn ho c bang 1.
Đ t
f1(x) =
a1
g1(x), f2(x) =
a2
g2(x)
b1 b2
voi ai
toi giản và g1(x), g2(x) nguyên bản. Khi đó
f (x) = f1(x) f2(x) =
a1a2
g1(x)g2(x) =
p
g1(x)g2(x) vói (p,q) = 1.
b1b2 q
Do đó f ∈ Z[x] nên moi h so của khai trien tích g1(x)g2(x) đeu là b i so của q.
Suy ra đa thức tích g1(x)g2(x) không nguyên bản. Đieu này trái vói ket quả của Bo
đe Gauss. V y f bat khả quy trên Q[x].
2.1.1 Đa th c vỚi h so th c và ph c
Cho m t đa thức vói h so thục thì chưa chac đa thức đó có nghi m trong trưòng
so thục, cụ the đa thức x2
+ 1 không có nghi m trong trưòng so thục. Dưói đây ta
sẽ thay moi đa thức b c n vói h so phức có đúng n nghi m phức. Đe chứng minh
ra hãy đưa vào các bo đe sau đây:
Bo đe 2.1.4. Moi đa thŕc với h so thực có b c lé có ít nhat m t nghi m thực.
38
Viết đề tài giá sinh viên – ZALO:0973.287.149-TEAMLUANVAN.COM
Chŕng minh. Giả sử
f (x) = anxn
+ an−1xn−1
+ .... + a0, an 0 và n lẻ.
Qua giáo trình giải tích ta biet rang vói những giá trị dương và âm của x, khá lón
ve giá trị tuy t đoi, hàm so f (x) có các dau trái nhau. V y có những giá trị thục
của x, a và b chȁng hạn, sao cho.
f (a) < 0, f (b) > 0.
M t khác hàm so f (x) là liên tục, vì v y có m t giá trị c của x, nam giữa a và b,
sao cho f(x) = 0.
Bo đe 2.1.5. Moi đa thŕc b c hai ax2
+ bx + c, với h so phŕc, bao giờ cũng có
hai nghi m phŕc.
Bo đe 2.1.6. Moi đa thŕc b c lớn hơn 0 với h so thực có ít nhat m t nghi m phŕc.
Định lí 2.1.7. Moi đa thŕc b c lớn hơn 0 với h so phŕc có ít nhat m t nghi m
phŕc.
Chŕng minh. Giả sử f (x) là m t đa thức b c n > 0
f (x) = a0 +a1x+.....+ anxn
vói h so phức. Đ t
f (x) = a0 +a1x+.....+ anxn
vói các ai là các liên hop của các ai vói i = 0,...,n. Xét đa thức
g(x) = f (x) f (x).
Ta có
Vói
g(x) = b0 +b1x + .... + b2nx2n
bk = ∑
i+ j=k
aiaj, k = 0,1,.....,2n.
39
Viết đề tài giá sinh viên – ZALO:0973.287.149-TEAMLUANVAN.COM
Vì
bk = ∑
i+ j=k
aiaj = bk
nên các h so bk là thục. Theo Bo đe 2.1.6 g(x) có ít nhat m t nghi m phức
z = s + it,
g(z) = f (z) f (z) = 0.
Do đó ho c f (x) = 0 ho c f (z) = 0. Neu f (z) = 0,
f (z) = a0 +a1z+..... + anzn
= 0.
thì
a0 + a1z + ...anzn = a0 + a1z + ... + anzn = 0,
tức là f (z) = 0. Như v y ho c z ho c z là nghi m của f (x).
H qua 2.1.8. Các đa thŕc bat khá quy của vành C[x], C là trường so phŕc, là các
đa thŕc b c nhat.
Chŕng minh. Các đa thức b c nhat là bat khả quy. Giả sử f (x) là m t đa thức của
C[x] có b c lón hơn 1. Theo Định lí 2.1.7, f (x) có m t nghi m phức c. V y f (x)
có m t ưóc thục sụ x−c, do đó f(x) không bat khả quy.
H qua 2.1.9. Moi đa thŕc b c n > 0 với h so phŕc có n nghi m phŕc.
H qua 2.1.10. Các đa thŕc bat khá quy của R[x], R là trường so thực, là các đa
thŕc b c nhat và các đa thŕc b c hai ax2
+bx+c với bi t so b2
−4ac < 0.
Chŕng minh. Các đa thức b c nhat và các đa thức b c hai vói bi t so âm rõ ràng
là những đa thức bat khả quy của R[x]. Giả sử p(x) là m t đa thức bat khả quy của
R[x] vói b c lón hơn m t. V y p(x) không có nghi m thục. Theo Định lí 2.1.7,
p(x) có m t nghi m phức z và p(x) chia het cho đa thức b c hai vói h so thục.
g(x) = x2
− (z + z)x + zz.
40
Viết đề tài giá sinh viên – ZALO:0973.287.149-TEAMLUANVAN.COM
Đa thức g(x) không khả nghịch và là ưóc của phan tử bat khả quy p(x), v y g(x)
phải là liên ket của p(x), tức là
p(x) = ug(x), 0 /= u ∈ R.
Ta có đieu can chứng minh.
2.1.2 Đa th c bat kha quy cua vành Q[x]
Đoi vói trưòng so thục R và trưòng so phức C, van đe xét xem m t đa thức đã cho
của vành R[x] hay C[x] có bat khả quy hay không rat đơn giản, nhưng trong vành
Q[x] vói Q là trưòng so hữu tỉ thì van đe phức tạp hơn nhieu. Đoi vói các đa thức
b c hai và ba của Q[x], vi c xét xem có bat khả quy hay không đưoc đưa ve vi c
tìm nghi m hữu tỉ của đa thức đó. Các đa thức b c hai và b c ba của Q[x] là bat
khả quy khi và chỉ khi chúng không có nghi m hữu tỉ. Đoi vói các đa thức b c lón
hơn ba thì van đe phức tạp hơn nhieu. Chȁng hạn đa thức x4
+2x2
+1 = (x2
+1)2
rõ ràng không có nghi m hữu tỉ nào, nhưng nó có m t ưóc thục sụ x2
+ 1 , v y
không phải là bat khả quy.
Ta đã biet, moi đa thức f(x) vói h so hữu tỉ đeu có the viet dưói dạng
f (x) = b−1
g(x)
trong đó b là m t so nguyên khác 0, g(x) là m t đa thức vói h so nguyên. Trong
vành Q[x], f(x) và g(x) là liên ket v y f(x) là bat khả quy khi và chỉ khi g(x) là bat
khả quy. Do đó tiêu chuan Eisenstein mà ta đưa ra dưói đây đe xét m t đa thức của
Q[x] có bat khả quy hay không là tiêu chuan cho các đa thức vói h so nguyên.
Bo đe 2.1.11. Neu f (x) là m t đa thŕc với h so nguyên có b c lớn hơn 0 và f (x)
không bat khá quy trong Q[x], thì f (x) phân tích được thành m t tích nhrng đa
thŕc b c khác 0 với h so nguyên.
Trong nghiên cứu các đa thức bat khả quy, Tiêu chuan Eisenstein sau đây là
đ c bi t quan trong.
41
Viết đề tài giá sinh viên – ZALO:0973.287.149-TEAMLUANVAN.COM
Định lí 2.1.12 (Tiêu chuan Eisenstein). Cho f (x) ∈ Z[x], deg f(x) = n,
f (x) = a0xn
+ a1xn
− 1 + ... + an.
Neu có so nguyên to p thóa mãn ba đieu ki n
(1) a0 không chia het cho p;
(2) a1,a2,...,an chia het cho p;
(3) an không chia het cho p2
.
thì đa thŕc f(x) bat khá quy trên Q[x].
Chŕng minh. Giả sử f(x) có những ưóc thục sụ trong Q[x] theo Bo đe 2.1.11, f(x)
có the viet
trong đó
Ta có
f (x) = g(x)h(x),
g(x) = b0 + b1x + ... + brxr
, bi ∈ Z, 0 < r < n,
h(x) = c0 + c1x + ... +s xs
, ci ∈ Z, 0 < s < n.
a0 = b0c0,
a1 = b1c0 + b0c1,
...
ak = bkc0 + bk−1c1 + ... + b0ck,
an = brcs.
Theo giả thiet p chia het a0 = b0c0; v y vì p là nguyên to nên ho c p chia het cho
b0 ho c p chia het cho c0. Giả sử p chia het b0 the thì p không chia het c0, vì neu
the thì p2
sẽ chia het a0=b0c0, trái vói giả thiet, p không the chia het moi h so
42
Viết đề tài giá sinh viên – ZALO:0973.287.149-TEAMLUANVAN.COM
của g(x), vì neu the thì p sẽ chia het an = brcs, trái vói giả thiet. V y giả sử bk là
h so đau tiên của g(x) không chia het cho p. Ta hãy xét
ak = bkc0 + bk−1c1 + ... + b0ck,
trong đó ak, bk−1, . . . , b0 đeu chia het cho p. V y bkc0 phải chia het cho p. Vì p là
nguyên to, ta suy ra ho c bk chia het cho p, ho c c0 chia het cho p, mâu thuȁn vói
giả thiet ve bk và c0.
2.2 M t so bài toán đien hình
Bài toán 2.2.1 (IMO 1993). Cho n ∈ N và n > 1. Chŕng minh rang đa thŕc
f (x) = xn
+5xn−1
+3
bat khá quy trên Z[x].
Lời giái. Vói n = 2 có f (x) = x2
+ 5x + 3 thì bat khả quy trên Z[x].
Xét n ≥ 3. Giả sử f (x) = g(x)h(x) vói g(x), h(x) ∈ Z[x] và có b c lơn hơn ho c
bang 1. Do degg(x)+degh(x) = n ≥ 3 nên suy ra trong hai so degg(x) và degh(x)
có m t so lơn hơn 1.
Vì f(0) = 3 là so nguyên to nên ho c |g(0)| = 1 ho c |h(0)| = 1. Giả sử
g(x) = xk
+ b1xk−1
+ ... + bk vói k > 1
và |g(0)| = 1.
Goi a1,a2,...,ak là các nghi m (nói chung là nghi m phức) của g(x). Khi đó
ta có
Vì |g(0)| = 1 nên
g(x) = (x − a1)(x − a2)...(x − ak).
|a1a2 ...ak| = 1. (2.1)
43
Viết đề tài giá sinh viên – ZALO:0973.287.149-TEAMLUANVAN.COM
k
i=1
2
2
2 2
Do g(ai) = 0 nên f(ai) = 0, vói i ∈ {1,...,k}.
Nhân các đȁng thức đó lại và sử dụng (2.1) ta đưoc
|(a1 + 5)(a2 + 5)...(ak + 5)| = 3 (2.2)
M t khác ta có
g(−5) = |(a1 + 5)(a2 + 5)...(ak + 5)| và 3 = f (−5) = g(−5)h(−5)
nên |(a1 +5)(a2 +5)...(ak +5)| nh n giá trị 1 ho c 3.
Đieu này trái vói (2.2) vì k > 1. Từ đó ta có ta có đieu phải chứng minh.
Bài toán 2.2.2 (Dụ tuyen IMO). Cho đa thŕc f(x) = ∏n
(x−ai)−2 với n ≥ 3 và
a1,a2,...an là các so nguyên đôi m t khác nhau. Chŕng minh rang neu f (x) khá
quy trên Z[x] thì n = 3.
Lời giái. Giả sử f(x) = g(x)h(x) vói g(x), h(x) ∈ Z[x] và degg = p ≥ q = degh ≥ 1.
Ngoài ra, giả sử các h so b c cao nhat của g và h đeu bang 1.
Trong phép đ t cho x = ai ta sẽ nh n đưoc f (ai) = g(ai)h(ai) = −2. Như v y
ta có
(g(ai),h(ai)) ∈ {(−2,1),(2,−1),(1,−2),(−1,2)}
V y vói moi i, ta có g(ai) + h(ai) = +1 vói ít nhat n
giá trị của i, ho c g(ai) +
h(ai) = −1 vói ít nhat n
giá trị của i.
Giả sử g(ai) + h(ai) =1 vói k giá trị của i ∈ {1,...,n}. Rõ ràng k < n (vì đa
thức g(x)+h(x)−1 0 và có b c nhỏ hơn thục sụ n.
Giả sử k ≥ n
≥ 2, vì neu k < n
thì xét đa thức g(x)+h(x)+1.
Giả sử g(ai)+h(ai) = 1 vói i ∈ {1,...,k}, neu không như v y thì ta đ t lại thứ
tụ của chỉ so.
Suy ra
g(x) + h(x) − 1 = (x − a1)(x − a2)...(x − ak)w(x) vói w(x) ∈ Z[x].
44
Viết đề tài giá sinh viên – ZALO:0973.287.149-TEAMLUANVAN.COM
Cho x = ak+1 vói chú ý g(ak+1)+h(ak+1) = −1 ta đưoc
−2 = (ak+1 −a1)(ak+1 −a2)...(ak+1 − ak)w(ak+1).
The nhưng trị tuy t đoi của m t tích gom ít nhat là bon thừa so nguyên to khác
nhau và khác 0 thì luôn luôn lón hơn 2. V y nên k ≤ 3 và dȁn đen 2 ≤ k ≤ 3.
Xét k = 3. Neu n > 4 thì ta có
(a4 −a1)(a4 − a2)(a4 −a3)w(a4) = −2;
(a5 −a1)(a5 − a2)(a5 −a3)w(a5) = −2
Do đó {a1,a2,a3} ≡ {a4 ±1,a4 ±2,a4 −2}. Nhưng a5 > a4 > a3 nên |a5 −a3| ≥ 3
vói moi i ∈ {1,2,3}. Đieu này vô lý.
Neu n = 4 thì
g(x)+h(x) − 1 = (x− a1)(x −a2)w(x)
do degw ≥ 1 do degg ≤ 3 và degh ≤ 3. Suy ra (ai − a1)(ai − a2)w(ai) = −2 vói
i = 3,4.
Neu degw = 1 thì degg(x) = 3, h(x) = x − a và
0 = f (a) = (a − a1)(a − a2)(a − a3)(a − a4) = 2.
V y w(x) là đa thức hang. Do đó degg = degh = 2, w(x) ≡ 2 và w(a3) = w(a4) = 2.
Từ đó
−1 = (a3 −a1)(a3 −a2) và {a1,a2} ≡ {a3 −1,a3 + 1}.
Tương tụ ta cũng có −1 = (a4 −a1)(a4 −a2) nên {a1,a2} ≡ {a4 − 1,a4 + 1}. Do
đó a4 = a3 vô lý.
Tóm lại, chỉ có n = 3 thỏa mãn yêu cau của bài toán.
Bài toán 2.2.3. Cho n so ai ∈ Z. Chŕng minh rang đa thŕc
P(x) = (x − a1)(x − a2)...(x − an) − 1
bat khá quy trong Z[x].
45
Viết đề tài giá sinh viên – ZALO:0973.287.149-TEAMLUANVAN.COM
Lời giái. Giả sử ta có phan tích P(x) = G(x)H(x) trong Z[x] và degG < n, degH <
n.
Ta có P(ai) = G(ai)H(ai) = −1. V y G(ai) và H(ai) nh n hai giá trị ±1 nên
G(ai)+H(ai) = 0 Khi đó đa thức G(x)+H(x) có deg(G+H) < n mà có n nghi m.
Suy ra G(x) + H(x) ≡ 0, tức là G(x) = −H(x). Do đó P(x) = −[H(x)]2
. So sánh
h so của xn
của hai ve thì ve trái lón hơn không, ve phải nhỏ hơn không, đieu này
vô lý, nên ta có đieu phải chứng minh.
Bài toán 2.2.4. Cho n so ai ∈ Z. Chŕng minh rang đa thŕc
P(x) = (x − a1)2
(x − a2)2
...(x − an)2
+ 1
bat khá quy trong Z[x].
Lời giái. Vì P(x) > 0 vói moi x nên đa thức P(x) vô nghi m. Suy ra hai nhân tử
G(x) và H(x) cũng vô nghi m nên không đoi dau.
Giả sử G(x) và H(x) dương vói moi x. Ta có H(ai)G(ai) = 1 nên suy ra H(ai) =
1 và G(ai) = 1. Neu degH < n suy ra H(x)−1 có n nghi m. Kéo theo rang H(x) ≡
1. Đieu này vô lý, nên degH = degG = n, suy ra
H(x) − 1 = A(x − a1)...(x − an),
G(x) − 1 = B(x − a1)...(x −an).
The vào ta có
(x − a1)2
(x − a2)2
...(x − an)2
+ 1
= [1+ A(x −a1)...(x − an)][1+ B(x −a1)...(x −an)].
So sánh h so của x2n
thì ta có 1 = AB. So sánh h so tụ do
a2
a2
...a2
+ 1 = 1 + (A + B)a1a2 ...an + a2
a2
...a2
.
1 2 n 1 2 n
Suy ra A + B = 0. Do đó 1 = AB = −A2
. Đieu này vô lý. V y bài toán đưoc chứng
minh.
46
Viết đề tài giá sinh viên – ZALO:0973.287.149-TEAMLUANVAN.COM
1
1
1
Chương 3
M t so chu đe khác
3.1 Đa th c nhieu bien
Bài toán 3.1.1. Cho đa thŕc f (x,y) = ax2
+ 2bxy + 2y2
luôn luôn dương. Chŕng
minh
( f (x1,y1, f (x2,y2))2 f (x1 −x2 −y2) ≥ (ac −b2
với moi x1,x2,y1,y2.
)(x1y2 − x)
Lời giái. Từ giả thiet ta có b2
− ac < 0 hay ac − b2
> 0. Ta có đong nhat thức
(ax1
2
+ 2bx1y1 + cy1
2
)(ax2
2
+ 2bx2y2 + cy2
2
)
(ax1x2 + by1y2 + bx2y1 + cy1y2)2
+ (ac − b2
)(x1y2 − x2y1)2
. (3.1)
Đ t
E1 = f (x1,y1) > 0,E2 = f (x2,y2) > 0
F = |ax1x2 +bx1y2 +bx2y1 +cy1y2)| ≥ 0.
Từ (3.1) suy ra E1E2 = F2
+ (ac − b2
)(x1y2 −x2y1)2
. Mà f(x1 − x2,y1 − y2) =
E1 +E2 ±2F. Do đó
( f (x1,y1), f (x2,y2))2 f (x1,x2,y1 − y2)
= (E1E2)2 .(E1 + E2 − 2F)
≥ (E1E2) .(2(E1E2)2
1
— 2F)
= 2E1E2 − 2((E1E2)2 .F
2
47
Viết đề tài giá sinh viên – ZALO:0973.287.149-TEAMLUANVAN.COM
!
1 1 2 2 2
1 2 2
2 2 2 2 2 2
1
= 2F + 2(ac − b) (x1y2 − 22y1) − 2F(F + (ac − b )(x1y2 − x2y1) 2
ac b2
x y 1
x y 2 2
= 2F2
+ 2(ac− b)2
(x1y2 −x2y1)2
−2F2
1+ — ( 1 2 − 2 1)
≥ 2F2
+ 2(ac − b)2
(x(y2x2y1)2
— 2F 2
1+
F2
(ac − b2
)(x1y2 − x2y1)2
2F2
= (ac − b2
)(x1y2 − x2y1)2
V y
( f (x ,y , f (x ,y ))
1
f (x − x − y ) ≥ (ac − b2
vói moi x1, x2,y1, y2.
Bài toán 3.1.2 (Olympic Sinh viên Toàn quoc 2010).
)(x1y2 − x)
(a) Chŕng minh rang ŕng với mői so nguyên dương n, bieu thŕc xn
+ yn
+ zn
có
the bieu dien dưới dạng đa thŕc Pn(s, p,q) b c không quá n của s = x+y+z,
p = xy+yz+zx, q = xyz.
(b) Hãy tìm tőng các h so của đa thŕc P2010(s, p,q).
Lời giái. (a) Ký hi u Sn = xn
+yn
+zn
. Trưóc het ta chứng minh
Sn = sSn−1 − pSn−2 + qSn−3. (3.2)
Th t v y, ta có
Sn = sSn−1 − pSn−2 + qSn−3
= (x + y + z)(xn−1
+ yn−1
+zn−1
)
−(xy+yz+ zx)(xn−2
+ yn−2
+zn−2
)+xyz(xn−3
+yn−3
+zn−3
)
= xn
+yn
+zn
+xyn−1
+xn−1
y+xzn−1
+zxn−1
+yzn−1
+zyn−1
— xn−1
y + xyn−1
+ xn−1
z + xzn−1
+ yzn−1
y + xyzn−2
+
+xyn−2
z+xn−2
yz + (xyzn−2
+xyn−2
z +xn−2
yz)
= xn
+yn
+zn
= Sn.
!
48
Viết đề tài giá sinh viên – ZALO:0973.287.149-TEAMLUANVAN.COM
Ta sẽ chứng minh m nh đe : “bieu thức xn
+yn
+zn
có the bieu dien dưói dạng đa
thức Pn(s, p,q) b c không quá n của s = x + y + z, p = xy + yz + zx, q = xyz.” bang
phương pháp quy nạp.
Ta có
S1 = x + y + z = s
S2 = x2
+y2
+z2
= (x+y+z)2
−2(zy+yz+zx)
= s2
− 2p,
S3 = x3
+y3
+z3
= (x+y+z)3
−3(x+y+z)(xy+yz+zx)+3xyz
= s3
− 3sp + 3q.
V y m nh đe đúng vói n = 1, n = 2, n = 3.
Giả sử m nh đe đúng vói n = k − 1, n = k − 2, n = k − 3. Khi đó theo (3.2),
suy ra m nh đe cũng đúng vói n = k. Do đó ta có đieu phải chứng minh.
(b) Giả sử x2010
+ y2010
+ z2010
= P2010(s, p,q). Ta can tìm tong các h so của
P2010(s, p,q), tức là tính P2010(1,1,1).
Xét h phương trình
x + y + z = 1
xy + yz + zx = 1
xyz = 1
Khi đó theo Định lí Viet suy ra x, y, z là nghi m của phương trình t3
−t2
+t −1 = 0,
tức là t ∈ {1,i,−i}. V y chỉ can chon x = 1,y = i,z = −i, ta đưoc tong các h so
của đa thức P2010(s, p,q) là
P2010(1,1,1) = 12010
+i2010
+(−i)2010
= −1.
49
Viết đề tài giá sinh viên – ZALO:0973.287.149-TEAMLUANVAN.COM
3.2 Đa th c đoi x ng
Những bài toán ve giải phương trình, h phương trình b c cao, chứng minh bat
đȁng thức nhieu bien so,. . . thưòng rat phức tạp. M t trong những dạng nhieu bien
quan trong và đep đẽ là các đa thức nhieu bien mà trong đó vai trò của các bien so
là bình đȁng. Ta sẽ goi những đa thức như v y là các đa thŕc đoi xŕng.
Mục này chúng tôi sẽ dành đe nghiên cứu các đa thức đoi xứng nhieu bien, mà
t p trung chi tiet vào trưòng hop hai bien, các trưòng hop ba bien và nhieu bien
cũng sẽ đưoc đe c p nhưng vói mức đ và phạm vi hạn che hơn. Chúng tôi dụa
vào các tài li u [4, 6] đe trình bày mục này.
Phan này ta sẽ nghiên cứu m t so bài toán chia đa thức đoi xứng. Tài li u tham
khảo chính là [4].
Bài toán 3.2.1. Chŕng minh rang đa thŕc x2n
− xn
yn
+ y2n
chia het cho x2
+ xy + y2
khi và chỉ khi n không phái là b i của 3.
Lời giái. Sử dụng các công thức khai trien
x3
− y3
= (x − y)(x2
+ xy + y2
),
xk
− yk
= (x − y)(xk−1
+ xk−2
y + ... + xyk−2
+ yk−1
)
ta thay x3k
− y3k
chia het cho x2
+ xy + y2
. Ta xét các trưòng hop sau
1. n = 3k. Ta có
x2n
+ xn
yn
+ y2n
= x6k
+ x3k
y3k
+ y6k
= (x6k
− y6k
) + (x3k
− y3k
) + 3y6k
.
Từ đó suy ra x2n
−xn
yn
+y2n
không chia het cho x2
+xy+y2
.
2. n = 3k + 1. Ta có
x2n
+ xn
yn
+ y2n
= x6k+2
+ x3k+1
y3k+1
+ y6k+2
50
Viết đề tài giá sinh viên – ZALO:0973.287.149-TEAMLUANVAN.COM
= x2
(x6k
− y6k
) + xy3k+1
(x3k
− y3k
) + y6k
(x2
+ xy + y2
).
Như v y đa thức x2n
−xn
yn
+y2n
chia het cho x2
+xy+y2
.
3. n = 3k + 2. Ta có
x2n
+ xn
yn
+ y2n
= x6k+4
+ x3k+2
y3k+2
+ y6k+4
= x4
(x6k
− y6k
) + x2
y3k+2
(x3k
− y3k
)
+y6k
(x2
+ x2
y2
y4
)(x2
− xy + y2
).
Như v y x2n
−xn
yn
+y2n
chia het cho x2
+xy+y2
.
Tóm lại, đa thức x2n
−xn
yn
+ y2n
chia het cho x2
+ xy + y2
khi và chỉ khi n không
phải là b i của 3.
Bài toán 3.2.2. Chŕng minh rang với moi n ∈ Z+, đa thŕc x2n
− xn
yn
+ y2n
không
chia het cho x2
+xy+y2
.
Lời giái. Giả sử x2n
− xn
yn
+ y2n
chia het cho x2
+ xy + y2
, tức là
x2n
−xn
yn
+y2n
= (x2
+xy+y2
)q(x,y),
trong đó q(x, y) là đa thức đoi xứng vói h so nguyên (do h so b c cao nhat của
đa thức chia bang 1, còn các h so của đa thức bị chia và đa thức chia là các so
nguyên). Trong đȁng thức trên cho x = y = 1, ta đưoc 1 = 3q(1, 1), vô lí vì q(1, 1)
là m t so nguyên. Đieu này chứng tỏ đa thức x2n
− xn
yn
+ y2n
không chia het cho
x2
+xy+y2
.
Bài toán 3.2.3. Với n ∈ Z+ nào thì x2n
+xn
yn
+y2n
không chia het cho x2
+xy+y2
?
Lời giái. Giả sử
x2n
+xn
yn
+y2n
= (x2
−xy+y2
)q(x,y). (3.3)
trong đó q(x,y) là đa thức đoi xứng vói h so nguyên.
Ta xét hai trưòng hop
51
Viết đề tài giá sinh viên – ZALO:0973.287.149-TEAMLUANVAN.COM
1. n là so lẻ. Trong đȁng thức (3.3) thay x bỏi −x ta đưoc
x2n
− xn
yn
+ y2n
= (x2
+ xy + y2
)q(−x, y).
Theo Bài toán 3.2.2 đȁng thức này không the xảy ra.
2. n là so chȁn. Trong (3.3) thay x bỏi −x, ta đưoc.
x2n
+xn
yn
+ y2n
= (x2
+xy+y2
)q(−x,y).
Theo ví dụ Bài toán 3.2.1 thì đȁng thức trên đúng khi và chỉ khi n = 3m + 1
ho c n = 3m+2.
Neu n = 3m + 1 thì do n là so chȁn, nên m phải là so lẻ, hay m = 2k + 1, do
đó n = 6k +4.
Neu n = 3k + 2, thì do n là so chȁn, nên m phải là so chȁn, hay m = 2k, do
đó n = 6k +2.
V y x2n
+xn
yn
+y2n
chia het cho x2
−xy+y2
khi và chỉ khi n = 6k+2 ho c
n = 6k + 4, vói k ∈ Z, n ∈ Z+.
Bài toán 3.2.4. Với n ∈ Z+ nào thì x2n
− xn
yn
+ y2n
chia het cho x2
− xy + y2
?
Lời giái. Giả sử
x2n
− xn
yn
+ y2n
= (x2
− xy + y2
)q(x,y), (3.4)
trong đó q(x,y) là đa thức đoi xứng vói h so nguyên. Xét hai trưòng hop
1. n là so chȁn. Trong (3.4) thay x bỏi −x, ta đưoc
x2n
− xn
yn
+ y2n
= (x2
+ xy + y2
)q(−x, y).
Theo Bài toán 3.2.2 đȁng thức này không the xảy ra.
2. n là so lẻ. Trong (3.4) thay x bỏi −x, ta đưoc
x2n
+xn
yn
+ y2n
= (x2
+xy+y2
)q(−x,y).
52
Viết đề tài giá sinh viên – ZALO:0973.287.149-TEAMLUANVAN.COM
Theo Bài toán 3.2.1, đȁng thức trên đúng khi và chỉ khi n = 3m + 1 ho c
n = 3m + 2.
Neu n = 3m + 1 thì do n là so lẻ, nên m phải là so chȁn, tức là m = 2k và khi
đó n = 6k +1.
Neu n = 3m + 2, thì do n là so lẻ, nên m phải là so lẻ, tức là m = 2k − 1. Khi
đó n = 6k − 1. V y x2n
− xn
yn
+ y2n
chia het cho x2
− xy + y2
khi và chỉ khi
n = 6k ±1, k ∈ Z, n ∈ Z+.
Bài toán 3.2.5. Xác định n đe (x + y)n
+ xn
+ yn
chia het cho x2
+ xy + y2
.
Lời giái. Giả sử (x+y)n
xn
+yn
chia het cho x2
+xy+y2
. Khi đó ta có
(x + y)n
xn
+ yn
= (x2
+ xy + y2
)q(x,y) (3.5)
trong đó q(x,y)là đa thức đoi xứng vói h so nguyên. Trong (3.5) thay x, y tương
ứng bỏi x2
, y2
, ta có
(x2
+ y2
)n
+ x2n
+ y2n
= (x4
+ x2
y2
+ y4
)q(x2
, y2
)
= (x2
+ xy + y2
)(x2
− xy + y2
)q(x2
,y2
) (3.6)
Đȁng thức (3.6) chứng tỏ (x2
+y2
)n
+x2n
+y2n
phải chia het cho x2
− xy+y2
. Ta
có
(x2
+ y2
)n
−(xy)n
= (x2
+y2
−xy)[(x2
+y2
)n−1
+ (x2
+y2
)n−2
+...
+(x2
+ y2
)(xy)n−2
+ (xy)n−1
]. (3.7)
Tiep theo ta có
(x2
+ y2
)n
+ x2n
+ y2n
= [(x2
+ y2
)n
− (xy)n
] + (x2n
+ xn
yn
+ y2n
). (3.8)
Từ (3.7) và (3.8) suy ra (x2
+ y2
)n
+ x2n
+ y2n
chia het cho x2
− xy + y2
khi và chỉ
khi x2n
+xn
yn
+y2n
chia het cho x2
−xy+y2
. Theo Bài toán 3.2.3 đieu này có đưoc
khi và chỉ khi n = 6k + 2 ho c n = 6k + 4 vói k ∈ Z, n ∈ Z+.
53
Viết đề tài giá sinh viên – ZALO:0973.287.149-TEAMLUANVAN.COM
Ngưoc lại, giả thiet rang n = 2m, vói m = 3k + 1 ho c m = 3k + 2. The thì
(x + y)n
+ xn
+ yn
= (x + y)2m
+ x2m
+ y2m
= [(x + y)2m
− (xy)m
] + (x2m
+ xm
ym
+ y2m
).
Đe ý rang
(x + y)2m
− (xy)m
= [(x + y)2m
− (xy)m
]
= [(x + y)2 − xy]p(x, y) = (x2
+ xy + y2
)p(x, y),
trong đó p(x,y) là đa thức đoi xứng vói h so nguyên. Do đó (x + y)2m
− (xy)m
chia het cho x2
+xy + y2
. M t khác vì m = 3k +1, m = 3k + 2 nên theo Bài toán
3.2.1 đa thức x2m
+xm
ym
+y2m
chia het cho x2
+xy+y2
.
V y đa thức (x+y)n
+xn
+yn
chia het cho x2
+xy+y2
khi và chỉ khi n = 6k+2
ho c n = 6k +4, vói k ∈ Z, n ∈ Z+.
3.3 Phương trình hàm đa th c
Ő khía cạnh giải tích toán hoc, đa thức là m t hàm so rat đ c bi t. Ta có the tính
giá trị hàm so chỉ bang các phép toán c ng, trừ, nhân. Vi c tính đạo hàm, tích phân
cũng rat de. M t bài toán quan trong trong lý thuyet các đa thức là bài toán giải
phương trình hàm đa thức. M t cách ngan gon, phương trình hàm đa thức là các
phương trình mà an hàm của nó là các đa thức đại so . . .
Lu n văn này sẽ dành phan này đe tìm hieu m t so bài toán ve phương trình
hàm đa thức xuat hi n trong các kỳ thi Olympic. Những chủ đe sâu sac hơn ve lĩnh
vục này, chúng ta có the tham khảo Nguyen Văn M u [2].
Bài toán 3.3.1 (Đe thi chon đ i tuyen TP.HCM năm 2006-2007). Tìm tat cá các
đa thŕc thóa mãn
P2
(2x) = 4[P(x2
)− xP(2x)] với moi x ∈ R.
54
Viết đề tài giá sinh viên – ZALO:0973.287.149-TEAMLUANVAN.COM
Lời giái. Ta có phương trình đã cho tương đương vói
[P(2x)+2x]2
= 4[P(x2
) +x2
] vói moi x ∈ R.
Đ t F(x) = P(x)+x, phương trình hàm đã cho trỏ thành
F2
(2x) = 4F(x2
)2
.
Đ t d = degF(x) thì ta có d2
= d. Từ đây suy ra d = 0 ho c d = 1.
• Khi d = 0 thì F(x) = const = c. Thay vào phương trình đã cho ta có
c2
= 4c.
Tức là ta có F(x) ≡ 0 ho c F(x) ≡ 4. Như v y
P(x) = 0 ho c P(x) = 4 −x.
• Khi d = 1 thì ta có F(x) = ax + b vói a /= 0. Thay vào phương trình và thu
gon hai ve ta có
4a2
x2
+4abx+b2
= 4ax2
+4b.
Đong nhat h so ta đưoc a = 0, b = 1. Như v y F(x) = x, tức là P(x) ≡ 0.
Tóm lại nghi m của phương trình đã cho là P(x) = 0, P(x) = 4−x, P(x) ≡ 0.
Bài toán 3.3.2 (Rumani 1980). Tìm tat cá các đa thŕc thóa mãn P(x2
) = [P(x)]2
với moi x ∈ R.
Lời giái. Giả sử đa thức can tìm có dạng
P(x) = anxn
+ an−1xn−1
+ ... + a1x + a0 vói an 0.
Giả thiet rang m t trong các h so an−1,an−2,...,a0 là khác không. Goi k, vói
k < n, là so lón nhat sao cho ak /= 0. Khi đó ta có
P(x2
) = anx2n
+akx2k
+... +a1x + a0
55
Viết đề tài giá sinh viên – ZALO:0973.287.149-TEAMLUANVAN.COM
n
x
= (anx2n
+ akx2k
+ ... + a1x + a0)=[P(x)]2
.
Đong nhat h so ta nh n đưoc 0 = 2anak. Đieu này trái vói giả thiet ak
an−1 = an−2 = ... = a0 = 0.
0. Suy ra
V y P(x) = anxn
.
Từ đieu ki n
anx2n
= P(x2
) = [P(x)]2
= a2
x2n
ta nh n đưoc an = 1. V y P(x) = xn
là đa thức can tìm.
Bài toán 3.3.3 (Rumani 1980). P(x2
− 2x) = [P(x)]2
với moi x ∈ R.
Lời giái. Đ t y = x − 1, Q(y) = P(y − 1). Khi đó
[P(x − 2)]2
= [P(y − 1)]2
= [Q(y)]2
,
(x2
−2x) = P(y2
−1) = Q(y2
).
Do đó P(x2
− 2x) = [P(x−2)]2
vói moi x ∈ R. V y Q(y2
) = [Q(y)]2
vói moi x ∈ R.
Theo Bài toán 3.3.2 ta có Q(y) = yn
, hay P(y) = (y + 1)n
vói moi n ∈ N∗.
V y P(x) = (x + 1)n
vói moi n ∈ N∗.
Liên quan đen các phương trình hàm, ta sẽ thảo lu n m t bài toán mà ràng
bu c bài toán có dạng bat phương trình như sau.
Bài toán 3.3.4 (Albanian TST 2009). Tìm tat cá các đa thŕc P(x) khác không có
h so không âm thóa mãn
P(x)P
1
≤ [P(1)]2
với x > 0.
Lời giái. Giả sử
P(x) = adxd
+ ... a1x + a0
là đa thức thỏa mãn yêu cau bài toán. Khi đó ai ≥ 0 vói moi i = 0,1,...,d.
56
Viết đề tài giá sinh viên – ZALO:0973.287.149-TEAMLUANVAN.COM
x
x
x
Vói moi x > 0, theo bat đȁng thức Bunyakovsky ta có
P(x)P
1
= adxd
+ ...a1x + a0 adx−d
+ ...a1x−1
+ a0
≥ (ad + ...a1 + a0)
= [P(1)]2
.
Ket hop giả thiet
ta có
P(x)P
1
≤ [P(1)]2
vói x > 0
P(x)P
1
= [P(1)]2
vói x > 0.
Thay dạng của đa thức P(x) như đã giả thiet, ta có
adxd
+ ...a1x + a0 adx−d
+ ...a1x−1
+ a0 = [P(1)]2
vói x > 0.
Bien đoi tương đương ta nh n đưoc
adxd
+ ...a1x + a0 ad + ...a1xd−1
+ a0xd
= [P(1)]2
xd
vói x > 0.
So sánh h so của xd+1,xd+2,...,x2d ta đưoc
a0 = a1 = ... = ad−1 = 0.
Do v y P(x) = adxd
vói ad > 0.
Thử lại ta thay đa thức P(x) = adxd
vói ad > 0 thỏa mãn các yêu cau bài toán.
Tóm lại, P(x) = adxd
vói ad > 0 là các đa thức can tìm.
3.4 Đa th c Chebyshev
Đa thức Chebyshev là m t lóp đ c bi t các đa thức, nó là soi dây liên ket đep đẽ
giữa đại so và lưong giác. Các đa thức Chebyshev không chỉ là đoi tưong nghiên
cứu của đại so, mà nó còn là công cụ quan trong trong giải tích toán hoc, lý thuyet
xap xỉ.
2
57
Viết đề tài giá sinh viên – ZALO:0973.287.149-TEAMLUANVAN.COM
n
Mục này dành đe nghiên cứu sơ lưoc ve các đa thức Chebyshev và các bài toán
liên quan. Tài li u tham khảo chính đưoc sử dụng ỏ đây là [3, 6].
3.4.1 Định nghĩa - Tính chat
Các đa thức Tn(x) vói n ∈ N đưoc xác định truy hoi
T0(x) = 1, T1(x) = x
Tn+1(x) = 2xTn(x) − Tn−1(x)
đưoc goi là các đa thŕc Chebyshev loại 1.
Các đa thức Un(x) vói n ∈ N đưoc xác định truy hoi
U0(x) = 0, U1(x) = 1
Un+1(x) = 2xUn(x) −Un−1(x) vói n ≥ 1
Tính chat 3.4.1. Các đa thŕc Chebyshev loại 1 có các tính chat sau:
(1) Với moi x ∈ [−1,1] ta có
Tn(x) = cos(narccosx).
(2) Tn(x) là đa thŕc b c n, có h so cao nhat là 2n−1
.
(3) Tn(x) là hàm so chȁn khi n chȁn và là hàm so lé khi n lé.
(4) Ta có ước lượng
|Tn(x) ≤ 1 với moi x ∈ [−1,1].
(5) Phương trình |Tn(x)| = 1 có đúng n nghi m phân bi t trong [−1,1] được
cho bới
x = cos
π
với k = 0,1,2,...,n − 1.
đưoc goi là các đa thŕc Chebyshev loại 2.
58
Viết đề tài giá sinh viên – ZALO:0973.287.149-TEAMLUANVAN.COM
Tính chat 3.4.2. Các đa thŕc Chebyshev loại 2 có các tính chat sau:
(1) Với moi x ∈ (−1,1) ta có
(2) Ta có
Un(x) =
sin(n arccos x)
√
1 − x2
.
Un(x) =
1
Tn′(x).
n
(3) Un(x) là đa thŕc b c n, có h so cao nhat là 2n−1
.
(4) Un(x) là hàm so chȁn khi n lé và là hàm so lé khi n chȁn.
(5) Ta có ước lượng
|Tn(x) ≤ n với moi x ∈ (−1,1).
3.4.2 M t so bài toán chon loc
Bài toán chon loc đau tiên mà chúng tôi sẽ trình bày là ve sụ bieu dien của mői đa
thức như là chuői các đa thức Chebyshev.
Bài toán 3.4.3. Chŕng minh rang moi đa thŕc f (x) b c n ≥ 1 đeu có the bieu dien
duy nhat dưới dạng
n
f (x) = ∑aiTi(x), an = 0. (3.9)
i=0
Lời giái. Ta có Tn(x) là đa thức b c n có h so cao nhat là 2n−1
nên ta có the viet
Tn(x) = 2n−1
xn
+ ϕ(x)
vói là ϕ(x) đa thức b c nhỏ hơn n. Suy ra
xn
=
1
Tn(x)−
1
ϕ(x).
2n−1
Bang quy nạp ta chứng minh đưoc:
2n−1
f (x) = a0 + a1T1(x)+ a2T2(x)+... + anTn(x).
59
Viết đề tài giá sinh viên – ZALO:0973.287.149-TEAMLUANVAN.COM
−
∈
2 8
+
4
+
2
+
Bây giò ta chứng minh tính duy nhat của cách bieu dien này. Giả sử
f (x) = a0 + a1T1(x) + a2T2(x) + ... + anTn(x)
= a′
0 + a1
′
T1(x)+a2
′
T2(x)+ ...+ a′
nTn(x).
Khi đó
V y
Hay
n
∑ ai ai′ Ti(x) = 0 vói moi x R.
i=0
a0 −a′
0 = a1 −a′
1 = ... = an −a′
n = 0.
a0 = a0
′
,a1 = a′
1,...,an = a′
n.
Bài toán 3.4.4. Cho đa thŕc thực f (x) = ax3
+bx2
+cx + d và so α > 0. Biet rang
| f (x)| ≤ α với moi x ∈ [−1,1]. Tìm giá trị lớn nhat của giá trị tuy t đoi các h so
của đã thŕc đã cho.
Lời giái. Đ t
A = f (−1) = −a + b − c + d
B = f −
1
= −
a
+
b
−
c
+ d
2 8 4 2
C = f
1
=
a b c
d
D = f (1) = a+b+c+d
E = f (0) = d.
Từ đây ta có
a = −
2
A+
4
B−
4
C +
2
D
3 3 3 3
b =
1
A+
1
D −E
2 2
c =
1
A −
8
B +
8
C −
1
D
6 6 6 6
d = E.
60
Viết đề tài giá sinh viên – ZALO:0973.287.149-TEAMLUANVAN.COM
2n
−
n
.
∑
j=1
1 − xj P(xj). ≤ n
·n = 2
Từ giả thiet ta có
Vói các đa thức
|a| ≤ 4α, |b| ≤ 2α, |c| ≤ 3α, |d| ≤ α.
f(x) = α 4x3
−3x và g(x) = α 2x2
− 1 ,
thay vào ta thay bat đȁng thức sẽ trỏ thành đȁng thức. V y
max|a| = 4α, max|b| = 2α, max|c| = 3α, max|d| = α.
Bài toán 3.4.5. Cho đa thŕc Pn−1(x) b c không vượt quá n−1 có h so b c cao
nhat a0, thóa mãn đieu ki n
√
1− x2 |Pn−1(x)| ≤ 1 với moi x ∈ [−1,1]. Chŕng minh
rang |a0| ≤ 2n−1
.
Lời giái. Ta viet đa thức đã cho dưói dạng n i suy Lagrange theo các nút n i suy
xj = cos 2 j−1
π là các nghi m của đa thức Chebyshev Tn(x). Ta có
P x
1 n
1 j−1
q
1 x2P x
Tn(x)
Suy ra
n−1( ) = ∑ ( )
j=1
— j n−1 (
j)
x − xj
.
a
2n−1 n
1 j−1
q
1 x2P x
V y nên
0 =
n
∑ ( )
j=1
— j ( j).
2n−1 n
.
q
2
2n−1
n−1
Ta có đieu phải chứng minh.
Bài toán 3.4.6. Giá thiet rang đa thŕc Pn−1(x) thóa mãn các đieu ki n của Bài
toán 3.4.3. Chŕng minh rang |Pn−1(x)| ≤ n với moi x ∈ [−1,1].
Lời giái. Vói các xj đưoc chon như ỏ Bài toán 3.4.5 thì do hàm so y = cos(x)
nghịch bien trong (0,π) nên
−1 < xn < xn−1 < ... < x2 < x1 < 1.
n
−
|a0| ≤ .
.
61
Viết đề tài giá sinh viên – ZALO:0973.287.149-TEAMLUANVAN.COM
−
n
n−1( )| ≤
n ∑ . j j).
.x − xj.
≤
n
∑ (x − xj)
.
√
1−x2 ≥
q
1−x2 = sin(arccosx1) = sin
π
.
1 ≥
sinx
≥
2
, sin
π
≥
π
,
2
=
1√
1−x2 ≥
1
= ∑
Neu x1 < x < 1 thì
P x
1 n
.
q
1 x2P x
. |Tn(x)| 1 n
Tn(x) (3.10)
(do x−xj > 0 và Tn(x) có dau không đoi trên (x1,1]). M t khác
n
Tn(x) = 2n−1
∏(x xj)
j=1
nên ta có
n
Tn
′
(x) = 2n−1
∑
n
j=1 (x − xj) n Tn(x) . (3.11)
Ta có
k=1 (x − xk) k=1 x−xk
nên từ (3.10) và (3.11) suy ra
|Tn
′(x)|
= |Un(x)| ≤ n
|Pn−1(x)| ≤ n vói moi x ∈ (x1;1].
Hoàn toàn tương tụ ta cũng có
|Pn−1(x)| ≤ n vói moi x ∈ [−1,xn)
Xét xn ≤ x ≤ x1. Khi đó ta có
1
2n
Do
suy ra
x π 2n 2n n n n
1
|Pn−1(x)| ≤ 1
Bài toán đưoc chứng minh xong.
= n.
Bài toán 3.4.7 (Định lý Berstein-Markov). Cho đa thŕc
Pn(x) = a0xn
+a1xn−1
+...+an
thóa mãn đieu ki n |Pn(x)| ≤ 1 với moi x ∈ [−1,1]. Chŕng minh rang khi đó
|Pn
′(x)| ≤ n2
với moi x ∈ [−1,1]
n
j=1
j=1
∏
| − n−1 (
62
Viết đề tài giá sinh viên – ZALO:0973.287.149-TEAMLUANVAN.COM
. . n.
.sin(α)Pn
′
(cos(α)). ≤ n kéo theo
√
1−x2 .Pn
′(x). ≤ 1.
Lời giái. Đ t x = cosa. Khi đó theo giả thiet thì |Pn (cosa)| ≤ 1. Do Pn(cosa) có
dạng
n
Pn (cosa) = ∑ (aj cos jα +bj sin jα).
j=0
nên ta có the áp dụng ket quả của Bài toán 3.4.6, ta đưoc
Cũng theo Bài toán 3.4.6, ta có
Pn
′(x) ≤
. n .
. n .
V y |Pn
′(x)| ≤ n2
. Ta có đieu phải chứng minh.
63
Viết đề tài giá sinh viên – ZALO:0973.287.149-TEAMLUANVAN.COM
Ket lu n
1 Nh ng ket qua đã đạt đư c
Lu n văn “Đa thŕc trong các bài thi hoc sinh giói” đã đạt đưoc các ket quả sau:
1. Trình bày đưoc các tính chat của đa thức, các bài toán ve phép chia đa thức,
ưóc - b i, nghi m và phương trình b c cao, đạo hàm và khai trien Taylor;
2. Lý thuyet và các bài toán ve các đa thức bat khả quy trên các vành (trưòng)
so;
3. M t so chủ đe nâng cao như các đa thức nhieu bien, đa thức đoi xứng,
phương trình hàm đa thức và đa thức Chebyshev.
2 Đe xuat m t so hưỚng nghiên c u tiep theo
Đa thức đại so là m t trong những chủ đe quan trong và sâu sac của toán hoc. Sau
những ket quả đã đạt đưoc trong lu n văn, chúng tôi hi vong và co gang sẽ tiep tục
nghiên cứu các chủ đe liên quan, chȁng hạn:
• Các phân thức hữu tý và các bài toán liên quan,
• Bài toán bieu dien đa thức, sụ phân bo nghi m của đa thức và ứng dụng,
• Các khía cạnh giải tích của đa thức, phương trình hàm đa thức. . .
64
Viết đề tài giá sinh viên – ZALO:0973.287.149-TEAMLUANVAN.COM
Tài li u tham khao
Tieng Vi t
[1] Nguyen Hữu Đien (2006), Đa thŕc và ŕng dnng, NXB Giáo dục.
[2] Nguyen Văn M u (1997), Phương trình hàm, NXB Giáo dục.
[3] Nguyen Văn M u (2007), Chuyên đe Đa thŕc đại so và phân thŕc hru tý,
NXB Giáo dục.
[4] Nguyen Văn M u, Nguyen Văn Ngoc (2010), Chuyên đe Đa thŕc đoi xŕng
và áp dnng, NXB Giáo dục.
[5] Lê Thị Thanh Nhàn (2015), Lý thuyet đa thŕc, NXB ĐHQG Hà N i.
[6] Lê Hoành Phò, Nguyen Văn Nho, Nguyen Tài Chung (2016), Chuyên kháo
đa thŕc (tái bản lan thứ nhat), NXB ĐHQG Hà N i.
Tieng Anh
[7] Dusˇan Djukic´ (2007), Polynomials in One Variable, Olympiad Training Ma-
terials (see www.imomath.com).
[8] 51st International Mathematical Olympiad (2010), Shortlisted Problems with
Solutions.

More Related Content

What's hot

đại số tuyến tính 2 ( không gian eculid )
đại số tuyến tính 2 ( không gian eculid )đại số tuyến tính 2 ( không gian eculid )
đại số tuyến tính 2 ( không gian eculid )Bui Loi
 
19 phương phap chứng minh bất đẳng thức
19 phương phap chứng minh bất đẳng thức19 phương phap chứng minh bất đẳng thức
19 phương phap chứng minh bất đẳng thứcThế Giới Tinh Hoa
 
Hinh hoc so cap va thuc hanh giai toan
Hinh hoc so cap va thuc hanh giai toanHinh hoc so cap va thuc hanh giai toan
Hinh hoc so cap va thuc hanh giai toanTamPhan59
 
BÀI TẬP DẠY THÊM (PHIẾU BÀI TẬP) TOÁN 7 SÁCH CÁNH DIỀU CẢ NĂM (2 TẬP) CÓ LỜI ...
BÀI TẬP DẠY THÊM (PHIẾU BÀI TẬP) TOÁN 7 SÁCH CÁNH DIỀU CẢ NĂM (2 TẬP) CÓ LỜI ...BÀI TẬP DẠY THÊM (PHIẾU BÀI TẬP) TOÁN 7 SÁCH CÁNH DIỀU CẢ NĂM (2 TẬP) CÓ LỜI ...
BÀI TẬP DẠY THÊM (PHIẾU BÀI TẬP) TOÁN 7 SÁCH CÁNH DIỀU CẢ NĂM (2 TẬP) CÓ LỜI ...Nguyen Thanh Tu Collection
 
Topo daicuong1[1]
Topo daicuong1[1]Topo daicuong1[1]
Topo daicuong1[1]Bui Loi
 
Bài toán số học liên quan tới lũy thữa
Bài toán số học liên quan tới lũy thữaBài toán số học liên quan tới lũy thữa
Bài toán số học liên quan tới lũy thữaThế Giới Tinh Hoa
 
Anh Xa Lien Tuc Tren Khong Gian Topo
Anh Xa Lien Tuc Tren Khong Gian TopoAnh Xa Lien Tuc Tren Khong Gian Topo
Anh Xa Lien Tuc Tren Khong Gian Topoipaper
 
Phương Tích - Trục Đẳng Phương
Phương Tích - Trục Đẳng PhươngPhương Tích - Trục Đẳng Phương
Phương Tích - Trục Đẳng PhươngNhập Vân Long
 
72 hệ phương trình
72 hệ phương trình72 hệ phương trình
72 hệ phương trìnhHades0510
 
Chuyên đề tìm tên kim loai, oxit và muối lớp 9
Chuyên đề tìm tên kim loai, oxit và muối lớp 9Chuyên đề tìm tên kim loai, oxit và muối lớp 9
Chuyên đề tìm tên kim loai, oxit và muối lớp 9youngunoistalented1995
 
Cực trị của hàm số, ôn thi đại học môn toán
Cực trị của hàm số, ôn thi đại học môn toánCực trị của hàm số, ôn thi đại học môn toán
Cực trị của hàm số, ôn thi đại học môn toánhai tran
 
[Vnmath.com] 13-ki-thuat-giai-phuong-trinh-ham
[Vnmath.com] 13-ki-thuat-giai-phuong-trinh-ham[Vnmath.com] 13-ki-thuat-giai-phuong-trinh-ham
[Vnmath.com] 13-ki-thuat-giai-phuong-trinh-hamDuy Duy
 
Bd hsgchuyen de_24nguyen_ly_dirichlet_voi_cac_bai_toandai_sohinh_hoc_9667
Bd hsgchuyen de_24nguyen_ly_dirichlet_voi_cac_bai_toandai_sohinh_hoc_9667Bd hsgchuyen de_24nguyen_ly_dirichlet_voi_cac_bai_toandai_sohinh_hoc_9667
Bd hsgchuyen de_24nguyen_ly_dirichlet_voi_cac_bai_toandai_sohinh_hoc_9667cunbeo
 
Sử dụng máy tính cầm tay giải nhanh trắc nghiệm lượng giác – Trần Anh Khoa
Sử dụng máy tính cầm tay giải nhanh trắc nghiệm lượng giác – Trần Anh KhoaSử dụng máy tính cầm tay giải nhanh trắc nghiệm lượng giác – Trần Anh Khoa
Sử dụng máy tính cầm tay giải nhanh trắc nghiệm lượng giác – Trần Anh KhoaTrung Tam Gia Su Tri Viet
 
Bộ sưu tập bất đẳng thức của võ quốc bá cẩn
Bộ sưu tập bất đẳng thức của võ quốc bá cẩnBộ sưu tập bất đẳng thức của võ quốc bá cẩn
Bộ sưu tập bất đẳng thức của võ quốc bá cẩnThế Giới Tinh Hoa
 
9 phương pháp giải phương trình nghiệm nguyên
9 phương pháp giải phương trình nghiệm nguyên9 phương pháp giải phương trình nghiệm nguyên
9 phương pháp giải phương trình nghiệm nguyênThấy Tên Tao Không
 

What's hot (20)

đại số tuyến tính 2 ( không gian eculid )
đại số tuyến tính 2 ( không gian eculid )đại số tuyến tính 2 ( không gian eculid )
đại số tuyến tính 2 ( không gian eculid )
 
19 phương phap chứng minh bất đẳng thức
19 phương phap chứng minh bất đẳng thức19 phương phap chứng minh bất đẳng thức
19 phương phap chứng minh bất đẳng thức
 
Đồng dư thức
Đồng dư thứcĐồng dư thức
Đồng dư thức
 
Hinh hoc so cap va thuc hanh giai toan
Hinh hoc so cap va thuc hanh giai toanHinh hoc so cap va thuc hanh giai toan
Hinh hoc so cap va thuc hanh giai toan
 
BÀI TẬP DẠY THÊM (PHIẾU BÀI TẬP) TOÁN 7 SÁCH CÁNH DIỀU CẢ NĂM (2 TẬP) CÓ LỜI ...
BÀI TẬP DẠY THÊM (PHIẾU BÀI TẬP) TOÁN 7 SÁCH CÁNH DIỀU CẢ NĂM (2 TẬP) CÓ LỜI ...BÀI TẬP DẠY THÊM (PHIẾU BÀI TẬP) TOÁN 7 SÁCH CÁNH DIỀU CẢ NĂM (2 TẬP) CÓ LỜI ...
BÀI TẬP DẠY THÊM (PHIẾU BÀI TẬP) TOÁN 7 SÁCH CÁNH DIỀU CẢ NĂM (2 TẬP) CÓ LỜI ...
 
Topo daicuong1[1]
Topo daicuong1[1]Topo daicuong1[1]
Topo daicuong1[1]
 
Bài toán số học liên quan tới lũy thữa
Bài toán số học liên quan tới lũy thữaBài toán số học liên quan tới lũy thữa
Bài toán số học liên quan tới lũy thữa
 
Dãy số và giới hạn
Dãy số và giới hạnDãy số và giới hạn
Dãy số và giới hạn
 
Luận văn: Một số lớp bài toán về phương trình hàm, HAY, 9đ
Luận văn: Một số lớp bài toán về phương trình hàm, HAY, 9đLuận văn: Một số lớp bài toán về phương trình hàm, HAY, 9đ
Luận văn: Một số lớp bài toán về phương trình hàm, HAY, 9đ
 
Anh Xa Lien Tuc Tren Khong Gian Topo
Anh Xa Lien Tuc Tren Khong Gian TopoAnh Xa Lien Tuc Tren Khong Gian Topo
Anh Xa Lien Tuc Tren Khong Gian Topo
 
Phương Tích - Trục Đẳng Phương
Phương Tích - Trục Đẳng PhươngPhương Tích - Trục Đẳng Phương
Phương Tích - Trục Đẳng Phương
 
Phương trình hàm đa thức
Phương trình hàm đa thứcPhương trình hàm đa thức
Phương trình hàm đa thức
 
72 hệ phương trình
72 hệ phương trình72 hệ phương trình
72 hệ phương trình
 
Chuyên đề tìm tên kim loai, oxit và muối lớp 9
Chuyên đề tìm tên kim loai, oxit và muối lớp 9Chuyên đề tìm tên kim loai, oxit và muối lớp 9
Chuyên đề tìm tên kim loai, oxit và muối lớp 9
 
Cực trị của hàm số, ôn thi đại học môn toán
Cực trị của hàm số, ôn thi đại học môn toánCực trị của hàm số, ôn thi đại học môn toán
Cực trị của hàm số, ôn thi đại học môn toán
 
[Vnmath.com] 13-ki-thuat-giai-phuong-trinh-ham
[Vnmath.com] 13-ki-thuat-giai-phuong-trinh-ham[Vnmath.com] 13-ki-thuat-giai-phuong-trinh-ham
[Vnmath.com] 13-ki-thuat-giai-phuong-trinh-ham
 
Bd hsgchuyen de_24nguyen_ly_dirichlet_voi_cac_bai_toandai_sohinh_hoc_9667
Bd hsgchuyen de_24nguyen_ly_dirichlet_voi_cac_bai_toandai_sohinh_hoc_9667Bd hsgchuyen de_24nguyen_ly_dirichlet_voi_cac_bai_toandai_sohinh_hoc_9667
Bd hsgchuyen de_24nguyen_ly_dirichlet_voi_cac_bai_toandai_sohinh_hoc_9667
 
Sử dụng máy tính cầm tay giải nhanh trắc nghiệm lượng giác – Trần Anh Khoa
Sử dụng máy tính cầm tay giải nhanh trắc nghiệm lượng giác – Trần Anh KhoaSử dụng máy tính cầm tay giải nhanh trắc nghiệm lượng giác – Trần Anh Khoa
Sử dụng máy tính cầm tay giải nhanh trắc nghiệm lượng giác – Trần Anh Khoa
 
Bộ sưu tập bất đẳng thức của võ quốc bá cẩn
Bộ sưu tập bất đẳng thức của võ quốc bá cẩnBộ sưu tập bất đẳng thức của võ quốc bá cẩn
Bộ sưu tập bất đẳng thức của võ quốc bá cẩn
 
9 phương pháp giải phương trình nghiệm nguyên
9 phương pháp giải phương trình nghiệm nguyên9 phương pháp giải phương trình nghiệm nguyên
9 phương pháp giải phương trình nghiệm nguyên
 

Similar to Đa Thức Trong Các Bài Toán Thi Học Sinh Giỏi.docx

Xây Dựng Hệ Thống Phân Lịch Thi Tín Chỉ Tại Trường Cao Đẳng Thương Mại Đà Nẵn...
Xây Dựng Hệ Thống Phân Lịch Thi Tín Chỉ Tại Trường Cao Đẳng Thương Mại Đà Nẵn...Xây Dựng Hệ Thống Phân Lịch Thi Tín Chỉ Tại Trường Cao Đẳng Thương Mại Đà Nẵn...
Xây Dựng Hệ Thống Phân Lịch Thi Tín Chỉ Tại Trường Cao Đẳng Thương Mại Đà Nẵn...Dịch vụ viết thuê Luận Văn - ZALO 0932091562
 
Ứng Dụng Hình Học Giải Tích Vào Giải Phương Trình, Bất Phương Trình Và Hệ Phư...
Ứng Dụng Hình Học Giải Tích Vào Giải Phương Trình, Bất Phương Trình Và Hệ Phư...Ứng Dụng Hình Học Giải Tích Vào Giải Phương Trình, Bất Phương Trình Và Hệ Phư...
Ứng Dụng Hình Học Giải Tích Vào Giải Phương Trình, Bất Phương Trình Và Hệ Phư...Dịch vụ viết đề tài trọn gói 0934.573.149
 
Hàm Đơn Đi›U, Tựa Đơn Đi›U Và Một Số Ứng Dụng Của Phép Đơn Đi›U Hóa Hàm Số.docx
Hàm Đơn Đi›U, Tựa Đơn Đi›U Và Một Số Ứng Dụng Của Phép Đơn Đi›U Hóa Hàm Số.docxHàm Đơn Đi›U, Tựa Đơn Đi›U Và Một Số Ứng Dụng Của Phép Đơn Đi›U Hóa Hàm Số.docx
Hàm Đơn Đi›U, Tựa Đơn Đi›U Và Một Số Ứng Dụng Của Phép Đơn Đi›U Hóa Hàm Số.docxDV Viết Luận văn luanvanmaster.com ZALO 0973287149
 

Similar to Đa Thức Trong Các Bài Toán Thi Học Sinh Giỏi.docx (20)

Luận văn thạc sĩ - Đa thức trong các bài toán thi học sinh giỏi.doc
Luận văn thạc sĩ - Đa thức trong các bài toán thi học sinh giỏi.docLuận văn thạc sĩ - Đa thức trong các bài toán thi học sinh giỏi.doc
Luận văn thạc sĩ - Đa thức trong các bài toán thi học sinh giỏi.doc
 
M T So L P Phương Trình Diophantine.docx
M T So L P Phương Trình Diophantine.docxM T So L P Phương Trình Diophantine.docx
M T So L P Phương Trình Diophantine.docx
 
Kĩ thuật tổng hợp Giải bất phương trình hỗn hợp.docx
Kĩ thuật tổng hợp Giải bất phương trình hỗn hợp.docxKĩ thuật tổng hợp Giải bất phương trình hỗn hợp.docx
Kĩ thuật tổng hợp Giải bất phương trình hỗn hợp.docx
 
Bất đẳng thức Trong lớp các hàm lượng giác và lượng giác ngược.docx
Bất đẳng thức Trong lớp các hàm lượng giác và lượng giác ngược.docxBất đẳng thức Trong lớp các hàm lượng giác và lượng giác ngược.docx
Bất đẳng thức Trong lớp các hàm lượng giác và lượng giác ngược.docx
 
Về phương trình hàm Loại giá trị trung bình và áp dụng.docx
Về phương trình hàm Loại giá trị trung bình và áp dụng.docxVề phương trình hàm Loại giá trị trung bình và áp dụng.docx
Về phương trình hàm Loại giá trị trung bình và áp dụng.docx
 
Ve Bat Đang Thức Ho¨ Lder Và Áp Dụng.docx
Ve  Bat  Đang  Thức  Ho¨ Lder  Và  Áp  Dụng.docxVe  Bat  Đang  Thức  Ho¨ Lder  Và  Áp  Dụng.docx
Ve Bat Đang Thức Ho¨ Lder Và Áp Dụng.docx
 
Hàm Đơn Đi U, Tựa Đơn Đi U Và M T So Ứng Dụng Của Phép Đơn Đi U Hóa Hàm So.docx
Hàm Đơn Đi U, Tựa Đơn Đi U Và M T So Ứng Dụng Của Phép Đơn Đi U Hóa Hàm So.docxHàm Đơn Đi U, Tựa Đơn Đi U Và M T So Ứng Dụng Của Phép Đơn Đi U Hóa Hàm So.docx
Hàm Đơn Đi U, Tựa Đơn Đi U Và M T So Ứng Dụng Của Phép Đơn Đi U Hóa Hàm So.docx
 
Bat Phương Trình Hàm Sinh B I Các Đại Lư Ng Trung Bình B C Tùy Ý Và Các Dạng ...
Bat Phương Trình Hàm Sinh B I Các Đại Lư Ng Trung Bình B C Tùy Ý Và Các Dạng ...Bat Phương Trình Hàm Sinh B I Các Đại Lư Ng Trung Bình B C Tùy Ý Và Các Dạng ...
Bat Phương Trình Hàm Sinh B I Các Đại Lư Ng Trung Bình B C Tùy Ý Và Các Dạng ...
 
Luận văn thạc sĩ - Một số lớp phương trình hàm trong số học.doc
Luận văn thạc sĩ - Một số lớp phương trình hàm trong số học.docLuận văn thạc sĩ - Một số lớp phương trình hàm trong số học.doc
Luận văn thạc sĩ - Một số lớp phương trình hàm trong số học.doc
 
Về hệ số nhị thức, hệ số đa thức Và một số bài toán liên quan.doc
Về hệ số nhị thức, hệ số đa thức Và một số bài toán liên quan.docVề hệ số nhị thức, hệ số đa thức Và một số bài toán liên quan.doc
Về hệ số nhị thức, hệ số đa thức Và một số bài toán liên quan.doc
 
Xây Dựng Hệ Thống Phân Lịch Thi Tín Chỉ Tại Trường Cao Đẳng Thương Mại Đà Nẵn...
Xây Dựng Hệ Thống Phân Lịch Thi Tín Chỉ Tại Trường Cao Đẳng Thương Mại Đà Nẵn...Xây Dựng Hệ Thống Phân Lịch Thi Tín Chỉ Tại Trường Cao Đẳng Thương Mại Đà Nẵn...
Xây Dựng Hệ Thống Phân Lịch Thi Tín Chỉ Tại Trường Cao Đẳng Thương Mại Đà Nẵn...
 
Bat Đang Thức V I Hàm Loi B Ph N Và Ứng Dụng.docx
Bat Đang Thức V I Hàm Loi B Ph N Và Ứng Dụng.docxBat Đang Thức V I Hàm Loi B Ph N Và Ứng Dụng.docx
Bat Đang Thức V I Hàm Loi B Ph N Và Ứng Dụng.docx
 
Về Đa Thức Bất Khả Quy Trên Trường Hữu Hạn.docx
Về Đa Thức Bất Khả Quy Trên Trường Hữu Hạn.docxVề Đa Thức Bất Khả Quy Trên Trường Hữu Hạn.docx
Về Đa Thức Bất Khả Quy Trên Trường Hữu Hạn.docx
 
Ứng Dụng Hình Học Giải Tích Vào Giải Phương Trình, Bất Phương Trình Và Hệ Phư...
Ứng Dụng Hình Học Giải Tích Vào Giải Phương Trình, Bất Phương Trình Và Hệ Phư...Ứng Dụng Hình Học Giải Tích Vào Giải Phương Trình, Bất Phương Trình Và Hệ Phư...
Ứng Dụng Hình Học Giải Tích Vào Giải Phương Trình, Bất Phương Trình Và Hệ Phư...
 
Về Phương Trình Hàm Jensen, Tính Ổn Định Và Ứng Dụng.docx
Về Phương Trình Hàm Jensen, Tính Ổn Định Và Ứng Dụng.docxVề Phương Trình Hàm Jensen, Tính Ổn Định Và Ứng Dụng.docx
Về Phương Trình Hàm Jensen, Tính Ổn Định Và Ứng Dụng.docx
 
Luận văn thạc sĩ toán học - Một số lớp phương trình diophantine.doc
Luận văn thạc sĩ toán học - Một số lớp phương trình diophantine.docLuận văn thạc sĩ toán học - Một số lớp phương trình diophantine.doc
Luận văn thạc sĩ toán học - Một số lớp phương trình diophantine.doc
 
Xấp xỉ hàm đa điều hòa dưới Bởi hàm green đa cực.doc
Xấp xỉ hàm đa điều hòa dưới Bởi hàm green đa cực.docXấp xỉ hàm đa điều hòa dưới Bởi hàm green đa cực.doc
Xấp xỉ hàm đa điều hòa dưới Bởi hàm green đa cực.doc
 
Xap xỉ diophantine và phân so liên tục trong giải Phương trình pell.docx
Xap xỉ diophantine và phân so liên tục  trong giải Phương trình pell.docxXap xỉ diophantine và phân so liên tục  trong giải Phương trình pell.docx
Xap xỉ diophantine và phân so liên tục trong giải Phương trình pell.docx
 
Hàm Đơn Đi›U, Tựa Đơn Đi›U Và Một Số Ứng Dụng Của Phép Đơn Đi›U Hóa Hàm Số.docx
Hàm Đơn Đi›U, Tựa Đơn Đi›U Và Một Số Ứng Dụng Của Phép Đơn Đi›U Hóa Hàm Số.docxHàm Đơn Đi›U, Tựa Đơn Đi›U Và Một Số Ứng Dụng Của Phép Đơn Đi›U Hóa Hàm Số.docx
Hàm Đơn Đi›U, Tựa Đơn Đi›U Và Một Số Ứng Dụng Của Phép Đơn Đi›U Hóa Hàm Số.docx
 
Bài Toán Phân Hoạch So Nguyên Dương.docx
Bài Toán Phân Hoạch    So Nguyên Dương.docxBài Toán Phân Hoạch    So Nguyên Dương.docx
Bài Toán Phân Hoạch So Nguyên Dương.docx
 

More from DV Viết Luận văn luanvanmaster.com ZALO 0973287149

More from DV Viết Luận văn luanvanmaster.com ZALO 0973287149 (20)

Ảnh Hưởng Của Marketing Quan Hệ Đến Lòng Trung Thành Của Khách Hàng.Tình Huốn...
Ảnh Hưởng Của Marketing Quan Hệ Đến Lòng Trung Thành Của Khách Hàng.Tình Huốn...Ảnh Hưởng Của Marketing Quan Hệ Đến Lòng Trung Thành Của Khách Hàng.Tình Huốn...
Ảnh Hưởng Của Marketing Quan Hệ Đến Lòng Trung Thành Của Khách Hàng.Tình Huốn...
 
Phát triển nguồn nhân lực tại Uỷ ban nhân dân huyện Trà Bồng, tỉnh Quảng Ngãi...
Phát triển nguồn nhân lực tại Uỷ ban nhân dân huyện Trà Bồng, tỉnh Quảng Ngãi...Phát triển nguồn nhân lực tại Uỷ ban nhân dân huyện Trà Bồng, tỉnh Quảng Ngãi...
Phát triển nguồn nhân lực tại Uỷ ban nhân dân huyện Trà Bồng, tỉnh Quảng Ngãi...
 
Báo cáo tốt Nghiệp tài chính hợp nhất tại tổng công ty Indochina gol...
Báo cáo tốt Nghiệp  tài chính hợp nhất tại tổng công ty Indochina gol...Báo cáo tốt Nghiệp  tài chính hợp nhất tại tổng công ty Indochina gol...
Báo cáo tốt Nghiệp tài chính hợp nhất tại tổng công ty Indochina gol...
 
Tạo động lực thúc đẩy nhân viên làm việc tại ngân hàng TMCP Ngoại Thương Việt...
Tạo động lực thúc đẩy nhân viên làm việc tại ngân hàng TMCP Ngoại Thương Việt...Tạo động lực thúc đẩy nhân viên làm việc tại ngân hàng TMCP Ngoại Thương Việt...
Tạo động lực thúc đẩy nhân viên làm việc tại ngân hàng TMCP Ngoại Thương Việt...
 
Phát triển công nghiệp trên địa bàn Thành phố Tam Kỳ, Tỉnh Quảng Na...
Phát triển công nghiệp trên địa bàn Thành phố Tam Kỳ, Tỉnh Quảng Na...Phát triển công nghiệp trên địa bàn Thành phố Tam Kỳ, Tỉnh Quảng Na...
Phát triển công nghiệp trên địa bàn Thành phố Tam Kỳ, Tỉnh Quảng Na...
 
Giải pháp phát triển cho vay xuất nhập khẩu tại ngân hàng NN&PTNN ch...
Giải pháp phát triển cho vay xuất nhập khẩu tại ngân hàng NN&PTNN ch...Giải pháp phát triển cho vay xuất nhập khẩu tại ngân hàng NN&PTNN ch...
Giải pháp phát triển cho vay xuất nhập khẩu tại ngân hàng NN&PTNN ch...
 
Hoàn thiện công tác lập báo cáo tài chính hợp nhất tại tổng công ...
Hoàn thiện công tác lập báo cáo tài chính hợp nhất tại tổng công ...Hoàn thiện công tác lập báo cáo tài chính hợp nhất tại tổng công ...
Hoàn thiện công tác lập báo cáo tài chính hợp nhất tại tổng công ...
 
Luận Văn Thạc Sĩ Quản trị thành tích nhân viên tại Cục Hải quan TP Đà Nẵng.doc
Luận Văn Thạc Sĩ  Quản trị thành tích nhân viên tại Cục Hải quan TP Đà Nẵng.docLuận Văn Thạc Sĩ  Quản trị thành tích nhân viên tại Cục Hải quan TP Đà Nẵng.doc
Luận Văn Thạc Sĩ Quản trị thành tích nhân viên tại Cục Hải quan TP Đà Nẵng.doc
 
Hoàn thiện công tác quản lý thuế thu nhập cá nhân tại cục thuế Tỉ...
Hoàn thiện công tác quản lý thuế thu nhập cá nhân tại cục thuế Tỉ...Hoàn thiện công tác quản lý thuế thu nhập cá nhân tại cục thuế Tỉ...
Hoàn thiện công tác quản lý thuế thu nhập cá nhân tại cục thuế Tỉ...
 
Đề Tài Phát triển bền vững nông nghiệp Huyện Ba Tơ, Tỉnh Quảng Ngãi....
Đề Tài Phát triển bền vững nông nghiệp Huyện Ba Tơ, Tỉnh Quảng Ngãi....Đề Tài Phát triển bền vững nông nghiệp Huyện Ba Tơ, Tỉnh Quảng Ngãi....
Đề Tài Phát triển bền vững nông nghiệp Huyện Ba Tơ, Tỉnh Quảng Ngãi....
 
Hoàn thiện công tác bảo trợ xã hội trên địa bàn huyện Phong Điền, tỉnh Thừa T...
Hoàn thiện công tác bảo trợ xã hội trên địa bàn huyện Phong Điền, tỉnh Thừa T...Hoàn thiện công tác bảo trợ xã hội trên địa bàn huyện Phong Điền, tỉnh Thừa T...
Hoàn thiện công tác bảo trợ xã hội trên địa bàn huyện Phong Điền, tỉnh Thừa T...
 
Đề Tài Luận VănPhát triển sản phẩm du lịch tại thành phố Đà Nẵng.doc
Đề Tài Luận VănPhát triển sản phẩm du lịch tại thành phố Đà Nẵng.docĐề Tài Luận VănPhát triển sản phẩm du lịch tại thành phố Đà Nẵng.doc
Đề Tài Luận VănPhát triển sản phẩm du lịch tại thành phố Đà Nẵng.doc
 
Đào tạo nghề cho lao động thuộc diện thu hồi đất trên địa bàn Thàn...
Đào tạo nghề cho lao động thuộc diện thu hồi đất trên địa bàn Thàn...Đào tạo nghề cho lao động thuộc diện thu hồi đất trên địa bàn Thàn...
Đào tạo nghề cho lao động thuộc diện thu hồi đất trên địa bàn Thàn...
 
Tóm Tắt Luận Văn Thạc Sĩ Quản Trị Kinh Doanh Xây dựng chính sách Marketing tạ...
Tóm Tắt Luận Văn Thạc Sĩ Quản Trị Kinh Doanh Xây dựng chính sách Marketing tạ...Tóm Tắt Luận Văn Thạc Sĩ Quản Trị Kinh Doanh Xây dựng chính sách Marketing tạ...
Tóm Tắt Luận Văn Thạc Sĩ Quản Trị Kinh Doanh Xây dựng chính sách Marketing tạ...
 
Đề Tài Nghiên cứu rủi ro cảm nhận đối với mua hàng thời trang trực tuyến.docx
Đề Tài Nghiên cứu rủi ro cảm nhận đối với mua hàng thời trang trực tuyến.docxĐề Tài Nghiên cứu rủi ro cảm nhận đối với mua hàng thời trang trực tuyến.docx
Đề Tài Nghiên cứu rủi ro cảm nhận đối với mua hàng thời trang trực tuyến.docx
 
Giải pháp nâng cao động lực thúc đẩy người lao động tại công ty khai...
Giải pháp nâng cao động lực thúc đẩy người lao động tại công ty khai...Giải pháp nâng cao động lực thúc đẩy người lao động tại công ty khai...
Giải pháp nâng cao động lực thúc đẩy người lao động tại công ty khai...
 
Giải pháp phát triển dịch vụ ngân hàng điện tử tại ngân hàng đầu ...
Giải pháp phát triển dịch vụ ngân hàng điện tử tại ngân hàng đầu ...Giải pháp phát triển dịch vụ ngân hàng điện tử tại ngân hàng đầu ...
Giải pháp phát triển dịch vụ ngân hàng điện tử tại ngân hàng đầu ...
 
Giải pháp phát triển dịch vụ ngân hàng điện tử tại ngân hàng đầu ...
Giải pháp phát triển dịch vụ ngân hàng điện tử tại ngân hàng đầu ...Giải pháp phát triển dịch vụ ngân hàng điện tử tại ngân hàng đầu ...
Giải pháp phát triển dịch vụ ngân hàng điện tử tại ngân hàng đầu ...
 
Quản trị quan hệ khách hàng tại Chi nhánh Viettel Đà Nẵng – Tập đoàn Viễn thô...
Quản trị quan hệ khách hàng tại Chi nhánh Viettel Đà Nẵng – Tập đoàn Viễn thô...Quản trị quan hệ khách hàng tại Chi nhánh Viettel Đà Nẵng – Tập đoàn Viễn thô...
Quản trị quan hệ khách hàng tại Chi nhánh Viettel Đà Nẵng – Tập đoàn Viễn thô...
 
Đề Tài Đánh giá thành tích đội ngũ giảng viên trường Đại Học Phạm ...
Đề Tài Đánh giá thành tích đội ngũ giảng viên trường Đại Học Phạm ...Đề Tài Đánh giá thành tích đội ngũ giảng viên trường Đại Học Phạm ...
Đề Tài Đánh giá thành tích đội ngũ giảng viên trường Đại Học Phạm ...
 

Recently uploaded

SÁNG KIẾN ÁP DỤNG CLT (COMMUNICATIVE LANGUAGE TEACHING) VÀO QUÁ TRÌNH DẠY - H...
SÁNG KIẾN ÁP DỤNG CLT (COMMUNICATIVE LANGUAGE TEACHING) VÀO QUÁ TRÌNH DẠY - H...SÁNG KIẾN ÁP DỤNG CLT (COMMUNICATIVE LANGUAGE TEACHING) VÀO QUÁ TRÌNH DẠY - H...
SÁNG KIẾN ÁP DỤNG CLT (COMMUNICATIVE LANGUAGE TEACHING) VÀO QUÁ TRÌNH DẠY - H...Nguyen Thanh Tu Collection
 
SLIDE - Tu van, huong dan cong tac tuyen sinh-2024 (đầy đủ chi tiết).pdf
SLIDE - Tu van, huong dan cong tac tuyen sinh-2024 (đầy đủ chi tiết).pdfSLIDE - Tu van, huong dan cong tac tuyen sinh-2024 (đầy đủ chi tiết).pdf
SLIDE - Tu van, huong dan cong tac tuyen sinh-2024 (đầy đủ chi tiết).pdfhoangtuansinh1
 
30 ĐỀ PHÁT TRIỂN THEO CẤU TRÚC ĐỀ MINH HỌA BGD NGÀY 22-3-2024 KỲ THI TỐT NGHI...
30 ĐỀ PHÁT TRIỂN THEO CẤU TRÚC ĐỀ MINH HỌA BGD NGÀY 22-3-2024 KỲ THI TỐT NGHI...30 ĐỀ PHÁT TRIỂN THEO CẤU TRÚC ĐỀ MINH HỌA BGD NGÀY 22-3-2024 KỲ THI TỐT NGHI...
30 ĐỀ PHÁT TRIỂN THEO CẤU TRÚC ĐỀ MINH HỌA BGD NGÀY 22-3-2024 KỲ THI TỐT NGHI...Nguyen Thanh Tu Collection
 
sách sinh học đại cương - Textbook.pdf
sách sinh học đại cương   -   Textbook.pdfsách sinh học đại cương   -   Textbook.pdf
sách sinh học đại cương - Textbook.pdfTrnHoa46
 
GNHH và KBHQ - giao nhận hàng hoá và khai báo hải quan
GNHH và KBHQ - giao nhận hàng hoá và khai báo hải quanGNHH và KBHQ - giao nhận hàng hoá và khai báo hải quan
GNHH và KBHQ - giao nhận hàng hoá và khai báo hải quanmyvh40253
 
Kiểm tra cuối học kì 1 sinh học 12 đề tham khảo
Kiểm tra cuối học kì 1 sinh học 12 đề tham khảoKiểm tra cuối học kì 1 sinh học 12 đề tham khảo
Kiểm tra cuối học kì 1 sinh học 12 đề tham khảohoanhv296
 
Campbell _2011_ - Sinh học - Tế bào - Ref.pdf
Campbell _2011_ - Sinh học - Tế bào - Ref.pdfCampbell _2011_ - Sinh học - Tế bào - Ref.pdf
Campbell _2011_ - Sinh học - Tế bào - Ref.pdfTrnHoa46
 
Giới thiệu Dự án Sản Phụ Khoa - Y Học Cộng Đồng
Giới thiệu Dự án Sản Phụ Khoa - Y Học Cộng ĐồngGiới thiệu Dự án Sản Phụ Khoa - Y Học Cộng Đồng
Giới thiệu Dự án Sản Phụ Khoa - Y Học Cộng ĐồngYhoccongdong.com
 
GIÁO ÁN DẠY THÊM (KẾ HOẠCH BÀI DẠY BUỔI 2) - TIẾNG ANH 7 GLOBAL SUCCESS (2 CỘ...
GIÁO ÁN DẠY THÊM (KẾ HOẠCH BÀI DẠY BUỔI 2) - TIẾNG ANH 7 GLOBAL SUCCESS (2 CỘ...GIÁO ÁN DẠY THÊM (KẾ HOẠCH BÀI DẠY BUỔI 2) - TIẾNG ANH 7 GLOBAL SUCCESS (2 CỘ...
GIÁO ÁN DẠY THÊM (KẾ HOẠCH BÀI DẠY BUỔI 2) - TIẾNG ANH 7 GLOBAL SUCCESS (2 CỘ...Nguyen Thanh Tu Collection
 
Danh sách sinh viên tốt nghiệp Đại học - Cao đẳng Trường Đại học Phú Yên năm ...
Danh sách sinh viên tốt nghiệp Đại học - Cao đẳng Trường Đại học Phú Yên năm ...Danh sách sinh viên tốt nghiệp Đại học - Cao đẳng Trường Đại học Phú Yên năm ...
Danh sách sinh viên tốt nghiệp Đại học - Cao đẳng Trường Đại học Phú Yên năm ...hoangtuansinh1
 
30 ĐỀ PHÁT TRIỂN THEO CẤU TRÚC ĐỀ MINH HỌA BGD NGÀY 22-3-2024 KỲ THI TỐT NGHI...
30 ĐỀ PHÁT TRIỂN THEO CẤU TRÚC ĐỀ MINH HỌA BGD NGÀY 22-3-2024 KỲ THI TỐT NGHI...30 ĐỀ PHÁT TRIỂN THEO CẤU TRÚC ĐỀ MINH HỌA BGD NGÀY 22-3-2024 KỲ THI TỐT NGHI...
30 ĐỀ PHÁT TRIỂN THEO CẤU TRÚC ĐỀ MINH HỌA BGD NGÀY 22-3-2024 KỲ THI TỐT NGHI...Nguyen Thanh Tu Collection
 
3-BẢNG MÃ LỖI CỦA CÁC HÃNG ĐIỀU HÒA .pdf - ĐIỆN LẠNH BÁCH KHOA HÀ NỘI
3-BẢNG MÃ LỖI CỦA CÁC HÃNG ĐIỀU HÒA .pdf - ĐIỆN LẠNH BÁCH KHOA HÀ NỘI3-BẢNG MÃ LỖI CỦA CÁC HÃNG ĐIỀU HÒA .pdf - ĐIỆN LẠNH BÁCH KHOA HÀ NỘI
3-BẢNG MÃ LỖI CỦA CÁC HÃNG ĐIỀU HÒA .pdf - ĐIỆN LẠNH BÁCH KHOA HÀ NỘIĐiện Lạnh Bách Khoa Hà Nội
 
powerpoint mẫu họp phụ huynh cuối kì 2 học sinh lớp 7 bgs
powerpoint mẫu họp phụ huynh cuối kì 2 học sinh lớp 7 bgspowerpoint mẫu họp phụ huynh cuối kì 2 học sinh lớp 7 bgs
powerpoint mẫu họp phụ huynh cuối kì 2 học sinh lớp 7 bgsNmmeomeo
 
BỘ LUYỆN NGHE VÀO 10 TIẾNG ANH DẠNG TRẮC NGHIỆM 4 CÂU TRẢ LỜI - CÓ FILE NGHE.pdf
BỘ LUYỆN NGHE VÀO 10 TIẾNG ANH DẠNG TRẮC NGHIỆM 4 CÂU TRẢ LỜI - CÓ FILE NGHE.pdfBỘ LUYỆN NGHE VÀO 10 TIẾNG ANH DẠNG TRẮC NGHIỆM 4 CÂU TRẢ LỜI - CÓ FILE NGHE.pdf
BỘ LUYỆN NGHE VÀO 10 TIẾNG ANH DẠNG TRẮC NGHIỆM 4 CÂU TRẢ LỜI - CÓ FILE NGHE.pdfNguyen Thanh Tu Collection
 
kinh tế chính trị mác lênin chương hai và hàng hoá và sxxhh
kinh tế chính trị mác lênin chương hai và hàng hoá và sxxhhkinh tế chính trị mác lênin chương hai và hàng hoá và sxxhh
kinh tế chính trị mác lênin chương hai và hàng hoá và sxxhhdtlnnm
 
30 ĐỀ PHÁT TRIỂN THEO CẤU TRÚC ĐỀ MINH HỌA BGD NGÀY 22-3-2024 KỲ THI TỐT NGHI...
30 ĐỀ PHÁT TRIỂN THEO CẤU TRÚC ĐỀ MINH HỌA BGD NGÀY 22-3-2024 KỲ THI TỐT NGHI...30 ĐỀ PHÁT TRIỂN THEO CẤU TRÚC ĐỀ MINH HỌA BGD NGÀY 22-3-2024 KỲ THI TỐT NGHI...
30 ĐỀ PHÁT TRIỂN THEO CẤU TRÚC ĐỀ MINH HỌA BGD NGÀY 22-3-2024 KỲ THI TỐT NGHI...Nguyen Thanh Tu Collection
 
Các điều kiện bảo hiểm trong bảo hiểm hàng hoá
Các điều kiện bảo hiểm trong bảo hiểm hàng hoáCác điều kiện bảo hiểm trong bảo hiểm hàng hoá
Các điều kiện bảo hiểm trong bảo hiểm hàng hoámyvh40253
 
Đề cương môn giải phẫu......................
Đề cương môn giải phẫu......................Đề cương môn giải phẫu......................
Đề cương môn giải phẫu......................TrnHoa46
 
PHƯƠNG THỨC VẬN TẢI ĐƯỜNG SẮT TRONG VẬN TẢI
PHƯƠNG THỨC VẬN TẢI ĐƯỜNG SẮT TRONG VẬN TẢIPHƯƠNG THỨC VẬN TẢI ĐƯỜNG SẮT TRONG VẬN TẢI
PHƯƠNG THỨC VẬN TẢI ĐƯỜNG SẮT TRONG VẬN TẢImyvh40253
 

Recently uploaded (20)

SÁNG KIẾN ÁP DỤNG CLT (COMMUNICATIVE LANGUAGE TEACHING) VÀO QUÁ TRÌNH DẠY - H...
SÁNG KIẾN ÁP DỤNG CLT (COMMUNICATIVE LANGUAGE TEACHING) VÀO QUÁ TRÌNH DẠY - H...SÁNG KIẾN ÁP DỤNG CLT (COMMUNICATIVE LANGUAGE TEACHING) VÀO QUÁ TRÌNH DẠY - H...
SÁNG KIẾN ÁP DỤNG CLT (COMMUNICATIVE LANGUAGE TEACHING) VÀO QUÁ TRÌNH DẠY - H...
 
SLIDE - Tu van, huong dan cong tac tuyen sinh-2024 (đầy đủ chi tiết).pdf
SLIDE - Tu van, huong dan cong tac tuyen sinh-2024 (đầy đủ chi tiết).pdfSLIDE - Tu van, huong dan cong tac tuyen sinh-2024 (đầy đủ chi tiết).pdf
SLIDE - Tu van, huong dan cong tac tuyen sinh-2024 (đầy đủ chi tiết).pdf
 
30 ĐỀ PHÁT TRIỂN THEO CẤU TRÚC ĐỀ MINH HỌA BGD NGÀY 22-3-2024 KỲ THI TỐT NGHI...
30 ĐỀ PHÁT TRIỂN THEO CẤU TRÚC ĐỀ MINH HỌA BGD NGÀY 22-3-2024 KỲ THI TỐT NGHI...30 ĐỀ PHÁT TRIỂN THEO CẤU TRÚC ĐỀ MINH HỌA BGD NGÀY 22-3-2024 KỲ THI TỐT NGHI...
30 ĐỀ PHÁT TRIỂN THEO CẤU TRÚC ĐỀ MINH HỌA BGD NGÀY 22-3-2024 KỲ THI TỐT NGHI...
 
sách sinh học đại cương - Textbook.pdf
sách sinh học đại cương   -   Textbook.pdfsách sinh học đại cương   -   Textbook.pdf
sách sinh học đại cương - Textbook.pdf
 
GNHH và KBHQ - giao nhận hàng hoá và khai báo hải quan
GNHH và KBHQ - giao nhận hàng hoá và khai báo hải quanGNHH và KBHQ - giao nhận hàng hoá và khai báo hải quan
GNHH và KBHQ - giao nhận hàng hoá và khai báo hải quan
 
Kiểm tra cuối học kì 1 sinh học 12 đề tham khảo
Kiểm tra cuối học kì 1 sinh học 12 đề tham khảoKiểm tra cuối học kì 1 sinh học 12 đề tham khảo
Kiểm tra cuối học kì 1 sinh học 12 đề tham khảo
 
1 - MÃ LỖI SỬA CHỮA BOARD MẠCH BẾP TỪ.pdf
1 - MÃ LỖI SỬA CHỮA BOARD MẠCH BẾP TỪ.pdf1 - MÃ LỖI SỬA CHỮA BOARD MẠCH BẾP TỪ.pdf
1 - MÃ LỖI SỬA CHỮA BOARD MẠCH BẾP TỪ.pdf
 
Campbell _2011_ - Sinh học - Tế bào - Ref.pdf
Campbell _2011_ - Sinh học - Tế bào - Ref.pdfCampbell _2011_ - Sinh học - Tế bào - Ref.pdf
Campbell _2011_ - Sinh học - Tế bào - Ref.pdf
 
Giới thiệu Dự án Sản Phụ Khoa - Y Học Cộng Đồng
Giới thiệu Dự án Sản Phụ Khoa - Y Học Cộng ĐồngGiới thiệu Dự án Sản Phụ Khoa - Y Học Cộng Đồng
Giới thiệu Dự án Sản Phụ Khoa - Y Học Cộng Đồng
 
GIÁO ÁN DẠY THÊM (KẾ HOẠCH BÀI DẠY BUỔI 2) - TIẾNG ANH 7 GLOBAL SUCCESS (2 CỘ...
GIÁO ÁN DẠY THÊM (KẾ HOẠCH BÀI DẠY BUỔI 2) - TIẾNG ANH 7 GLOBAL SUCCESS (2 CỘ...GIÁO ÁN DẠY THÊM (KẾ HOẠCH BÀI DẠY BUỔI 2) - TIẾNG ANH 7 GLOBAL SUCCESS (2 CỘ...
GIÁO ÁN DẠY THÊM (KẾ HOẠCH BÀI DẠY BUỔI 2) - TIẾNG ANH 7 GLOBAL SUCCESS (2 CỘ...
 
Danh sách sinh viên tốt nghiệp Đại học - Cao đẳng Trường Đại học Phú Yên năm ...
Danh sách sinh viên tốt nghiệp Đại học - Cao đẳng Trường Đại học Phú Yên năm ...Danh sách sinh viên tốt nghiệp Đại học - Cao đẳng Trường Đại học Phú Yên năm ...
Danh sách sinh viên tốt nghiệp Đại học - Cao đẳng Trường Đại học Phú Yên năm ...
 
30 ĐỀ PHÁT TRIỂN THEO CẤU TRÚC ĐỀ MINH HỌA BGD NGÀY 22-3-2024 KỲ THI TỐT NGHI...
30 ĐỀ PHÁT TRIỂN THEO CẤU TRÚC ĐỀ MINH HỌA BGD NGÀY 22-3-2024 KỲ THI TỐT NGHI...30 ĐỀ PHÁT TRIỂN THEO CẤU TRÚC ĐỀ MINH HỌA BGD NGÀY 22-3-2024 KỲ THI TỐT NGHI...
30 ĐỀ PHÁT TRIỂN THEO CẤU TRÚC ĐỀ MINH HỌA BGD NGÀY 22-3-2024 KỲ THI TỐT NGHI...
 
3-BẢNG MÃ LỖI CỦA CÁC HÃNG ĐIỀU HÒA .pdf - ĐIỆN LẠNH BÁCH KHOA HÀ NỘI
3-BẢNG MÃ LỖI CỦA CÁC HÃNG ĐIỀU HÒA .pdf - ĐIỆN LẠNH BÁCH KHOA HÀ NỘI3-BẢNG MÃ LỖI CỦA CÁC HÃNG ĐIỀU HÒA .pdf - ĐIỆN LẠNH BÁCH KHOA HÀ NỘI
3-BẢNG MÃ LỖI CỦA CÁC HÃNG ĐIỀU HÒA .pdf - ĐIỆN LẠNH BÁCH KHOA HÀ NỘI
 
powerpoint mẫu họp phụ huynh cuối kì 2 học sinh lớp 7 bgs
powerpoint mẫu họp phụ huynh cuối kì 2 học sinh lớp 7 bgspowerpoint mẫu họp phụ huynh cuối kì 2 học sinh lớp 7 bgs
powerpoint mẫu họp phụ huynh cuối kì 2 học sinh lớp 7 bgs
 
BỘ LUYỆN NGHE VÀO 10 TIẾNG ANH DẠNG TRẮC NGHIỆM 4 CÂU TRẢ LỜI - CÓ FILE NGHE.pdf
BỘ LUYỆN NGHE VÀO 10 TIẾNG ANH DẠNG TRẮC NGHIỆM 4 CÂU TRẢ LỜI - CÓ FILE NGHE.pdfBỘ LUYỆN NGHE VÀO 10 TIẾNG ANH DẠNG TRẮC NGHIỆM 4 CÂU TRẢ LỜI - CÓ FILE NGHE.pdf
BỘ LUYỆN NGHE VÀO 10 TIẾNG ANH DẠNG TRẮC NGHIỆM 4 CÂU TRẢ LỜI - CÓ FILE NGHE.pdf
 
kinh tế chính trị mác lênin chương hai và hàng hoá và sxxhh
kinh tế chính trị mác lênin chương hai và hàng hoá và sxxhhkinh tế chính trị mác lênin chương hai và hàng hoá và sxxhh
kinh tế chính trị mác lênin chương hai và hàng hoá và sxxhh
 
30 ĐỀ PHÁT TRIỂN THEO CẤU TRÚC ĐỀ MINH HỌA BGD NGÀY 22-3-2024 KỲ THI TỐT NGHI...
30 ĐỀ PHÁT TRIỂN THEO CẤU TRÚC ĐỀ MINH HỌA BGD NGÀY 22-3-2024 KỲ THI TỐT NGHI...30 ĐỀ PHÁT TRIỂN THEO CẤU TRÚC ĐỀ MINH HỌA BGD NGÀY 22-3-2024 KỲ THI TỐT NGHI...
30 ĐỀ PHÁT TRIỂN THEO CẤU TRÚC ĐỀ MINH HỌA BGD NGÀY 22-3-2024 KỲ THI TỐT NGHI...
 
Các điều kiện bảo hiểm trong bảo hiểm hàng hoá
Các điều kiện bảo hiểm trong bảo hiểm hàng hoáCác điều kiện bảo hiểm trong bảo hiểm hàng hoá
Các điều kiện bảo hiểm trong bảo hiểm hàng hoá
 
Đề cương môn giải phẫu......................
Đề cương môn giải phẫu......................Đề cương môn giải phẫu......................
Đề cương môn giải phẫu......................
 
PHƯƠNG THỨC VẬN TẢI ĐƯỜNG SẮT TRONG VẬN TẢI
PHƯƠNG THỨC VẬN TẢI ĐƯỜNG SẮT TRONG VẬN TẢIPHƯƠNG THỨC VẬN TẢI ĐƯỜNG SẮT TRONG VẬN TẢI
PHƯƠNG THỨC VẬN TẢI ĐƯỜNG SẮT TRONG VẬN TẢI
 

Đa Thức Trong Các Bài Toán Thi Học Sinh Giỏi.docx

  • 1. ĐẠI HỌC THÁI NGUYÊN TRƯỜNG ĐẠI HỌC KHOA HỌC --------------------------- Tải tài liệu tại sividoc.com Viết đề tài giá sinh viên – ZALO:0973.287.149-TEAMLUANVAN.COM NGUYỄN THANH TÙNG ĐA THỨC TRONG CÁC BÀI TOÁN THI HỌC SINH GIỎI LUẬN VĂN THẠC SĨ TOÁN HỌC THÁI NGUYÊN - 2017
  • 2. ĐẠI HỌC THÁI NGUYÊN TRƯỜNG ĐẠI HỌC KHOA HỌC --------------------------- Tải tài liệu tại sividoc.com Viết đề tài giá sinh viên – ZALO:0973.287.149-TEAMLUANVAN.COM Viết đề tài giá sinh viên – ZALO:0973.287.149-TEAMLUANVAN.COM
  • 3. ĐẠI HỌC THÁI NGUYÊN TRƯỜNG ĐẠI HỌC KHOA HỌC --------------------------- Tải tài liệu tại sividoc.com Viết đề tài giá sinh viên – ZALO:0973.287.149-TEAMLUANVAN.COM NGUYỄN THANH TÙNG ĐA THỨC TRONG CÁC BÀI TOÁN THI HỌC SINH GIỎI LUẬN VĂN THẠC SĨ TOÁN HỌC Chuyên ngành: Phương pháp Toán sơ cấp Mã số: 60 46 01 13 NGƯỜI HƯỚNG DẪN KHOA HỌC GS.TSKH. Đặng Hùng Thắng THÁI NGUYÊN - 2017
  • 4. 1 Viết đề tài giá sinh viên – ZALO:0973.287.149-TEAMLUANVAN.COM Mục lục Danh sách kí hi u 3 MƠ đau 4 Chương 1. Đa th c m t bien 7 1.1 Định nghĩa và các tính chat . . . . . . . . . . . . . . . . . . . . . 7 1.1.1 Định nghĩa . . . . . . . . . . . . . . . . . . . . . . . . . 7 1.1.2 Các phép tính trên đa thức . . . . . . . . . . . . . . . . . 8 1.1.3 Các tính chat cơ bản . . . . . . . . . . . . . . . . . . . . 9 1.2 Phép chia đa thức. Ưóc chung lón nhat và nhỏ nhat ............................ 11 1.2.1 Phép chia đa thức..................................................................... 11 1.2.2 Thu t toán Euclide ................................................................... 11 1.3 Nghi m của đa thức. Phương trình b c cao........................................... 16 1.3.1 Nghi m của đa thức ................................................................. 16 1.3.2 Phương trình b c cao ............................................................... 22 1.4 Đạo hàm của đa thức. Định lý Taylor .................................................. 32 Chương 2. Đa th c bat kha quy 36 2.1 Đa thức bat khả quy ............................................................................. 36 2.1.1 Đa thức vói h so thục và phức................................................ 37 2.1.2 Đa thức bat khả quy của vành Q[x] ......................................... 40
  • 5. 2 Viết đề tài giá sinh viên – ZALO:0973.287.149-TEAMLUANVAN.COM 2.2 M t so bài toán đien hình ..................................................................42 Chương 3. M t so chu đe khác 46 3.1 Đa thức nhieu bien ............................................................................... 46 3.2 Đa thức đoi xứng.................................................................................. 49 3.3 Phương trình hàm đa thức.................................................................... 53 3.4 Đa thức Chebyshev .............................................................................. 56 3.4.1 Định nghĩa - Tính chat............................................................. 57 3.4.2 M t so bài toán chon loc .......................................................58 Ket lu n 63 Tài li u tham khao 64
  • 6. 3 Viết đề tài giá sinh viên – ZALO:0973.287.149-TEAMLUANVAN.COM Danh sách kí hi u Z vành so nguyên Q trưòng so hữu tý R trưòng so thục C trưòng so phức R vành F trưòng R[x] vành đa thức vói h so trên vành R degP(x) b c của đa thức P(x) P(x) . Q(x), Q(x) | P(x) đa thức Q(x) là ưóc của đa thức P(x) gcd(P(X),Q(X)) ưóc chung lón nhat của P(X) và Q(X) a ≡ b (mod p) a đong dư vói b theo modulo p m i=1 m i=1 ai ký hi u tong a1 +a2 +···+am bi ký hi u tích b1b2 ···bm ∑ ∏
  • 7. 4 Viết đề tài giá sinh viên – ZALO:0973.287.149-TEAMLUANVAN.COM MƠ đau Đa thức là m t đoi tưong quan trong của Toán hoc cả ve m t lý thuyet cũng như ứng dụng. Đoi vói Toán hoc pho thông, hoc sinh làm quen vói các phép toán trên đa thức (c ng trừ nhân chia), giải các phương trình b c nhat, b c hai và m t so dạng phương trình b c cao. Trong các kỳ thi hoc sinh giỏi quoc gia và quoc te, chủ đe đa thức cũng đưoc khai thác sâu hơn vói các bài toán hay và tương đoi khó ve phương trình đại so b c cao, phương trình hàm đa thức, đathức bat khả quy, tính chia het của đa thức ... Các bài toán nâng cao ve đa thức xuat hi n cũng khá nhieu trong các tạp chí toán hoc cho hoc sinh khá giỏi (như Tạp chí Toán hoc và Tuoi trẻ, Kvant, Crux,. . . ). Tuy nhiên hi n nay có ít các tài li u ve tieng Vi t trình bày m t cách h thong cả lý thuyet và bài t p ve đa thức, vói định hưóng boi dưõng hoc sinh giỏi Toán và boi dưõng giáo viên dạy chuyên Toán. Mục tiêu của lu n văn là tìm hieu m t cách đay đủ những ket quả quan trong của đa thức có nhieu ứng dụng trong Toán pho thông. Trên cơ sỏ đó, phân loại và h thong hoá (theo dạng cũng như phương pháp giải) các bài t p nâng cao ve đa thức đã có cũng như sáng tác, bo sung thêm những bài toán mói. Chúng tôi rat co gang đe lu n văn này trỏ thành m t tài li u tham khảo tot, thiet thục phục vụ cho vi c giảng dạy hoc sinh giỏi và boi dưõng giáo viên. Thông qua vi c viet lu n văn hoc viên sẽ mỏ r ng nâng cao hieu biet ve đa thức, hình thành các kỹ năng giải các bài toán khó ve đa thức, kỹ năng tìm kiem thu th p chon loc các thông tin. N i dung của lu n văn đưoc trình bày trong ba chương như sau:
  • 8. 5 Viết đề tài giá sinh viên – ZALO:0973.287.149-TEAMLUANVAN.COM • Chương 1. Đa thŕc m t bien. Trong chương này chúng tôi sẽ trình bày ngan gon ve định nghĩa và các tính chat của đa thức. Các van đe nen tảng ve phép chia đa thức, ưóc - b i, nghi m và phương trình b c cao, đạo hàm và khai trien Taylor sẽ đưoc trình bày. • Chương 2. Đa thŕc bat khá quy. Đa thức bat khả quy là m t trong những chủ đe trong tâm của lý thuyet các đa thức. Nó vừa mang tính chat lý thuyet, vừa mang tính ứng dụng, đ c bi t là các bài t p nâng cao trong các đe thi có tính chat tuyen chon. Chương này chúng tôi t p trung nghiên cứu các đa thức bat khả quy trên các vành (trưòng) so quen biet của toán hoc sơ cap. • Chương 3. M t so chủ đe khác. Chương này dành đe nghiên cứu m t so van đe nâng cao của lý thuyet đa thức, mà mục đích của nó là đe hieu biet sâu sac hơn lý thuyet, đong thòi là nen tảng cho các ứng dụng. Các van đe đưoc quan tâm trong chương này là các đa thức nhieu bien, đa thức đoi xứng, phương trình hàm đa thức và đa thức Chebyshev. Lu n văn này đưoc thục hi n tại Trưòng Đại hoc Khoa hoc - Đại hoc Thái Nguyên và hoàn thành vói sụ hưóng dȁn của GS.TSKH. Đ ng Hùng Thang (Trưòng ĐHKHTN - ĐHQG Hà N i). Tác giả xin đưoc bày tỏ lòng biet ơn chân thành và sâu sac tói ngưòi hưóng dȁn khoa hoc của mình, ngưòi đã đ t van đe nghiên cứu, dành nhieu thòi gian hưóng dȁn và t n tình giải đáp những thac mac của tác giả trong suot quá trình làm lu n văn. Tác giả xin trân trong cảm ơn Ban Giám hi u Trưòng Đại hoc Khoa hoc - Đại hoc Thái Nguyên, Ban Chủ nhi m Khoa Toán–Tin, cùng các giảng viên đã tham gia giảng dạy, đã tạo moi đieu ki n tot nhat đe tác giả hoc t p và nghiên cứu. Tác giả muon gửi những lòi cảm ơn tot đep nhat tói t p the lóp Cao hoc Toán khóa 9 (2015-2017) đã đ ng viên và giúp đõ tác giả rat nhieu trong suot quá trình hoc t p. Nhân dịp này, tác giả cũng xin chân thành cảm ơn Sỏ Giáo dục và Đào tạo Hải
  • 9. 6 Viết đề tài giá sinh viên – ZALO:0973.287.149-TEAMLUANVAN.COM Phòng, Ban Giám hi u và các đong nghi p ỏ Trưòng THPT Hùng Vương đã tạo đieu ki n cho tác giả hoàn thành tot nhi m vụ hoc t p và công tác của mình. Cuoi cùng, tác giả muon dành những lòi cảm ơn đ c bi t nhat đen bo me và đại gia đình đã luôn đ ng viên và chia sẻ những khó khăn đe tác giả hoàn thành tot lu n văn này. Thái Nguyên, ngày 02 tháng 11 năm 2017 Tác giả Nguyen Thanh Tùng
  • 10. 7 Viết đề tài giá sinh viên – ZALO:0973.287.149-TEAMLUANVAN.COM Chương 1 Đa th c m t bien 1.1 Định nghĩa và các tính chat Phan này chúng tôi sẽ trình bày ngan gon nhat có the lý thuyet các đa thức m t bien. Những chi tiet hơn có the tham khảo Lê Thị Thanh Nhàn [5] ho c Nguyen Văn M u [3]. 1.1.1 Định nghĩa Giả sử R là m t vành giao hoán có đơn vị. Định nghĩa 1.1.1. Bieu thức có dạng anxn +an−1xn−1 + ...+ a1x +a0 vói an 0 trong đó an, an−1,...,a1, a0 là những phan tử thu c vành R, đưoc goi là m t đa thŕc trên vành R. Trong định nghĩa này, ai đưoc goi là các h so của đa thức, h so an đưoc goi là h so b c cao nhat của đa thức, so tụ nhiên n đưoc goi là b c của đa thức, ký hi u là deg P(x), x đưoc goi là ȁn, hay bien hay đoi so của đa thức, an đưoc goi là h so cao nhat, a0 đưoc goi là h so tự do của đa thức. Neu ai = 0 vói i = 1,2,...,n−1 và a0 0 thì ta có b c của đa thức là không. Neu ai = 0 vói i = 1,2,...,n thì f (x) = 0, ta goi đa thức này là đa thŕc không.
  • 11. 8 Viết đề tài giá sinh viên – ZALO:0973.287.149-TEAMLUANVAN.COM Nói chung ngưòi ta không định nghĩa b c của đa thức không nhưng ta coi b c của nó là −∞. Hai đa thức f và g đưoc goi là bang nhau, và viet f = g, neu chúng cùng là đa thức không, ho c cả hai khác đa thức không, đong thòi deg f = deg g và các h so tương ứng bang nhau. T p hop tat cả các đa thức lay h so trong vành R đưoc ký hi u là R[x], và đưoc goi là vành đa thŕc trên R. Khi R là m t trưòng, thì vành R[x] là m t vành giao hoán có đơn vị. Vói lý do là ứng dụng lý thuyet đa thức trong các bài thi hoc sinh giỏi, hay nói chung là các kỳ thi có tính chat tuyen chon, lu n văn này thưòng xét R là Z, Q, R, ho c C, khi đó các đa thức thu c Z[x], Q[x], R[x], ho c C[x] đưoc goi tên lan lưot là các đa thŕc nguyên, đa thŕc hru tý, đa thŕc thực, ho c đa thŕc phŕc. 1.1.2 Các phép tính trên đa th c Cho hai đa thức f (x) = anxn + an−1xn−1 +...+ a1x + a0, g(x) = bnxn + bn−1xn−1 +... +b1x + b0. Ta định nghĩa các phép tính so hoc như sau • Phép c ng f (x)+ g(x) = (an +bn)xn +(an−1 +bn−1)xn−1 +. . . +(a1 +b1)x+(a0 + b0). • Phép trừ f (x)− g(x) = (an −bn)xn +(an−1 −bn−1)xn−1 +. . .+(a1 −b1)x −(a0 − b0). • Phép nhân f (x)g(x) = c2nx2n + c2n−1x2n−1 + ... + c1x + c0
  • 12. 9 Viết đề tài giá sinh viên – ZALO:0973.287.149-TEAMLUANVAN.COM trong đó ck = a0bk +a1bk−1 +...+akb0 vói k = 0,1,...,n. 1.1.3 Các tính chat cơ ban Định lí 1.1.2. Giá sr F m t trường. Với hai đa thŕc f (x) và g(x) thu c vành đa thŕc F[x], luôn ton tại c p đa thŕc q(x) và r(x) duy nhat thu c vành đa thŕc F[x] đe với degr(x) < degg(x). f(x) = g(x)·q(x)+ r(x), Ta goi q(x) và r(x) lan lưot là đa thŕc thương (ho c thương) và đa thŕc dư (hay dư) trong phép chia f (x) cho g(x). Neu r(x) = 0 thì ta nói f(x) chia het cho g(x), hay g(x) chia het f(x) hay f(x) là b i của g(x) hay g(x) là ước của f(x). Ta sẽ kí hi u là f . g hay g | f . Giả sử f (x) = anxn + an−1xn−1 + ... + a1x + a0 là m t đa thức thu c vành đa thức R[x]. Xét phan tử α ∈ R bat kỳ. Khi đó f (α) = anαn + an−1αn−1 + .. . + a1α + a0 đưoc goi là giá trị của đa thŕc f (x) tại α. Neu f (α) = 0 thì α đưoc goi là m t nghi m của đa thức f (x). Neu ton tại k ∈ N, k > 1 sao cho f (x) . (x−α)k nhưng f (x) không chia het cho (x−α)k+1 thì α đưoc goi là nghi m b i k của đa thức f (x). Đ c bi t, k = 1 thì α đưoc goi là nghi m đơn, k = 2 thì α đưoc goi là nghi m kép. Bài toán tìm nghi m của đa thức f(x) = anxn + an−1xn−1 +...+a1x+a0 vói an = / 0. trong vành R đưoc goi là giái phương trình đại so b c n trong R.
  • 13. 10 Viết đề tài giá sinh viên – ZALO:0973.287.149-TEAMLUANVAN.COM k Định lí 1.1.3. Giá sr F m t trường, α ∈ F và f (x) ∈ F[x]. Dư của phép chia f (x) cho (x −α) chính là f (α). Định lí 1.1.4 (Định lý Bézout). Phan tr α ∈ F là nghi m của đa thŕc f (x) ∈ F[x] khi và chỉ khi f (x) chia het cho (x − α). Định lý sau đây cho ta m t đánh giá ve so nghi m của m t đa thức thục. Định lí 1.1.5. M t đa thŕc thực b c n đeu có không quá n nghi m thực. Định lý này có m t so h quả sau đây. H qua 1.1.6. Đa thŕc có vô so nghi m là đa thŕc không. H qua 1.1.7. Neu m t đa thŕc có b c không vượt quá n mà nh n cùng m t giá trị tại n+1 điem khác nhau của ȁn thì đa thŕc đó là đa thŕc hang. H qua 1.1.8. Hai đa thŕc có b c không vượt quá n mà nh n cùng m t giá trị thóa mãn bang nhau tại n + 1 giá trị khác nhau của ȁn thì hai đa thŕc đó đong nhat bang nhau. Trong trưòng hop đa thức phức, ta có ket quả sau đây ve so lưong nghi m của nó. Định lí 1.1.9. M t đa thŕc phŕc b c n có đúng n nghi m tính cá b i. Cuoi cùng của mục này, ta sẽ trình bày không chứng minh m t ket quả ve dạng bieu dien của các đa thức thục. Định lí 1.1.10. Bat kỳ đa thŕc thực f (x) ∈ R[x] nào có b c n và h so b c cao nhat an /= 0 có the phân tích m t cách duy nhat (không tính thŕ tự) thành các nhân tr m s f (x) = an ∏(x − di) ∏(x2 + bkx + ck) i=1 k=1 với di, bk, sk ∈ R, 2s+ m = n và b2 − 4ck < 0 với m, n ∈ N.
  • 14. 11 Viết đề tài giá sinh viên – ZALO:0973.287.149-TEAMLUANVAN.COM q Trong lý thuyet đa thức, m t phan rat quan trong đó là khảo sát các nghi m hữu tý và nguyên của m t đa thức. Ta có định lý sau đây. Định lí 1.1.11. Xét đa thŕc nguyên f (x) ∈ Z[x] có dạng f (x) = a0xn +a1xn−1 +...+ an−1x+an, a0 0. Nghi m hru tý neu có x = p với gcd(p, q) = 1 thì p là ước của h so tự do và q là ước của h so b c cao nhat, tŕc là p | an, q | a0. H qua 1.1.12. Xét đa thŕc nguyên f (x) ∈ Z[x] có dạng chuȁn tac, tŕc là h so b c cao nhat là 1. Khi đó mői nghi m hru tý neu có của đa thŕc f (x) đeu là nghi m nguyên. 1.2 Phép chia đa th c. ƯỚc chung lỚn nhat và nho nhat 1.2.1 Phép chia đa th c Định nghĩa 1.2.1. M t đa thức d(x) chia het hai đa thức f (x) và g(x) goi là ước chung của f (x) và g(x). Neu d(x) là m t ưóc chung chia het cho moi ưóc chung khác của hai đa thức f (x) và g(x) đúng thì ta goi d(x) là ước chung lớn nhat của f(x) và g(x) . Rõ ràng các ưóc chung lón nhat sai khác hang so, đe bảo đảm tính duy nhat ta có the quy ưóc chon ưóc chung lón nhat dạng chuan tac (h so cao nhat bang 1). Ta kí hi u là d(x) = (f (x),g(x)) = gcd(( f(x),g(x)). 1.2.2 Thu t toán Euclide Ta chia liên tiep f(x) = g(x)·q(x)+ r(x) g(x) = r(x)·q1(x)+r1(x)
  • 15. 12 Viết đề tài giá sinh viên – ZALO:0973.287.149-TEAMLUANVAN.COM r(x) = r1(x) · q2(x) + r2(x) ... rk−2(x) = rk−1(x) · qk(x) + rk(x) rk−1(x) = rk(x ·qk+1(x). Khi đó (f(x),g(x)) = rk ∗(x) vói rk ∗(x) = c·rk(x) đa thức có h so b c cao nhat là 1 (ngưòi ta thưòng goi những đa thức này là đa thŕc chuȁn tac hay đa thŕc monic). Ket quả neu d(x) = ( f (x),g(x)) thì ton tại hai đa thức u(x),v(x) ∈ R[x] đe có bieu dien f(x)·u(x)+ g(x)·v(x) = d(x). Hơn nữa ta có the chon degu < degg và degv < deg f. Bây giò ta xét m t so ví dụ đe hieu sâu sac hơn lý thuyet. Các ví dụ này chúng tôi tham khảo trong Lê Hoành Phò [6]. Bài toán 1.2.2. Cho P(x) = x + x3 +x9 +x27 +x81 +x243 . Tìm dư của phép chia đa thŕc P(x) cho (a) x−1, (b) x2 −1. Lời giái. (a) Ta có P(x) = (x − 1)Q(x) + r(x) vói degr(x) < deg(x − 1) = 1. Suy ra degr(x) = 0 nên dư r(x) = c. Do đó P(x) = (x − 1) · Q(x) + c. Chon x = 1 suy ra P(1) = c hay c = P(1) = 6. V y dư r(x) = 6. (b) Ta có P(x) = (x2 − 1)· H(x)+s(x) vói degs(x) ≤ 1. V y P(x) = (x2 − 1)·H(x)+ax+b. Chon x = 1 ta có P(1) = a + b = 6; x = −1 ta có P(−1) = −a + b = −6. Do đó a = 6, b = 0. V y ta có đa thức dư là r(x) = 6x.
  • 16. 13 Viết đề tài giá sinh viên – ZALO:0973.287.149-TEAMLUANVAN.COM Bài toán 1.2.3 (Trung Quoc 1981). Tìm dư của phép chia (a) x12 +x8 +x4 +1 cho x3 +x2 +x+1; (b) f(x100 ) cho f(x) với f(x) = x99 +x98 +...+x+1. Lời giái. (a) Ta có x12 +x8 +x4 +1 = (x3 +x2 +x+1)(x9 −x8 +2x5 + 2x4 +3x−3)+4. Suy ra dư là 4. (b) Ta có nên f (x) = x99 + x98 + ... + x + 1 f (x100 ) = x9900 + x9800 + ... + x100 + 1 = f (x) x9800 + 2x9701 −2x9700 +3x9601 −3x9600 + ... + 99x − 99) + 100 nên dư là 100. Bài toán 1.2.4. Xác định a và b đe đa thŕc f (x) = 6x4 − 7x3 + ax2 + 3x + 2 chia het cho x2 −x+b. Lời giái. Lay đa thức f (x) = 6x4 − 7x3 +ax2 +3x +2 chia cho g(x) = x2 − x +b thì đưoc thương q(x) = 6x2 − x + (a − 5b − 1) và phan dư r(x) = (a − 5b + 2)x + (−ab−6b2 +a+b+2). Vì f (x) . g(x) nên r(x) = 0 suy ra a− 5b +2 = 0, (1.1) —ab +6b2 +b+2 = 0. (1.2) Từ (1.1) suy ra a = 5b − 2. Thay vào (1.2) ta có b2 + 3b + 2 = 0 nên b = −1 ho c b = −2. Vói b = −1 thì a = −7. Vói b = −2 thì a = −12. V y f (x) = 6x4 − 7x3 − 7x2 + 2 hay f (x) = 6x4 − 7x3 − 12x2 + 3x + 2.
  • 17. 14 Viết đề tài giá sinh viên – ZALO:0973.287.149-TEAMLUANVAN.COM 3 2 2 2 2 2 2 Bài toán 1.2.5. Tìm ước chung lớn nhat của hai đa thŕc f(x) = x2 + x3 − 3x2 − 4x−1 và g(x) = x3 +x2 −x−1. Lời giái. Ta thục hi n các phép chia liên tiep và hő tro vói phép nhân thêm hang so thì f(x) = q(x)·g(x)+r(x) q(x) = x, r(x) = −2x2 − 3x − 1 và g(x) = q1(x) · r(x) + r1(x) thì q1(x) = − 1 x− 1 , r1(x) = − 3 x − 3 , và tiep tục r(x) = q2(x) · r1(x) + r2(x) thì q2(x) = − 2 (2x + 1), r2(x) = 0 Do đó ( f(x),g(x)) = x + 1 vói quy ưóc lay h so cao nhat bang 1 từ r1(x) = −3 x− 3 . Bài toán 1.2.6 (New York 1973, Bỉ 1981). Chŕng minh rang với moi giá trị n ∈ N, đa thŕc (x+1)2n+1 +xn+2 chia het cho đa thŕc x2 +x+1. Lời giái. Ta chứng minh bang quy nạp theo n ∈ N. Vói n = 0 khȁng định đúng vì khi đó (x + 1)2n+1 + xn+2 = x2 + x. Giả sử khȁng định đúng vói n − 1, nghĩa là (x + 1)2n−1 + xn+1 chia het cho x2 + x+1. Khi đó đa thức (x+1)2n+1 +xn+2 = (x +1)2 ·(x + 1)2n−1 +x·xn+1 = (x2 +2x+1)(x+1)2n−1 +x ·xn+1 = (x2 + x + 1)(x + 1)2n−1 + x[x + 1)2n−1 + xn+1 ] chia het cho x2 + x + 1. Suy ra đieu chứng minh vói n.
  • 18. 15 Viết đề tài giá sinh viên – ZALO:0973.287.149-TEAMLUANVAN.COM Bài toán 1.2.7. Cho hai so nguyên dương n và k. Chŕng minh rang xn − 1 . xk − 1 khi và chỉ khi n là b i so của k. Lời giái. Ta có the phát bieu bài toán dưói dạng: Đe xn − 1 . xk − 1 đieu ki n can và đủ là n là b i so của k. Đieu ki n đủ. Giả sử n là b i so của k tức là n = km vói m nguyên dương. Khi đó xn −1 = xkm −1 = (xk )m −1 = (xk −1)[x(k−1) +xk(m−1) +xk(m−2) +...+xk +1]. Đȁng thức này chứng tỏ rang xn − 1 . xk − 1. Đieu ki n can. Ta lay so nguyên dương n chia cho so nguyên dương k. Giả sử q và r là thương và so dư trong phép chia, tức là có n = kq + r vói 0 ≤ r < k. Khi đó xn − 1 = xkq+r − 1 = xkq+r − xr + xr − 1 = xr (xkq − 1) + xr − 1. (1.3) Ő trên ta đã chứng minh xkq −1 . xk −1. Vì v y neu xn −1 . xk −1 thì từ (1.3) suy ra xr −1 . xk −1. Nhưng r < k nên xr −1 . xk −1 khi r = 0. Thành ra neu xn −1 . xk −1 thì r = 0, tức là n = kq, hay n là b i so của k. Bài toán 1.2.8 (Rumani 1962). Tìm đieu ki n của các so nguyên p và q sao cho đa thŕc (a) P(x) = x2 + px + q nh n cùng giá trị chȁn (lé) với moi x ∈ Z. (b) Q(x) = x3 + px + q nh n các giá trị chia het cho 3 với moi x ∈ Z. Lời giái. (a) P(x) nh n giá trị cùng chȁn (ho c lẻ) vói moi x ∈ Z khi và chỉ khi mői so P(x + 1)− P(x) = 2x + 1+ p chia het cho 2 nghĩa là p lẻ.
  • 19. 16 Viết đề tài giá sinh viên – ZALO:0973.287.149-TEAMLUANVAN.COM Khi đó tính chȁn lẻ của P(x) phụ thu c vào tính chȁn lẻ của q = P(0). Như v y tat cả giá trị của P(x) là chȁn (lẻ) khi p lẻ và q chȁn (tương ứng q lẻ). (b) Vì Q(x) = x(x2 + p) + q nên Q(3x) = 3x(9x2 + p)+ q chia het cho 3. Vói giá trị đó thì Q(3x± 1) = (3x ±1)(9x2 ±6x +1+ p)+q ≡ ±(1+ p) (mod 3) chia het cho 3 khi và chỉ khi 1 + p chia het cho 3. V y Q(x) chia het cho 3 (vói moi x ∈ Z) khi q = 0, q ≡ 2 (mod 3). Bài toán 1.2.9 (Hong Kong 2008). Xét đa thŕc f(x) = cmxm +cm−1xm−1 +...+c1x +c0 với các h so ci là nhrng so nguyên khác không. Xây dựng dãy so (an) như sau: a1 = 0 và an+1 = f (an) với n = 1,2,... Giá sr i, j là nhrng so nguyên dương vài < j. Chŕng minh rang (aj+1 −aj) là b i của (ai+1 −ai). Chŕng minh. Do f(x) là đa thức h so nguyên nên [f(a) − f(b)] chia het cho (a−b), vói a và b là hai so nguyên phân bi t. Do đó ai+2 − ai+1 = f (ai+1) − f (ai) chia het cho (ai+1 − ai), vói moi i > 0. Như v y neu i, j là những so nguyên dương và i < j thì (aj+1 − a j) chia het cho (ai+1 −ai). Ta có đieu phải chứng minh. 1.3 Nghi m cua đa th c. Phương trình b c cao 1.3.1 Nghi m cua đa th c Bài toán 1.3.1. Cho đa thŕc b c chȁn và tat cá h so đeu lé. Chŕng minh đa thŕc không có nghi m hru tý. Chŕng minh. Xét P(x) = a0xn +a1xn−1 +...+an−1x+an vói a0 /= 0.
  • 20. 17 Viết đề tài giá sinh viên – ZALO:0973.287.149-TEAMLUANVAN.COM q q p p2 ps Vói n chȁn, các h so ai lẻ. Giả sử đa thức có nghi m hữu tý x = p thì p | a0, q | an. Suy ra p, q lẻ. The x = p vào đa thức ta có an pn + an−1qpn−1 + ... + a0qn = 0. Đieu này vô lý vì ve trái là tong của m t so lẻ các so hạng lẻ nên không the bang 0. V y đa thức không có nghi m hữu tý. Ta có đieu phải chứng minh. Bài toán 1.3.2. Cho so tự nhiên n ≥ 2, chŕng minh phương trình xn xn−1 x2 x n! + (n−1)! +...+ 2! + 1! +1 = 0. (1.4) không có nghi m hru tý. Chŕng minh. Ta chứng minh bài toán bang phương pháp phản chứng. Giả sử phương trình đã cho có nghi m hữu tý α. Khi đó α sẽ là nghi m hữu tý của đa thức P(x) = xn + nxn−1 +...+ xk k! ... + n! x2 2! n! x + 1! n!. Nhưng do P(x) là m t đa thức b c n vói h so nguyên, và h so b c cao nhat, của xn , là 1 nên α phải là so nguyên và ta có αn +nαn−1 +...+ αk k! +...+ n! α2 1! n! = 0. (1.5) Goi p là m t ưóc nguyên to của n. Vói moi k = 1, 2, . . . , n, kí hi u rk là so mũ cao nhat của p thỏa mãn k! . prk, ta có rk = k + k +...+ k . (1.6) Vói s là so nguyên không âm thỏa mãn ps ≤ k < ps+1 , từ (1.6) ta suy ra r k k k p2 k ≤ p + p2 + ... + ps = k p − 1 < k. Do đó rn −rk > rn − k. Suy ra rn − rk ≥ rn − k +1. Vì v y ta đưoc n! : pn−k+1 vói moi k = 1,2,...,n. (1.7) k! n! + n! +
  • 21. 18 Viết đề tài giá sinh viên – ZALO:0973.287.149-TEAMLUANVAN.COM n 2 2 — − 2 Nhưng mà n . p nên từ (1.5) ta có αn . p và do đó α . p. Suy ra αk . pk vói moi k = 1,2,...,n. Ket hop đieu này vói (1.7) ta đưoc n!αk . pr+1 vói moi k = 1,2,...,n. Từ đây và (1.5) ta suy ra n! . pr+1 . k! n Mâu thuȁn vừa nh n đưoc chứng tỏ giả sử ban đau là sai và vì v y ta có đieu phải chứng minh. Bài toán 1.3.3 (Vi t Nam 1992). Cho đa thŕc P(x) = 1+x2 +x9 +xn1 +...+xns +x1992 . Với n1,...,ns là các so tự nhiên cho trước thóa mãn 9 < n1 < ... < ns < 1992. Chŕng minh rang nghi m của đa thŕc P(x) (neu có) không the lớn hơn 1− √ 5 . Lời giái. Ta có P(x) = 1+x2 +x9 +xn1 +...+xns +x1992 . Vói x ≥ 0 thì P(x) ≥ 1 > 0. Ta sẽ chứng minh P(x) > 0 vói moi x ∈ 1− √ 5 ;0 ! . Th t v y vói x < 0 và x /= −1 ta có P(x) ≥ 1 + x + x3 + x5 + ... + x2k+1 + ... + x1991 (x1990 +x1998 +...+1).(1−x2 ) = 1+x = 1+x 1 − x996 1 − x2 (1 − x2) 1 x2 +x x997 = 1−x2 . Mà vói x ∈ 1− √ 5 ;0 thì 1 − x2 > 0, −x997 > 0, 1 − x2 + x > 0.
  • 22. 19 Viết đề tài giá sinh viên – ZALO:0973.287.149-TEAMLUANVAN.COM 2 2 nên P(x) > 0 vói moi x ∈ 1− √ 5 ;0 ! . V y P(x) > 0 vói x ∈ 1− √ 5 ;+∞ . Ta có đieu phải chứng minh. Bài toán 1.3.4 (IMO 1976). Cho các đa thŕc Pk(x), với k = 1,2,3... xác định bới P1(x) = x2 − 2, Pi+1 = P1(Pi(x)), với i = 1,2,3,... Chŕng minh rang Pn(x) = x có 2n nghi m thực phân bi t. Lời giái. Ta thu hep vi c xét nghi m của phương trình trên đoạn −2 ≤ x ≤ 2. Đ t x = 2cos(t). Khi đó, bang quy nạp ta chứng minh đưoc Pn(x) = 2cos(2nt ). Thêm nữa, phương trình Pn(x) = x trỏ thành cos(2nt ) = cos(t). Từ đó ta đưoc 2n nghi m t = 2kπ , t = 2kπ , k = 1,2,...,n 2n −1 2n + 1 V y phương trình Pn(x) = x có 2n nghi m thục phân bi t. Bài toán 1.3.5. Cho đa thŕc f(x) = a0 +a1x+...+anxn có n nghi m thực. Chŕng minh với moi p > n−1 thì đa thŕc g(x) = a0 + a1.p.x + a2.p(p − 1)x2 + ... + an p(p − 1)...(p − n + 1)xn . cũng có n nghi m thực. Lời giái. Đe giải bài toán ta xét hai trưòng hop Trường hợp 1. Đa thức f (x) không nh n x = 0 làm nghi m. Ta chứng minh bang quy nạp. Vói n = 1 bài toán hien nhiên đúng .
  • 23. 20 Viết đề tài giá sinh viên – ZALO:0973.287.149-TEAMLUANVAN.COM Giả sử đúng vói n = k, ta chứng minh đúng vói n = k + 1, tức là neu đa thức f (x) = a0 +a1x +... + ak+1xk+1 có k +1 nghi m thục khác 0 thì đa thức g(x) = a0 + pa1x + ... + p(p − 1)...(p − k)ak+1xk+1 cũng có k +1 nghi m thục khác 0 vói moi p > k. Goi c là m t nghi m của f (x) thì f(x) = (x−c)q(x), (1.8) vói q(x) là m t đa thức nào đó là đa thức b c k của x, q(x) = b0 + b1x + ... + bkxk . (1.9) Thay (1.9) vào (1.8) roi đong nhat h so ta đưoc a0 = cb0, a1 = cb1 + b0, ... ak = cbk + bk−1, ak+1 = bk Do đó trong đó g(x) = a0 + pa1x + ... + p(p − 1)...(p − k)ak+1xk+1 = cb0 + p(cb1 + b0)x + .. . + p(p − 1)(p − k)bkxk+1 = cQ(x) + pxQ(x) − x2 Q(x) Q(x) = b0 + b1 px + ... + p(p − 1)...(p − k + 1)bkxk . Do f (x) có k +1 nghi m thục khác 0 nên q(x) có k nghi m thục khác 0. M t khác p > k nên p > k −1. Cho nên theo giả thiet quy nạp ta có đa thức Q(x) có k nghi m thục. Do đó g(x) có k +1 nghi m thục. V y theo nguyên lý quy nạp, bài toán đúng. Trường hợp 2. Đa thức f (x) nh n x = 0 làm nghi m.
  • 24. 21 Viết đề tài giá sinh viên – ZALO:0973.287.149-TEAMLUANVAN.COM d Giả sử x = 0 là nghi m b i k của f(x), vói k ∈ Z+, k ≤ n. Khi đó ta có f (x) = akxk + ... + anxn = (anxn−k + ... + ak)xk và g(x) = p(p − 1)...(p − k + 1)ak.xk +... + p(p − 1)...(p −n + 1)an.xn = p(p− 1)...(p −k + 1)xk [ak +...+(p −k)...(p − n+ 1)anxn−k ]. Vì f (x) có n nghi m thục nên H(x) = ak + ... + anxn−k có (n − k) nghi m thục khác 0. Do đó áp dụng ket quả của Trưòng hop 1 cho H(x) và p′ = p − k > n − k − 1 (do p > n − 1), ta đưoc đa thức R(x) = ak + ... +(p − k)...(p − n + 1)anxn−k có n − k nghi m thục. V y g(x) có n nghi m thục. Bài toán 1.3.6 (Trung Quoc 1996). Cho đa thŕc p(x) b c 5 có 5 nghi m thực phân bi t. Tìm so bé nhat của các h so khác 0. Lời giái. Xét p(x) = ax5 +bx4 +cx3 +dx+e, a 0. Neu có 4 h so bang 0 thì b = c = d = e = 0 nên p(x) = ax5 có nghi m b i (loại) tức là p(x) không the có m t h so khác 0. Do đó p(x) có ít nhat hai h so khác 0. Xét p(x) = ax5 +bxn , n ≥ 2 thì p(x) có nghi m b i. Ta tiep tục loại trưòng hop này. Xét p(x) = ax5 +dx = ax ax4 + a có toi đa ba nghi m. Ta cũng loại. Xét p(x) = ax5 +e có m t nghi m. Ta cũng loại. Do đó p(x) có ít nhat ba h so khác 0. Chon p(x) = x5 −5x3 +4x = x(x2 −4) thì p(x) có đúng 5 nghi m phân bi t và đúng ba h so khác 0. V y so bé nhat của h so khác 0 là 3.
  • 25. 22 Viết đề tài giá sinh viên – ZALO:0973.287.149-TEAMLUANVAN.COM 3 3 27 3 p 2p √ p 1.3.2 Phương trình b c cao Lý thuyet giai phương trình b c 3 tong quát Xét phương trình đa thức b c ba ax3 + bx2 + cx + d = 0, a /= 0. (1.10) Ngoài vi c tách nhóm so hạng ho c tìm m t nghi m roi phân tích nhân tử, dùng hang đȁng thức, ta có cách giải tong quát như sau: Trưóc het, chia 2 ve cho a /= 0 đưa ve phương trình: x3 + Bx2 +Cx + D = 0. Tiep theo đ t x = y − B đưa tiep ve phương trình: y3 − py = q, trong đó p b2 2B3 BC = 3 −C, q = − 27 + 3 − D. Có hai hưóng đe giải phương trình y3 − py = q. (1.11) Hướng thŕ nhat. Đ t y = u + v và chon u, v = p thì từ y3 = u3 + v3 + 3uv(u + v) ta có u3 +v3 = q và u3 .v3 = P3 . V y u3 ,v3 là nghi m phương trình 3 Z3 −qZ + 27 = 0. Neu ∆ < 0 sau này ta dùng so phức đe giải quyet. Hướng thŕ hai. Neu p = 0 thì (1.11) tương đương vói y3 = q, tức là y = √ q. Neu p > 0. Đ t y = 2 q p t thì (1.11) tương đương vói 4t3 − 3t = m (1.12) 3 √ 3q trong đó m = . Xét |m| ≤ 1, đ t m = cosα thì (1.12) có ba nghi m t1 = cos α , t2 = cos α +2π , t3 = cos α − 2π . 3 3 3
  • 26. 23 Viết đề tài giá sinh viên – ZALO:0973.287.149-TEAMLUANVAN.COM 3 2 2 Xét |m| > 1, đ t m = 1 d3 + 1 suy ra d3 = m ± √ m2 − 1. Phương trình 2 d 2 2 k3 2 k 2 2 d3 (1.12) có m t nghi m t = 1 d + 1 = 1 q 3 m+ √ m2 −1+ q 3 m− √ m2 −1 Neu p < 0. Đ t y = 2. q − p t thì (1.11) tương đương vói 4t3 + 3t = m. (1.13) Ta đ t tiep m = 1 k3 − 1 suy ra k3 = m ± √ m2 +1. Phương trình (1.13) có m t nghi m t = 1 k − 1 = 1 q 3 m+ √ m2 + 1+ q 3 m − √ m2 +1 . Ta goi các phương trình b c ba 4x3 + 3x + −m = 0, 4x3 −3x − m = 0 là các dạng phương trình b c 3 chuȁn tac. Ý nghĩa cơ bản là moi phương trình b c 3 đeu đưa ve đưoc dạng chuan tac đó. Chú ý thêm khi |m| ≥ 1, 4x3 +3x−m = (x−α)(4x2 +4αx+4α2 +3). Vói α = 1 q 3 m+ √ m2 + 1+ q 3 m− √ m2 + 1 có ∆′ = −12(α2 + 1) < 0 và 4x3 − 3x − m = (x − β)(4x2 + 4βx + 4β 2 + 3). Vói β = 1 q 3 m+ √ m2 −1+ q 3 m− √ m2 −1 có ∆′ = −12(1 − β 2) < 0.
  • 27. 24 Viết đề tài giá sinh viên – ZALO:0973.287.149-TEAMLUANVAN.COM 2 ax Các phương trình b c 4 đ c bi t 1. Phương trình có dạng ax4 + bx2 + c = 0, a 0. (1.14) Đ t t = x2 , t ≤ 0 thì đưa ve phương trình b c hai at2 + bt + c = 0 2. Phương trình có dạng (x +a)4 +(x+ b)4 = c. (1.15) Đ t t = x+ a+b thì đưa ve phương trình trùng phương At4 +Bt2 +C = 0 và giải như trên. 3. Phương trình có dạng (ax2 +bx +c)(ax2 +bx +d) = m. (1.16) Đ t t = ax2 +bx thì đưa ve phương trình b c hai (t +c)(t +d) = m. 4. Phương trình có dạng (x +a)(x + b)(x +c)(x +d) = m. (1.17) Neu có a + d = b + c thì ghép c p (x + a)(x + d) và (x + b)(x + c) roi đ t t = x2 +(a+d)x = x2 +(b+c)x đe đưa ve dạng trên. 5. Phương trình có dạng ax4 + bx3 + cx2 + dx + e = 0. (1.18) Neu ad2 = eb2 /= 0 thì chia hai ve cho x2 /= 0 roi đ t t = x+ e (Phương t r ì n h quy hoi mỏ r ng b c bon).
  • 28. 25 Viết đề tài giá sinh viên – ZALO:0973.287.149-TEAMLUANVAN.COM x 2 p x x Phương trình quy hoi (đoi x ng h so) Xét phương trình a0xn + a1xn−1 + a2xn−2 + ··· + an−2x2 + an−1x + an = 0 (1.19) trong đó a0 = an; a1 = an−1; a2 = an−2;... Xét n chȁn, n = 2m. Chia hai ve cho xm /= 0.. Đ t t = x + 1 , |t| ≥ 2 đưa ve phương trình b c m = n . Xét n lẻ, n = 2m + 1. Phương trình có nghi m x = −1 nên phân tích ra thừa so (x+1) và thừa so b c 2m lại là phương trình quy hoi b c chȁn như trên. Đôi khi ta mỏ r ng dạng quy hoi (quy hoi kèm tỉ l ) vói cách đ t t = x − 1 , t = x + a Phương trình b c cao Xét phương trình a0xn + a1xn−1 + ··· + an−1x + an = 0, a0 0. (1.20) Nguyên tac chung là bien đoi ve dạng tích, đ t an phụ đe đưa ve phương trình b c thap hơn. Đ c bi t • Neu tong các h so bang 0 thì có nghi m x = 1. • Neu tong đan dau các h so bang 0 thì có nghi m x = −1 . • Nhac lại, nghi m hữu tỉ neu có thì có dạng x = q vói p | an và q | a0. The trục tiep ho c dúng sơ đo Hoocne đe thử nghi m. Đôi khi phương trình b c cao đoi vói bien x mà lại b c thap đoi vói tham so thì ta chuyen ve phương trình theo an là tham so đó.
  • 29. 26 Viết đề tài giá sinh viên – ZALO:0973.287.149-TEAMLUANVAN.COM 2 2 2 q Bài toán 1.3.7. Giái các phương trình sau (a) 4x3 − 10x2 + 6x − 1 = 0. (b) 8x3 − 36x + 27 = 0. Lời giái. (a) Ta có 4x3 −10x2 +6x −1 = 0 ⇔ 4x3 −2x2 −8x2 +4x+ 2x −1 = 0 ⇔ 2x (2x −1)−4x(2x − 1)+(2x −1) = 0 ⇔ (2x − 1)(2x −4x +1) = 0 ⇔ 2x −1 = 0 ho c 2x − 4x + 1 = 0. V y nghi m của phương trình là 1 x = 2 , x = 2 ± √ 2 . 2 Bài toán 1.3.8. Tìm quan h gira p và q đe phương trình x3 + px + q = 0 có the viet dưới dạng x4 = (x2 − ax + b)2 . Áp dnng ket quá đó đe giái phương trình: x3 −18x+27 = 0. Lời giái. Ta có Suy ra x4 − (x2 − ax + b)2 = −m(x3 + px + q). a2 + 2b = 0, 2ab = pm, 2a = m,b2 = qm. Từ đó b = p, kéo theo p2 = mq. V y m = p 2 . M t khác a2 = −2b ⇒ a = m ⇒ m2 = −2b = −2p ⇒ m2 = −8p. Từ đó p 4 = 2 4 q2 −8 ⇒ p4 + 8pq2 = 0. V y quan h giữa p và q: p3 +8q2 = 0. Ta có x3 − 18x + 27 = 0 ⇔ x4 = (x2 + 6x − 18)2
  • 30. 27 Viết đề tài giá sinh viên – ZALO:0973.287.149-TEAMLUANVAN.COM 3 2 2 3 2 2 3 ⇔ (6x − 18)(2x2 + 6x − 18) = 0 x = 3 ho c x = −3 ± √ 35 . 2 Bài toán 1.3.9. Giái và bi n lu n phương trình x3 − 3x2 + 3(a + 1)x − (a + 1)2 = 0. (1.21) Lời giái. Ta có x3 − 3x2 + 3(a + 1)x − (a + 1)2 = 0 tương đương vói −x = −3x +3(a +1)x −(a +1) . Neu a = / −1 thì nhân hai ve vói (a+1), ta đưoc −x (a + 1) = −3x (a + 1) + 3(a + 1) x − (a + 1) C ng hai ve vói x3 ta đưoc −ax3 = (x−a−1)3 . Từ đó ta đưoc x−a−1 = − √ 3 ax, suy ra a + 1 x = √ 3 a+ 1 = a2 − √ 3 a. Neu a = −1 thì de thay phương trình có hai nghi m x1 = 0 và x2 = 3. Bài toán 1.3.10. Giái các phương trình sau (a) x4 − 2x3 − 6x2 + 16x −8 = 0; (b) x4 + x2 + 4x − 3 = 0; Lời giái. (a) Ta có x4 − 2x3 −6x2 +16x − 8 = 0 ⇔ (x− 2)(x3 − 6x +4) = 0 ⇔ (x − 2)(x − 2)(x2 + 2x − 2) = 0 ⇔ (x − 2)2 (x2 +2x − 2) = 0. V y nghi m của phương trình là x = 2, x = −1 ± √ 3. ⇔ √ 3
  • 31. 28 Viết đề tài giá sinh viên – ZALO:0973.287.149-TEAMLUANVAN.COM 2 2 2 2 2 2 2 0 x x x0 x0 0 2 0 x0 0 (b) Ta có x4 +x2 +4x−3 = 0 ⇔ x4 = −x2 −4x +3 ⇔ (x + 1) = (x −2) ⇔ (x +1) −(x −2) = 0 ⇔ (x −x+3)(x + x −1) = 0 V y nghi m của phương trình là x = −1± 2 √ 5 . Bài toán 1.3.11 (IMO 1973). Giá sr phương trình x4 + ax3 + bx2 + ax + 1 = 0 có nghi m. Tìm giá trị bé nhat của a2 +b2 . Lời giái. Goi x0 là nghi m của phương trình đã cho, tức là x4 +ax3 +bx2 +ax0 +1 = 0 0 0 0 Từ đây ta có ket lu n x0 /= 0 . Chia hai ve cho x2 , ta có x2 + ax0 + b + a + 1 = 0. 0 Phương trình này tương đương vói x0 2 x2 + 1 + a x0 + 1 +b = 0. Đ t y = x0 + 1 vói đieu ki n |y| = |x0|+| 1 | ≥ 2. Ta có (y2 − 2)+ay +b = 0 Suy ra |2− y | = |ay + b| ≤ √ a2 + b2 √ y2 +1 ⇒ a2 +b2 ≥ (2 − y2 )2 1 + y2 Đ t t = y2 , t ≥ 4. Ta chứng minh (2 −t)2 4 1+t ≥ 5 (1.22) Th t v y, ta có (1.22) tương đương vói 5(2 −t)2 ≥ 4(1+t), tức là 5t2 − 24t + 16 ≥ 0. Nhưng đieu này đúng vì t ≥ 4. Như v y giá trị bé nhat của a2 + b2 là 4/5. 2
  • 32. 29 Viết đề tài giá sinh viên – ZALO:0973.287.149-TEAMLUANVAN.COM √ 2 3 1 0 −3x = 4x −x x ≥ 2, 4x−x2 ≥ 0 x4 − 8x3 + 16x2 + 27x − 90 = 0. Bài toán 1.3.12 (Vi t Nam 2002). Giái phương trình q 4 − 3 √ 10 − 3x = x − 2 (1.23) Lời giái. Ta có bien đoi tương đương như sau q 4− 3 √ 10 −3x = x −2 ⇔ x ≥ 2 4−3 √ 10−3x = (x−2)2 ⇔ x ≥ 2 ⇔ 9(10 − 3x) = (4x − x2 )2 ⇔ x ≥ 2, 0 ≤ x ≤ 4 ⇔ 2 ≤ x ≤ 4 Bây giò ta sẽ giải phương trình x4 − 8x3 + 16x2 + 27x − 90 = 0. x4 − 8x3 + 16x2 + 27x − 90 = 0. (1.24) Bang cách thử trục tiep ta thay x = 3 là m t nghi m nên phương trình (1.24) đưoc viet lại thành (x − 3)(x + 2)(x2 − 7x + 15) = 0. V y phương trình có nghi m duy nhat x = 3. Bài toán 1.3.13 (Vi t Nam 1991). Giái phương trình x3 −3x2 − 8x+ 40 = 8 √ 4 4x+4. (1.25)
  • 33. 30 Viết đề tài giá sinh viên – ZALO:0973.287.149-TEAMLUANVAN.COM 2 2 2 8 8 8 8 8 3 3 3 3 8 3 3 3 3 Lời giái. Từ phương trình x3 − 3x2 − 8x+ 40 = 8 √ 4 4x+ 4 ta có đieu ki n x ≥ −1. Áp dụng bat đȁng thức AM-GM ta có Do đó 8 √ 4 4x+4 = 4 √ 4 4.4.4.(x + 1) ≤ x + 13. x3 −3x2 −8x+40 ≤ x +13 ⇔ x3 −3x2 −9x +27 ≤ 0 ⇔ (x + 3)(x − 6x + 9) ≤ 0 ⇔ (x +3)(x− 3) ≤ 0. Vì x ≥ −1 nên (x − 3)2 ≥ 0, suy ra x = 3. Thử lại ta thay đúng. V y phương trình có nghi m duy nhat x = 3. Bài toán 1.3.14. Chŕng minh rang (a) x = r 3 a+ a+1 q 8a−1 + r 3 a− a+1 q 8a−1 với a ≥ 1 là so tự nhiên (b) √ 3 2+ √ 3 4 là so vô tỉ. Lời giái. (a) Áp dụng hang đȁng thức (u+v)3 = u3 +v3 +3uv(u+v) ta có x3 = 2a +(1 −2a)x ⇔ x3 +(2a − 1)x − 2a = 0 ⇔ (x − 1)(x + x + 2a) = 0. Xét đa thức b c hai x2 + x + 2a có ∆ = 1 − 8a ≥ 0 Khi a = 1 , ta có x = q 3 1 + q 3 1 = 1 Khi a > 1 , ta có 1 − 8a âm nên đa thức x2 + x + 2a có nghi m thục duy nhat x = 1. V y vói a ≥ 1 thì x = s 3 a+ a+ 1 r 8a−1 + s 3 a− a+ 1 r 8a−1
  • 34. 31 Viết đề tài giá sinh viên – ZALO:0973.287.149-TEAMLUANVAN.COM là m t so tụ nhiên. (b) Đ t x = √ 3 2+ √ 3 4. Ta có x3 = 2+ 4+3 √ 3 8( √ 3 2+ √ 3 4) Tức là x3 − 6x − 6. Giả sử x ∈ Q. Do h so b c cao nhat của đa thức là 1 nên x là so nguyên. Ta có 2 < √ 3 2+ √ 3 4 < 4 nên x = 3. Do đó x3 −6x−6 = 3, vô lý. V y x là so vô tý. Bài toán 1.3.15 (Vi t Nam 1984). Tìm đa thŕc theo x có b c bé nhat với h so nguyên, biet m t nghi m là √ 2+ √ 3 3. Lời giái. Đ t a = √ 2+ √ 3 3 ta có a2 = 2+ 2 √ 2. √ 3 3+ √ 3 9 và Ta rút ra a3 = 2 √ 2+ 6 √ 3 3+ 3 √ 2. √ 3 9+ 3 (1.26) √ 3 3 = a− √ 2, √ 3 9 = a2 −2−2 √ 2(a− √ 2) = a2 + 2− 2 √ 2a. Thay vào (1.26) ta có a3 = 2 √ 2 + 6(a − √ 2) + 3 √ 2(a2 + 2 − 2 √ 2a) + 3, a3 + 6a − 3 = √ 2(3a2 + 2). Bình phương hai ve ta thay a là nghi m của đa thức x6 − 6x4 − 6x3 + 12x2 − 36x + 1 Bang phép đong nhat h so ta chứng minh đa thức trên không phân tích đưoc thành hai đa thức b c thap hơn có h nguyên nên đa thức trên chính là đa thức có b c bé nhat thỏa đe bài.
  • 35. 32 Viết đề tài giá sinh viên – ZALO:0973.287.149-TEAMLUANVAN.COM 1.4 Đạo hàm cua đa th c. Định lý Taylor Đạo hàm của m t hàm so là m t khải ni m thu c nhánh Giải tích toán hoc. Trong mục này chúng tôi sẽ trình bày ngan gon ve đạo hàm của hàm so đa thức. Định nghĩa 1.4.1. Cho f(x) ∈ R[x] và deg f(x) = n, anxn + an−1xn−1 + ... + a1x + a0 vói an 0 • Đa thức nanxn−1 + (n − 1)an−1xn−2 + ... + 2a2x + a1 đưoc goi là đạo hàm cap m t của đa thŕc f (x) (ho c ngan gon là đạo hàm của đa thŕc f (x)), và đưoc ký hi u là f′(x). Bang quy nạp, ta định nghĩa đạo hàm cap k của đa thức f (x) là đạo hàm của đạo hàm cap k − 1. Ta ký hi u đạo hàm cap 2, 3 và cap k tong quát lan lưot là f ′′(x), f ′′′(x), và f (k)(x). • Vói moi x0 ∈ R, bieu thức sau đây đưoc goi là khai trien Taylor của đa thức thục f(x) f f ′(x0) f (k)(x0) k f (n)(x0) n (x) = f (x0)+ 1! (x −x0)+...+ k! (x − x0) +...+ n! (x − x0) . Từ định nghĩa này ta có neu deg f = n thì deg f ′ = n − 1, deg f ′′ = n − 2,..., deg f (k) = n − k vói 1 ≤ k ≤ n, deg f (x) = 0. Bây giò ta xét m t so bài toán ve đạo hàm hàm và khai trien Taylor. Chúng tôi tham khảo [6] trong trình bày. Bài toán 1.4.2 (Ba Lan 1979). Cho đa thŕc P(x) có b c n > 1 và có n nghi m thực x1,x2,x3,...,xn phân bi t. Chŕng minh rang 1 1 1 P′(x1) + P′(x2) + ... + P′(xn) = 0.
  • 36. 33 Viết đề tài giá sinh viên – ZALO:0973.287.149-TEAMLUANVAN.COM − i=1 P′(x ) i Lời giái. Đ t ta suy ra P(x) = a(x−x1)(x−x2)...(x−xn),a = / 0 n P′(x) = P1(x)+P2(x)+...+Pn(x) vói Pi(x) = Ta thay ∏ j=1, j = / (x x j). i Pi(xj) = 0 vói moi j /= i kéo theo P′(xj) = Pj(xj) Xét đa thức 0 vói moi j = 1,2,...,n F n Pi(x) (x) = ∑ P′(xi) − 1 có b c không vưot quá n − 1. Vói i = 1,2,...,n ta có F(xi) = Pi(xi) − 1 = 0 suy ra F(x) có n nghi m phân bi t. V y F(x) = 0. M t khác h so của F(x) đoi vói xn−1 bang 0 nên a a a Như v y P′(x1) + P′(x2) + .. . + P′(xn) = 0. 1 1 1 P′(x1) + P′(x2) + ... + P′(xn) = 0. Bài toán 1.4.3 (CHDCLB Đức 1974). (a) Chŕng minh rang không ton tại đa thŕc P(x) đe với moi x ∈ R ta có các bat đȁng thŕc P′(x) > P′′(x), (1.27) P(x) > P′′(x). (1.28) (b) Khȁng định trên còn đúng không neu thay đői bat đȁng thŕc (1.27) bang bat đȁng thŕc P(x) > P′(x). (1.29)
  • 37. 34 Viết đề tài giá sinh viên – ZALO:0973.287.149-TEAMLUANVAN.COM k n+1 Lời giái. (a) Neu P(x) là hang so thì P′(x) = P′′(x) = 0, và bat đȁng thức (1.27) không thỏa mãn. Giả sử degP(x) = n ≥ 1 khi đó neu n lẻ thì deg(P(x)−P′′(x)) = n là so lẻ, từ đó P(x)−P′′(x) ≤ 0 vói ít nhat m t điem x ∈ R. Như v y đoi vói đa thức P(x) không thỏa mãn ho c bat đȁng thức (1.28) ho c bat đȁng thức (1.27). V y (a) đưoc chứng minh xong. (b) Chon P(x) = x2 + 3. Khi đó vói x ∈ R ta có P(x) − P′(x) ≡ x2 − 2x + 3 > 0 và P(x) − P′′(x) ≡ x2 + 1 > 0 nghĩa là khȁng định trên không còn đúng nữa. Bài toán 1.4.4 (Vi t Nam 1986). Cho f ∈ R[x] có deg f = n và f(k) = 2k , với k = 0,1,2,...,n. Tính f(n+1). Lời giái. Xét đa thức g(x) = 1+ x + x(x − 1) + ... + x(x − 1)(x − 2)...(x − n + 1) . 1! 2! n! Khi đó degg = n và n g(k) = ∑Ci = 2k = f (k) vói n + 1 giá trị nên f ≡ g. Do đó i=0 n f(n+1) = g(n+1) = ∑Ci = 2n+1 − 1. i=0 Bài toán 1.4.5 (Singapore 1978). Cho đa thŕc P(x) b c n và hai so a < b thóa mãn P(a) < 0,−P′(a) ≤ 0, P(a) ≤ 0,...,(−1)n P(n)(a) ≤ 0, P(b) > 0, P′(b) ≥ 0, P(b) ≥ 0,...,P(n)(b) ≥ 0. Chŕng minh các nghi m thực của P(x) thu c (a,b).
  • 38. 35 Viết đề tài giá sinh viên – ZALO:0973.287.149-TEAMLUANVAN.COM Lời giái. Khai trien Taylor ta có P P′(b) P(b) 2 P(n)(b) n (x) = P(b) + 1! (x − b) + 2! (x − b) +... + n! (x − b) . Neu x ≥ b thì P(x) > 0 suy ra P(x) không có nghi m x ≥ b. Tương tụ P P′(a) P(a) 2 P(n)(a) n (x) = P(a) + 1! (x − a) + 2! (x − a) −P′(a) P(a) +...+ 2 n! (x − a) (−1)n (a) n = P(a)+ 1! (a − x) + 2! (a − x) + ... + n! (a − x) Neu x < a thì P(x) < 0 suy ra P(x) không có nghi m x ≤ a V y các nghi m phải thu c (a,b). Bài toán đưoc chứng minh xong. Ta goi ưóc lưong ve nghi m ỏ trên là ước lượng Newton.
  • 39. 36 Viết đề tài giá sinh viên – ZALO:0973.287.149-TEAMLUANVAN.COM Chương 2 Đa th c bat kha quy 2.1 Đa th c bat kha quy Định nghĩa 2.1.1. Cho đa thức f ∈ Z[x]. Ta goi đa thức f là bat khá quy trên Z[x] neu f không phân tích đưoc thành tích hai đa thức thu c Z[x] vói b c lón hơn hay bang 1. Trưòng hop f là m t đa thức hữu tý, ta định nghĩa hoàn toàn tương tụ. Moi quan h ve tính bat khả quy trên Z[x] và Q[x] đưoc phát bieu như sau: Định lí 2.1.2. Neu đa thŕc f ∈ Z[x] bat khá quy trên Z[x] thì fcũng bat khá quy trên Q[x]. Đe chứng minh định lý này, ta can bo đe sau đây. Trưóc het, ta goi đa thức f ∈ Z[x] là nguyên bán neu các h so nguyên to cùng nhau. Bo đe 2.1.3 (Bo đe Gauss). Tích của hai đa thŕc nguyên bán là m t đa thŕc nguyên bán. Chŕng minh. Cho hai đa thức nguyên bản f (x) = a0xn + a1xn − 1 + ... + an, g(x) = b0xm + b1xm−1 + ... + bm.
  • 40. 37 Viết đề tài giá sinh viên – ZALO:0973.287.149-TEAMLUANVAN.COM bi Khi đó f (x)g(x) = c0xn+m + c1xn+m−1 + ... + cn+m. Giả sử đa thức tích f (x)g(x) không nguyên bản. Khi đó ton tại m t so nguyên to p là ưóc chung của các h so c0,...,cn+m Vì f nguyên bản nên goi ai là so đau tiên mà ai . p. Vì g nguyên bản nên goi bj là so đau tiên mà bj . p. Bang cách xét h so theo lũy thừa xi+ j ta có h so tương ứng không chia het cho p. Đieu này vô lý. V y f(x)g(x) là đa thức nguyên bản. Chŕng minh Định lý 2.1.2. Cho f ∈ Z[x] là m t đa thức bat khả quy. Giả sử f khả quy trên Q[x], tức là ta có bieu dien f (x) = f1(x) f2(x) vói f1, f2 ∈ Q[x], có b c lón hơn ho c bang 1. Đ t f1(x) = a1 g1(x), f2(x) = a2 g2(x) b1 b2 voi ai toi giản và g1(x), g2(x) nguyên bản. Khi đó f (x) = f1(x) f2(x) = a1a2 g1(x)g2(x) = p g1(x)g2(x) vói (p,q) = 1. b1b2 q Do đó f ∈ Z[x] nên moi h so của khai trien tích g1(x)g2(x) đeu là b i so của q. Suy ra đa thức tích g1(x)g2(x) không nguyên bản. Đieu này trái vói ket quả của Bo đe Gauss. V y f bat khả quy trên Q[x]. 2.1.1 Đa th c vỚi h so th c và ph c Cho m t đa thức vói h so thục thì chưa chac đa thức đó có nghi m trong trưòng so thục, cụ the đa thức x2 + 1 không có nghi m trong trưòng so thục. Dưói đây ta sẽ thay moi đa thức b c n vói h so phức có đúng n nghi m phức. Đe chứng minh ra hãy đưa vào các bo đe sau đây: Bo đe 2.1.4. Moi đa thŕc với h so thực có b c lé có ít nhat m t nghi m thực.
  • 41. 38 Viết đề tài giá sinh viên – ZALO:0973.287.149-TEAMLUANVAN.COM Chŕng minh. Giả sử f (x) = anxn + an−1xn−1 + .... + a0, an 0 và n lẻ. Qua giáo trình giải tích ta biet rang vói những giá trị dương và âm của x, khá lón ve giá trị tuy t đoi, hàm so f (x) có các dau trái nhau. V y có những giá trị thục của x, a và b chȁng hạn, sao cho. f (a) < 0, f (b) > 0. M t khác hàm so f (x) là liên tục, vì v y có m t giá trị c của x, nam giữa a và b, sao cho f(x) = 0. Bo đe 2.1.5. Moi đa thŕc b c hai ax2 + bx + c, với h so phŕc, bao giờ cũng có hai nghi m phŕc. Bo đe 2.1.6. Moi đa thŕc b c lớn hơn 0 với h so thực có ít nhat m t nghi m phŕc. Định lí 2.1.7. Moi đa thŕc b c lớn hơn 0 với h so phŕc có ít nhat m t nghi m phŕc. Chŕng minh. Giả sử f (x) là m t đa thức b c n > 0 f (x) = a0 +a1x+.....+ anxn vói h so phức. Đ t f (x) = a0 +a1x+.....+ anxn vói các ai là các liên hop của các ai vói i = 0,...,n. Xét đa thức g(x) = f (x) f (x). Ta có Vói g(x) = b0 +b1x + .... + b2nx2n bk = ∑ i+ j=k aiaj, k = 0,1,.....,2n.
  • 42. 39 Viết đề tài giá sinh viên – ZALO:0973.287.149-TEAMLUANVAN.COM Vì bk = ∑ i+ j=k aiaj = bk nên các h so bk là thục. Theo Bo đe 2.1.6 g(x) có ít nhat m t nghi m phức z = s + it, g(z) = f (z) f (z) = 0. Do đó ho c f (x) = 0 ho c f (z) = 0. Neu f (z) = 0, f (z) = a0 +a1z+..... + anzn = 0. thì a0 + a1z + ...anzn = a0 + a1z + ... + anzn = 0, tức là f (z) = 0. Như v y ho c z ho c z là nghi m của f (x). H qua 2.1.8. Các đa thŕc bat khá quy của vành C[x], C là trường so phŕc, là các đa thŕc b c nhat. Chŕng minh. Các đa thức b c nhat là bat khả quy. Giả sử f (x) là m t đa thức của C[x] có b c lón hơn 1. Theo Định lí 2.1.7, f (x) có m t nghi m phức c. V y f (x) có m t ưóc thục sụ x−c, do đó f(x) không bat khả quy. H qua 2.1.9. Moi đa thŕc b c n > 0 với h so phŕc có n nghi m phŕc. H qua 2.1.10. Các đa thŕc bat khá quy của R[x], R là trường so thực, là các đa thŕc b c nhat và các đa thŕc b c hai ax2 +bx+c với bi t so b2 −4ac < 0. Chŕng minh. Các đa thức b c nhat và các đa thức b c hai vói bi t so âm rõ ràng là những đa thức bat khả quy của R[x]. Giả sử p(x) là m t đa thức bat khả quy của R[x] vói b c lón hơn m t. V y p(x) không có nghi m thục. Theo Định lí 2.1.7, p(x) có m t nghi m phức z và p(x) chia het cho đa thức b c hai vói h so thục. g(x) = x2 − (z + z)x + zz.
  • 43. 40 Viết đề tài giá sinh viên – ZALO:0973.287.149-TEAMLUANVAN.COM Đa thức g(x) không khả nghịch và là ưóc của phan tử bat khả quy p(x), v y g(x) phải là liên ket của p(x), tức là p(x) = ug(x), 0 /= u ∈ R. Ta có đieu can chứng minh. 2.1.2 Đa th c bat kha quy cua vành Q[x] Đoi vói trưòng so thục R và trưòng so phức C, van đe xét xem m t đa thức đã cho của vành R[x] hay C[x] có bat khả quy hay không rat đơn giản, nhưng trong vành Q[x] vói Q là trưòng so hữu tỉ thì van đe phức tạp hơn nhieu. Đoi vói các đa thức b c hai và ba của Q[x], vi c xét xem có bat khả quy hay không đưoc đưa ve vi c tìm nghi m hữu tỉ của đa thức đó. Các đa thức b c hai và b c ba của Q[x] là bat khả quy khi và chỉ khi chúng không có nghi m hữu tỉ. Đoi vói các đa thức b c lón hơn ba thì van đe phức tạp hơn nhieu. Chȁng hạn đa thức x4 +2x2 +1 = (x2 +1)2 rõ ràng không có nghi m hữu tỉ nào, nhưng nó có m t ưóc thục sụ x2 + 1 , v y không phải là bat khả quy. Ta đã biet, moi đa thức f(x) vói h so hữu tỉ đeu có the viet dưói dạng f (x) = b−1 g(x) trong đó b là m t so nguyên khác 0, g(x) là m t đa thức vói h so nguyên. Trong vành Q[x], f(x) và g(x) là liên ket v y f(x) là bat khả quy khi và chỉ khi g(x) là bat khả quy. Do đó tiêu chuan Eisenstein mà ta đưa ra dưói đây đe xét m t đa thức của Q[x] có bat khả quy hay không là tiêu chuan cho các đa thức vói h so nguyên. Bo đe 2.1.11. Neu f (x) là m t đa thŕc với h so nguyên có b c lớn hơn 0 và f (x) không bat khá quy trong Q[x], thì f (x) phân tích được thành m t tích nhrng đa thŕc b c khác 0 với h so nguyên. Trong nghiên cứu các đa thức bat khả quy, Tiêu chuan Eisenstein sau đây là đ c bi t quan trong.
  • 44. 41 Viết đề tài giá sinh viên – ZALO:0973.287.149-TEAMLUANVAN.COM Định lí 2.1.12 (Tiêu chuan Eisenstein). Cho f (x) ∈ Z[x], deg f(x) = n, f (x) = a0xn + a1xn − 1 + ... + an. Neu có so nguyên to p thóa mãn ba đieu ki n (1) a0 không chia het cho p; (2) a1,a2,...,an chia het cho p; (3) an không chia het cho p2 . thì đa thŕc f(x) bat khá quy trên Q[x]. Chŕng minh. Giả sử f(x) có những ưóc thục sụ trong Q[x] theo Bo đe 2.1.11, f(x) có the viet trong đó Ta có f (x) = g(x)h(x), g(x) = b0 + b1x + ... + brxr , bi ∈ Z, 0 < r < n, h(x) = c0 + c1x + ... +s xs , ci ∈ Z, 0 < s < n. a0 = b0c0, a1 = b1c0 + b0c1, ... ak = bkc0 + bk−1c1 + ... + b0ck, an = brcs. Theo giả thiet p chia het a0 = b0c0; v y vì p là nguyên to nên ho c p chia het cho b0 ho c p chia het cho c0. Giả sử p chia het b0 the thì p không chia het c0, vì neu the thì p2 sẽ chia het a0=b0c0, trái vói giả thiet, p không the chia het moi h so
  • 45. 42 Viết đề tài giá sinh viên – ZALO:0973.287.149-TEAMLUANVAN.COM của g(x), vì neu the thì p sẽ chia het an = brcs, trái vói giả thiet. V y giả sử bk là h so đau tiên của g(x) không chia het cho p. Ta hãy xét ak = bkc0 + bk−1c1 + ... + b0ck, trong đó ak, bk−1, . . . , b0 đeu chia het cho p. V y bkc0 phải chia het cho p. Vì p là nguyên to, ta suy ra ho c bk chia het cho p, ho c c0 chia het cho p, mâu thuȁn vói giả thiet ve bk và c0. 2.2 M t so bài toán đien hình Bài toán 2.2.1 (IMO 1993). Cho n ∈ N và n > 1. Chŕng minh rang đa thŕc f (x) = xn +5xn−1 +3 bat khá quy trên Z[x]. Lời giái. Vói n = 2 có f (x) = x2 + 5x + 3 thì bat khả quy trên Z[x]. Xét n ≥ 3. Giả sử f (x) = g(x)h(x) vói g(x), h(x) ∈ Z[x] và có b c lơn hơn ho c bang 1. Do degg(x)+degh(x) = n ≥ 3 nên suy ra trong hai so degg(x) và degh(x) có m t so lơn hơn 1. Vì f(0) = 3 là so nguyên to nên ho c |g(0)| = 1 ho c |h(0)| = 1. Giả sử g(x) = xk + b1xk−1 + ... + bk vói k > 1 và |g(0)| = 1. Goi a1,a2,...,ak là các nghi m (nói chung là nghi m phức) của g(x). Khi đó ta có Vì |g(0)| = 1 nên g(x) = (x − a1)(x − a2)...(x − ak). |a1a2 ...ak| = 1. (2.1)
  • 46. 43 Viết đề tài giá sinh viên – ZALO:0973.287.149-TEAMLUANVAN.COM k i=1 2 2 2 2 Do g(ai) = 0 nên f(ai) = 0, vói i ∈ {1,...,k}. Nhân các đȁng thức đó lại và sử dụng (2.1) ta đưoc |(a1 + 5)(a2 + 5)...(ak + 5)| = 3 (2.2) M t khác ta có g(−5) = |(a1 + 5)(a2 + 5)...(ak + 5)| và 3 = f (−5) = g(−5)h(−5) nên |(a1 +5)(a2 +5)...(ak +5)| nh n giá trị 1 ho c 3. Đieu này trái vói (2.2) vì k > 1. Từ đó ta có ta có đieu phải chứng minh. Bài toán 2.2.2 (Dụ tuyen IMO). Cho đa thŕc f(x) = ∏n (x−ai)−2 với n ≥ 3 và a1,a2,...an là các so nguyên đôi m t khác nhau. Chŕng minh rang neu f (x) khá quy trên Z[x] thì n = 3. Lời giái. Giả sử f(x) = g(x)h(x) vói g(x), h(x) ∈ Z[x] và degg = p ≥ q = degh ≥ 1. Ngoài ra, giả sử các h so b c cao nhat của g và h đeu bang 1. Trong phép đ t cho x = ai ta sẽ nh n đưoc f (ai) = g(ai)h(ai) = −2. Như v y ta có (g(ai),h(ai)) ∈ {(−2,1),(2,−1),(1,−2),(−1,2)} V y vói moi i, ta có g(ai) + h(ai) = +1 vói ít nhat n giá trị của i, ho c g(ai) + h(ai) = −1 vói ít nhat n giá trị của i. Giả sử g(ai) + h(ai) =1 vói k giá trị của i ∈ {1,...,n}. Rõ ràng k < n (vì đa thức g(x)+h(x)−1 0 và có b c nhỏ hơn thục sụ n. Giả sử k ≥ n ≥ 2, vì neu k < n thì xét đa thức g(x)+h(x)+1. Giả sử g(ai)+h(ai) = 1 vói i ∈ {1,...,k}, neu không như v y thì ta đ t lại thứ tụ của chỉ so. Suy ra g(x) + h(x) − 1 = (x − a1)(x − a2)...(x − ak)w(x) vói w(x) ∈ Z[x].
  • 47. 44 Viết đề tài giá sinh viên – ZALO:0973.287.149-TEAMLUANVAN.COM Cho x = ak+1 vói chú ý g(ak+1)+h(ak+1) = −1 ta đưoc −2 = (ak+1 −a1)(ak+1 −a2)...(ak+1 − ak)w(ak+1). The nhưng trị tuy t đoi của m t tích gom ít nhat là bon thừa so nguyên to khác nhau và khác 0 thì luôn luôn lón hơn 2. V y nên k ≤ 3 và dȁn đen 2 ≤ k ≤ 3. Xét k = 3. Neu n > 4 thì ta có (a4 −a1)(a4 − a2)(a4 −a3)w(a4) = −2; (a5 −a1)(a5 − a2)(a5 −a3)w(a5) = −2 Do đó {a1,a2,a3} ≡ {a4 ±1,a4 ±2,a4 −2}. Nhưng a5 > a4 > a3 nên |a5 −a3| ≥ 3 vói moi i ∈ {1,2,3}. Đieu này vô lý. Neu n = 4 thì g(x)+h(x) − 1 = (x− a1)(x −a2)w(x) do degw ≥ 1 do degg ≤ 3 và degh ≤ 3. Suy ra (ai − a1)(ai − a2)w(ai) = −2 vói i = 3,4. Neu degw = 1 thì degg(x) = 3, h(x) = x − a và 0 = f (a) = (a − a1)(a − a2)(a − a3)(a − a4) = 2. V y w(x) là đa thức hang. Do đó degg = degh = 2, w(x) ≡ 2 và w(a3) = w(a4) = 2. Từ đó −1 = (a3 −a1)(a3 −a2) và {a1,a2} ≡ {a3 −1,a3 + 1}. Tương tụ ta cũng có −1 = (a4 −a1)(a4 −a2) nên {a1,a2} ≡ {a4 − 1,a4 + 1}. Do đó a4 = a3 vô lý. Tóm lại, chỉ có n = 3 thỏa mãn yêu cau của bài toán. Bài toán 2.2.3. Cho n so ai ∈ Z. Chŕng minh rang đa thŕc P(x) = (x − a1)(x − a2)...(x − an) − 1 bat khá quy trong Z[x].
  • 48. 45 Viết đề tài giá sinh viên – ZALO:0973.287.149-TEAMLUANVAN.COM Lời giái. Giả sử ta có phan tích P(x) = G(x)H(x) trong Z[x] và degG < n, degH < n. Ta có P(ai) = G(ai)H(ai) = −1. V y G(ai) và H(ai) nh n hai giá trị ±1 nên G(ai)+H(ai) = 0 Khi đó đa thức G(x)+H(x) có deg(G+H) < n mà có n nghi m. Suy ra G(x) + H(x) ≡ 0, tức là G(x) = −H(x). Do đó P(x) = −[H(x)]2 . So sánh h so của xn của hai ve thì ve trái lón hơn không, ve phải nhỏ hơn không, đieu này vô lý, nên ta có đieu phải chứng minh. Bài toán 2.2.4. Cho n so ai ∈ Z. Chŕng minh rang đa thŕc P(x) = (x − a1)2 (x − a2)2 ...(x − an)2 + 1 bat khá quy trong Z[x]. Lời giái. Vì P(x) > 0 vói moi x nên đa thức P(x) vô nghi m. Suy ra hai nhân tử G(x) và H(x) cũng vô nghi m nên không đoi dau. Giả sử G(x) và H(x) dương vói moi x. Ta có H(ai)G(ai) = 1 nên suy ra H(ai) = 1 và G(ai) = 1. Neu degH < n suy ra H(x)−1 có n nghi m. Kéo theo rang H(x) ≡ 1. Đieu này vô lý, nên degH = degG = n, suy ra H(x) − 1 = A(x − a1)...(x − an), G(x) − 1 = B(x − a1)...(x −an). The vào ta có (x − a1)2 (x − a2)2 ...(x − an)2 + 1 = [1+ A(x −a1)...(x − an)][1+ B(x −a1)...(x −an)]. So sánh h so của x2n thì ta có 1 = AB. So sánh h so tụ do a2 a2 ...a2 + 1 = 1 + (A + B)a1a2 ...an + a2 a2 ...a2 . 1 2 n 1 2 n Suy ra A + B = 0. Do đó 1 = AB = −A2 . Đieu này vô lý. V y bài toán đưoc chứng minh.
  • 49. 46 Viết đề tài giá sinh viên – ZALO:0973.287.149-TEAMLUANVAN.COM 1 1 1 Chương 3 M t so chu đe khác 3.1 Đa th c nhieu bien Bài toán 3.1.1. Cho đa thŕc f (x,y) = ax2 + 2bxy + 2y2 luôn luôn dương. Chŕng minh ( f (x1,y1, f (x2,y2))2 f (x1 −x2 −y2) ≥ (ac −b2 với moi x1,x2,y1,y2. )(x1y2 − x) Lời giái. Từ giả thiet ta có b2 − ac < 0 hay ac − b2 > 0. Ta có đong nhat thức (ax1 2 + 2bx1y1 + cy1 2 )(ax2 2 + 2bx2y2 + cy2 2 ) (ax1x2 + by1y2 + bx2y1 + cy1y2)2 + (ac − b2 )(x1y2 − x2y1)2 . (3.1) Đ t E1 = f (x1,y1) > 0,E2 = f (x2,y2) > 0 F = |ax1x2 +bx1y2 +bx2y1 +cy1y2)| ≥ 0. Từ (3.1) suy ra E1E2 = F2 + (ac − b2 )(x1y2 −x2y1)2 . Mà f(x1 − x2,y1 − y2) = E1 +E2 ±2F. Do đó ( f (x1,y1), f (x2,y2))2 f (x1,x2,y1 − y2) = (E1E2)2 .(E1 + E2 − 2F) ≥ (E1E2) .(2(E1E2)2 1 — 2F) = 2E1E2 − 2((E1E2)2 .F 2
  • 50. 47 Viết đề tài giá sinh viên – ZALO:0973.287.149-TEAMLUANVAN.COM ! 1 1 2 2 2 1 2 2 2 2 2 2 2 2 1 = 2F + 2(ac − b) (x1y2 − 22y1) − 2F(F + (ac − b )(x1y2 − x2y1) 2 ac b2 x y 1 x y 2 2 = 2F2 + 2(ac− b)2 (x1y2 −x2y1)2 −2F2 1+ — ( 1 2 − 2 1) ≥ 2F2 + 2(ac − b)2 (x(y2x2y1)2 — 2F 2 1+ F2 (ac − b2 )(x1y2 − x2y1)2 2F2 = (ac − b2 )(x1y2 − x2y1)2 V y ( f (x ,y , f (x ,y )) 1 f (x − x − y ) ≥ (ac − b2 vói moi x1, x2,y1, y2. Bài toán 3.1.2 (Olympic Sinh viên Toàn quoc 2010). )(x1y2 − x) (a) Chŕng minh rang ŕng với mői so nguyên dương n, bieu thŕc xn + yn + zn có the bieu dien dưới dạng đa thŕc Pn(s, p,q) b c không quá n của s = x+y+z, p = xy+yz+zx, q = xyz. (b) Hãy tìm tőng các h so của đa thŕc P2010(s, p,q). Lời giái. (a) Ký hi u Sn = xn +yn +zn . Trưóc het ta chứng minh Sn = sSn−1 − pSn−2 + qSn−3. (3.2) Th t v y, ta có Sn = sSn−1 − pSn−2 + qSn−3 = (x + y + z)(xn−1 + yn−1 +zn−1 ) −(xy+yz+ zx)(xn−2 + yn−2 +zn−2 )+xyz(xn−3 +yn−3 +zn−3 ) = xn +yn +zn +xyn−1 +xn−1 y+xzn−1 +zxn−1 +yzn−1 +zyn−1 — xn−1 y + xyn−1 + xn−1 z + xzn−1 + yzn−1 y + xyzn−2 + +xyn−2 z+xn−2 yz + (xyzn−2 +xyn−2 z +xn−2 yz) = xn +yn +zn = Sn. !
  • 51. 48 Viết đề tài giá sinh viên – ZALO:0973.287.149-TEAMLUANVAN.COM Ta sẽ chứng minh m nh đe : “bieu thức xn +yn +zn có the bieu dien dưói dạng đa thức Pn(s, p,q) b c không quá n của s = x + y + z, p = xy + yz + zx, q = xyz.” bang phương pháp quy nạp. Ta có S1 = x + y + z = s S2 = x2 +y2 +z2 = (x+y+z)2 −2(zy+yz+zx) = s2 − 2p, S3 = x3 +y3 +z3 = (x+y+z)3 −3(x+y+z)(xy+yz+zx)+3xyz = s3 − 3sp + 3q. V y m nh đe đúng vói n = 1, n = 2, n = 3. Giả sử m nh đe đúng vói n = k − 1, n = k − 2, n = k − 3. Khi đó theo (3.2), suy ra m nh đe cũng đúng vói n = k. Do đó ta có đieu phải chứng minh. (b) Giả sử x2010 + y2010 + z2010 = P2010(s, p,q). Ta can tìm tong các h so của P2010(s, p,q), tức là tính P2010(1,1,1). Xét h phương trình x + y + z = 1 xy + yz + zx = 1 xyz = 1 Khi đó theo Định lí Viet suy ra x, y, z là nghi m của phương trình t3 −t2 +t −1 = 0, tức là t ∈ {1,i,−i}. V y chỉ can chon x = 1,y = i,z = −i, ta đưoc tong các h so của đa thức P2010(s, p,q) là P2010(1,1,1) = 12010 +i2010 +(−i)2010 = −1.
  • 52. 49 Viết đề tài giá sinh viên – ZALO:0973.287.149-TEAMLUANVAN.COM 3.2 Đa th c đoi x ng Những bài toán ve giải phương trình, h phương trình b c cao, chứng minh bat đȁng thức nhieu bien so,. . . thưòng rat phức tạp. M t trong những dạng nhieu bien quan trong và đep đẽ là các đa thức nhieu bien mà trong đó vai trò của các bien so là bình đȁng. Ta sẽ goi những đa thức như v y là các đa thŕc đoi xŕng. Mục này chúng tôi sẽ dành đe nghiên cứu các đa thức đoi xứng nhieu bien, mà t p trung chi tiet vào trưòng hop hai bien, các trưòng hop ba bien và nhieu bien cũng sẽ đưoc đe c p nhưng vói mức đ và phạm vi hạn che hơn. Chúng tôi dụa vào các tài li u [4, 6] đe trình bày mục này. Phan này ta sẽ nghiên cứu m t so bài toán chia đa thức đoi xứng. Tài li u tham khảo chính là [4]. Bài toán 3.2.1. Chŕng minh rang đa thŕc x2n − xn yn + y2n chia het cho x2 + xy + y2 khi và chỉ khi n không phái là b i của 3. Lời giái. Sử dụng các công thức khai trien x3 − y3 = (x − y)(x2 + xy + y2 ), xk − yk = (x − y)(xk−1 + xk−2 y + ... + xyk−2 + yk−1 ) ta thay x3k − y3k chia het cho x2 + xy + y2 . Ta xét các trưòng hop sau 1. n = 3k. Ta có x2n + xn yn + y2n = x6k + x3k y3k + y6k = (x6k − y6k ) + (x3k − y3k ) + 3y6k . Từ đó suy ra x2n −xn yn +y2n không chia het cho x2 +xy+y2 . 2. n = 3k + 1. Ta có x2n + xn yn + y2n = x6k+2 + x3k+1 y3k+1 + y6k+2
  • 53. 50 Viết đề tài giá sinh viên – ZALO:0973.287.149-TEAMLUANVAN.COM = x2 (x6k − y6k ) + xy3k+1 (x3k − y3k ) + y6k (x2 + xy + y2 ). Như v y đa thức x2n −xn yn +y2n chia het cho x2 +xy+y2 . 3. n = 3k + 2. Ta có x2n + xn yn + y2n = x6k+4 + x3k+2 y3k+2 + y6k+4 = x4 (x6k − y6k ) + x2 y3k+2 (x3k − y3k ) +y6k (x2 + x2 y2 y4 )(x2 − xy + y2 ). Như v y x2n −xn yn +y2n chia het cho x2 +xy+y2 . Tóm lại, đa thức x2n −xn yn + y2n chia het cho x2 + xy + y2 khi và chỉ khi n không phải là b i của 3. Bài toán 3.2.2. Chŕng minh rang với moi n ∈ Z+, đa thŕc x2n − xn yn + y2n không chia het cho x2 +xy+y2 . Lời giái. Giả sử x2n − xn yn + y2n chia het cho x2 + xy + y2 , tức là x2n −xn yn +y2n = (x2 +xy+y2 )q(x,y), trong đó q(x, y) là đa thức đoi xứng vói h so nguyên (do h so b c cao nhat của đa thức chia bang 1, còn các h so của đa thức bị chia và đa thức chia là các so nguyên). Trong đȁng thức trên cho x = y = 1, ta đưoc 1 = 3q(1, 1), vô lí vì q(1, 1) là m t so nguyên. Đieu này chứng tỏ đa thức x2n − xn yn + y2n không chia het cho x2 +xy+y2 . Bài toán 3.2.3. Với n ∈ Z+ nào thì x2n +xn yn +y2n không chia het cho x2 +xy+y2 ? Lời giái. Giả sử x2n +xn yn +y2n = (x2 −xy+y2 )q(x,y). (3.3) trong đó q(x,y) là đa thức đoi xứng vói h so nguyên. Ta xét hai trưòng hop
  • 54. 51 Viết đề tài giá sinh viên – ZALO:0973.287.149-TEAMLUANVAN.COM 1. n là so lẻ. Trong đȁng thức (3.3) thay x bỏi −x ta đưoc x2n − xn yn + y2n = (x2 + xy + y2 )q(−x, y). Theo Bài toán 3.2.2 đȁng thức này không the xảy ra. 2. n là so chȁn. Trong (3.3) thay x bỏi −x, ta đưoc. x2n +xn yn + y2n = (x2 +xy+y2 )q(−x,y). Theo ví dụ Bài toán 3.2.1 thì đȁng thức trên đúng khi và chỉ khi n = 3m + 1 ho c n = 3m+2. Neu n = 3m + 1 thì do n là so chȁn, nên m phải là so lẻ, hay m = 2k + 1, do đó n = 6k +4. Neu n = 3k + 2, thì do n là so chȁn, nên m phải là so chȁn, hay m = 2k, do đó n = 6k +2. V y x2n +xn yn +y2n chia het cho x2 −xy+y2 khi và chỉ khi n = 6k+2 ho c n = 6k + 4, vói k ∈ Z, n ∈ Z+. Bài toán 3.2.4. Với n ∈ Z+ nào thì x2n − xn yn + y2n chia het cho x2 − xy + y2 ? Lời giái. Giả sử x2n − xn yn + y2n = (x2 − xy + y2 )q(x,y), (3.4) trong đó q(x,y) là đa thức đoi xứng vói h so nguyên. Xét hai trưòng hop 1. n là so chȁn. Trong (3.4) thay x bỏi −x, ta đưoc x2n − xn yn + y2n = (x2 + xy + y2 )q(−x, y). Theo Bài toán 3.2.2 đȁng thức này không the xảy ra. 2. n là so lẻ. Trong (3.4) thay x bỏi −x, ta đưoc x2n +xn yn + y2n = (x2 +xy+y2 )q(−x,y).
  • 55. 52 Viết đề tài giá sinh viên – ZALO:0973.287.149-TEAMLUANVAN.COM Theo Bài toán 3.2.1, đȁng thức trên đúng khi và chỉ khi n = 3m + 1 ho c n = 3m + 2. Neu n = 3m + 1 thì do n là so lẻ, nên m phải là so chȁn, tức là m = 2k và khi đó n = 6k +1. Neu n = 3m + 2, thì do n là so lẻ, nên m phải là so lẻ, tức là m = 2k − 1. Khi đó n = 6k − 1. V y x2n − xn yn + y2n chia het cho x2 − xy + y2 khi và chỉ khi n = 6k ±1, k ∈ Z, n ∈ Z+. Bài toán 3.2.5. Xác định n đe (x + y)n + xn + yn chia het cho x2 + xy + y2 . Lời giái. Giả sử (x+y)n xn +yn chia het cho x2 +xy+y2 . Khi đó ta có (x + y)n xn + yn = (x2 + xy + y2 )q(x,y) (3.5) trong đó q(x,y)là đa thức đoi xứng vói h so nguyên. Trong (3.5) thay x, y tương ứng bỏi x2 , y2 , ta có (x2 + y2 )n + x2n + y2n = (x4 + x2 y2 + y4 )q(x2 , y2 ) = (x2 + xy + y2 )(x2 − xy + y2 )q(x2 ,y2 ) (3.6) Đȁng thức (3.6) chứng tỏ (x2 +y2 )n +x2n +y2n phải chia het cho x2 − xy+y2 . Ta có (x2 + y2 )n −(xy)n = (x2 +y2 −xy)[(x2 +y2 )n−1 + (x2 +y2 )n−2 +... +(x2 + y2 )(xy)n−2 + (xy)n−1 ]. (3.7) Tiep theo ta có (x2 + y2 )n + x2n + y2n = [(x2 + y2 )n − (xy)n ] + (x2n + xn yn + y2n ). (3.8) Từ (3.7) và (3.8) suy ra (x2 + y2 )n + x2n + y2n chia het cho x2 − xy + y2 khi và chỉ khi x2n +xn yn +y2n chia het cho x2 −xy+y2 . Theo Bài toán 3.2.3 đieu này có đưoc khi và chỉ khi n = 6k + 2 ho c n = 6k + 4 vói k ∈ Z, n ∈ Z+.
  • 56. 53 Viết đề tài giá sinh viên – ZALO:0973.287.149-TEAMLUANVAN.COM Ngưoc lại, giả thiet rang n = 2m, vói m = 3k + 1 ho c m = 3k + 2. The thì (x + y)n + xn + yn = (x + y)2m + x2m + y2m = [(x + y)2m − (xy)m ] + (x2m + xm ym + y2m ). Đe ý rang (x + y)2m − (xy)m = [(x + y)2m − (xy)m ] = [(x + y)2 − xy]p(x, y) = (x2 + xy + y2 )p(x, y), trong đó p(x,y) là đa thức đoi xứng vói h so nguyên. Do đó (x + y)2m − (xy)m chia het cho x2 +xy + y2 . M t khác vì m = 3k +1, m = 3k + 2 nên theo Bài toán 3.2.1 đa thức x2m +xm ym +y2m chia het cho x2 +xy+y2 . V y đa thức (x+y)n +xn +yn chia het cho x2 +xy+y2 khi và chỉ khi n = 6k+2 ho c n = 6k +4, vói k ∈ Z, n ∈ Z+. 3.3 Phương trình hàm đa th c Ő khía cạnh giải tích toán hoc, đa thức là m t hàm so rat đ c bi t. Ta có the tính giá trị hàm so chỉ bang các phép toán c ng, trừ, nhân. Vi c tính đạo hàm, tích phân cũng rat de. M t bài toán quan trong trong lý thuyet các đa thức là bài toán giải phương trình hàm đa thức. M t cách ngan gon, phương trình hàm đa thức là các phương trình mà an hàm của nó là các đa thức đại so . . . Lu n văn này sẽ dành phan này đe tìm hieu m t so bài toán ve phương trình hàm đa thức xuat hi n trong các kỳ thi Olympic. Những chủ đe sâu sac hơn ve lĩnh vục này, chúng ta có the tham khảo Nguyen Văn M u [2]. Bài toán 3.3.1 (Đe thi chon đ i tuyen TP.HCM năm 2006-2007). Tìm tat cá các đa thŕc thóa mãn P2 (2x) = 4[P(x2 )− xP(2x)] với moi x ∈ R.
  • 57. 54 Viết đề tài giá sinh viên – ZALO:0973.287.149-TEAMLUANVAN.COM Lời giái. Ta có phương trình đã cho tương đương vói [P(2x)+2x]2 = 4[P(x2 ) +x2 ] vói moi x ∈ R. Đ t F(x) = P(x)+x, phương trình hàm đã cho trỏ thành F2 (2x) = 4F(x2 )2 . Đ t d = degF(x) thì ta có d2 = d. Từ đây suy ra d = 0 ho c d = 1. • Khi d = 0 thì F(x) = const = c. Thay vào phương trình đã cho ta có c2 = 4c. Tức là ta có F(x) ≡ 0 ho c F(x) ≡ 4. Như v y P(x) = 0 ho c P(x) = 4 −x. • Khi d = 1 thì ta có F(x) = ax + b vói a /= 0. Thay vào phương trình và thu gon hai ve ta có 4a2 x2 +4abx+b2 = 4ax2 +4b. Đong nhat h so ta đưoc a = 0, b = 1. Như v y F(x) = x, tức là P(x) ≡ 0. Tóm lại nghi m của phương trình đã cho là P(x) = 0, P(x) = 4−x, P(x) ≡ 0. Bài toán 3.3.2 (Rumani 1980). Tìm tat cá các đa thŕc thóa mãn P(x2 ) = [P(x)]2 với moi x ∈ R. Lời giái. Giả sử đa thức can tìm có dạng P(x) = anxn + an−1xn−1 + ... + a1x + a0 vói an 0. Giả thiet rang m t trong các h so an−1,an−2,...,a0 là khác không. Goi k, vói k < n, là so lón nhat sao cho ak /= 0. Khi đó ta có P(x2 ) = anx2n +akx2k +... +a1x + a0
  • 58. 55 Viết đề tài giá sinh viên – ZALO:0973.287.149-TEAMLUANVAN.COM n x = (anx2n + akx2k + ... + a1x + a0)=[P(x)]2 . Đong nhat h so ta nh n đưoc 0 = 2anak. Đieu này trái vói giả thiet ak an−1 = an−2 = ... = a0 = 0. 0. Suy ra V y P(x) = anxn . Từ đieu ki n anx2n = P(x2 ) = [P(x)]2 = a2 x2n ta nh n đưoc an = 1. V y P(x) = xn là đa thức can tìm. Bài toán 3.3.3 (Rumani 1980). P(x2 − 2x) = [P(x)]2 với moi x ∈ R. Lời giái. Đ t y = x − 1, Q(y) = P(y − 1). Khi đó [P(x − 2)]2 = [P(y − 1)]2 = [Q(y)]2 , (x2 −2x) = P(y2 −1) = Q(y2 ). Do đó P(x2 − 2x) = [P(x−2)]2 vói moi x ∈ R. V y Q(y2 ) = [Q(y)]2 vói moi x ∈ R. Theo Bài toán 3.3.2 ta có Q(y) = yn , hay P(y) = (y + 1)n vói moi n ∈ N∗. V y P(x) = (x + 1)n vói moi n ∈ N∗. Liên quan đen các phương trình hàm, ta sẽ thảo lu n m t bài toán mà ràng bu c bài toán có dạng bat phương trình như sau. Bài toán 3.3.4 (Albanian TST 2009). Tìm tat cá các đa thŕc P(x) khác không có h so không âm thóa mãn P(x)P 1 ≤ [P(1)]2 với x > 0. Lời giái. Giả sử P(x) = adxd + ... a1x + a0 là đa thức thỏa mãn yêu cau bài toán. Khi đó ai ≥ 0 vói moi i = 0,1,...,d.
  • 59. 56 Viết đề tài giá sinh viên – ZALO:0973.287.149-TEAMLUANVAN.COM x x x Vói moi x > 0, theo bat đȁng thức Bunyakovsky ta có P(x)P 1 = adxd + ...a1x + a0 adx−d + ...a1x−1 + a0 ≥ (ad + ...a1 + a0) = [P(1)]2 . Ket hop giả thiet ta có P(x)P 1 ≤ [P(1)]2 vói x > 0 P(x)P 1 = [P(1)]2 vói x > 0. Thay dạng của đa thức P(x) như đã giả thiet, ta có adxd + ...a1x + a0 adx−d + ...a1x−1 + a0 = [P(1)]2 vói x > 0. Bien đoi tương đương ta nh n đưoc adxd + ...a1x + a0 ad + ...a1xd−1 + a0xd = [P(1)]2 xd vói x > 0. So sánh h so của xd+1,xd+2,...,x2d ta đưoc a0 = a1 = ... = ad−1 = 0. Do v y P(x) = adxd vói ad > 0. Thử lại ta thay đa thức P(x) = adxd vói ad > 0 thỏa mãn các yêu cau bài toán. Tóm lại, P(x) = adxd vói ad > 0 là các đa thức can tìm. 3.4 Đa th c Chebyshev Đa thức Chebyshev là m t lóp đ c bi t các đa thức, nó là soi dây liên ket đep đẽ giữa đại so và lưong giác. Các đa thức Chebyshev không chỉ là đoi tưong nghiên cứu của đại so, mà nó còn là công cụ quan trong trong giải tích toán hoc, lý thuyet xap xỉ. 2
  • 60. 57 Viết đề tài giá sinh viên – ZALO:0973.287.149-TEAMLUANVAN.COM n Mục này dành đe nghiên cứu sơ lưoc ve các đa thức Chebyshev và các bài toán liên quan. Tài li u tham khảo chính đưoc sử dụng ỏ đây là [3, 6]. 3.4.1 Định nghĩa - Tính chat Các đa thức Tn(x) vói n ∈ N đưoc xác định truy hoi T0(x) = 1, T1(x) = x Tn+1(x) = 2xTn(x) − Tn−1(x) đưoc goi là các đa thŕc Chebyshev loại 1. Các đa thức Un(x) vói n ∈ N đưoc xác định truy hoi U0(x) = 0, U1(x) = 1 Un+1(x) = 2xUn(x) −Un−1(x) vói n ≥ 1 Tính chat 3.4.1. Các đa thŕc Chebyshev loại 1 có các tính chat sau: (1) Với moi x ∈ [−1,1] ta có Tn(x) = cos(narccosx). (2) Tn(x) là đa thŕc b c n, có h so cao nhat là 2n−1 . (3) Tn(x) là hàm so chȁn khi n chȁn và là hàm so lé khi n lé. (4) Ta có ước lượng |Tn(x) ≤ 1 với moi x ∈ [−1,1]. (5) Phương trình |Tn(x)| = 1 có đúng n nghi m phân bi t trong [−1,1] được cho bới x = cos π với k = 0,1,2,...,n − 1. đưoc goi là các đa thŕc Chebyshev loại 2.
  • 61. 58 Viết đề tài giá sinh viên – ZALO:0973.287.149-TEAMLUANVAN.COM Tính chat 3.4.2. Các đa thŕc Chebyshev loại 2 có các tính chat sau: (1) Với moi x ∈ (−1,1) ta có (2) Ta có Un(x) = sin(n arccos x) √ 1 − x2 . Un(x) = 1 Tn′(x). n (3) Un(x) là đa thŕc b c n, có h so cao nhat là 2n−1 . (4) Un(x) là hàm so chȁn khi n lé và là hàm so lé khi n chȁn. (5) Ta có ước lượng |Tn(x) ≤ n với moi x ∈ (−1,1). 3.4.2 M t so bài toán chon loc Bài toán chon loc đau tiên mà chúng tôi sẽ trình bày là ve sụ bieu dien của mői đa thức như là chuői các đa thức Chebyshev. Bài toán 3.4.3. Chŕng minh rang moi đa thŕc f (x) b c n ≥ 1 đeu có the bieu dien duy nhat dưới dạng n f (x) = ∑aiTi(x), an = 0. (3.9) i=0 Lời giái. Ta có Tn(x) là đa thức b c n có h so cao nhat là 2n−1 nên ta có the viet Tn(x) = 2n−1 xn + ϕ(x) vói là ϕ(x) đa thức b c nhỏ hơn n. Suy ra xn = 1 Tn(x)− 1 ϕ(x). 2n−1 Bang quy nạp ta chứng minh đưoc: 2n−1 f (x) = a0 + a1T1(x)+ a2T2(x)+... + anTn(x).
  • 62. 59 Viết đề tài giá sinh viên – ZALO:0973.287.149-TEAMLUANVAN.COM − ∈ 2 8 + 4 + 2 + Bây giò ta chứng minh tính duy nhat của cách bieu dien này. Giả sử f (x) = a0 + a1T1(x) + a2T2(x) + ... + anTn(x) = a′ 0 + a1 ′ T1(x)+a2 ′ T2(x)+ ...+ a′ nTn(x). Khi đó V y Hay n ∑ ai ai′ Ti(x) = 0 vói moi x R. i=0 a0 −a′ 0 = a1 −a′ 1 = ... = an −a′ n = 0. a0 = a0 ′ ,a1 = a′ 1,...,an = a′ n. Bài toán 3.4.4. Cho đa thŕc thực f (x) = ax3 +bx2 +cx + d và so α > 0. Biet rang | f (x)| ≤ α với moi x ∈ [−1,1]. Tìm giá trị lớn nhat của giá trị tuy t đoi các h so của đã thŕc đã cho. Lời giái. Đ t A = f (−1) = −a + b − c + d B = f − 1 = − a + b − c + d 2 8 4 2 C = f 1 = a b c d D = f (1) = a+b+c+d E = f (0) = d. Từ đây ta có a = − 2 A+ 4 B− 4 C + 2 D 3 3 3 3 b = 1 A+ 1 D −E 2 2 c = 1 A − 8 B + 8 C − 1 D 6 6 6 6 d = E.
  • 63. 60 Viết đề tài giá sinh viên – ZALO:0973.287.149-TEAMLUANVAN.COM 2n − n . ∑ j=1 1 − xj P(xj). ≤ n ·n = 2 Từ giả thiet ta có Vói các đa thức |a| ≤ 4α, |b| ≤ 2α, |c| ≤ 3α, |d| ≤ α. f(x) = α 4x3 −3x và g(x) = α 2x2 − 1 , thay vào ta thay bat đȁng thức sẽ trỏ thành đȁng thức. V y max|a| = 4α, max|b| = 2α, max|c| = 3α, max|d| = α. Bài toán 3.4.5. Cho đa thŕc Pn−1(x) b c không vượt quá n−1 có h so b c cao nhat a0, thóa mãn đieu ki n √ 1− x2 |Pn−1(x)| ≤ 1 với moi x ∈ [−1,1]. Chŕng minh rang |a0| ≤ 2n−1 . Lời giái. Ta viet đa thức đã cho dưói dạng n i suy Lagrange theo các nút n i suy xj = cos 2 j−1 π là các nghi m của đa thức Chebyshev Tn(x). Ta có P x 1 n 1 j−1 q 1 x2P x Tn(x) Suy ra n−1( ) = ∑ ( ) j=1 — j n−1 ( j) x − xj . a 2n−1 n 1 j−1 q 1 x2P x V y nên 0 = n ∑ ( ) j=1 — j ( j). 2n−1 n . q 2 2n−1 n−1 Ta có đieu phải chứng minh. Bài toán 3.4.6. Giá thiet rang đa thŕc Pn−1(x) thóa mãn các đieu ki n của Bài toán 3.4.3. Chŕng minh rang |Pn−1(x)| ≤ n với moi x ∈ [−1,1]. Lời giái. Vói các xj đưoc chon như ỏ Bài toán 3.4.5 thì do hàm so y = cos(x) nghịch bien trong (0,π) nên −1 < xn < xn−1 < ... < x2 < x1 < 1. n − |a0| ≤ . .
  • 64. 61 Viết đề tài giá sinh viên – ZALO:0973.287.149-TEAMLUANVAN.COM − n n−1( )| ≤ n ∑ . j j). .x − xj. ≤ n ∑ (x − xj) . √ 1−x2 ≥ q 1−x2 = sin(arccosx1) = sin π . 1 ≥ sinx ≥ 2 , sin π ≥ π , 2 = 1√ 1−x2 ≥ 1 = ∑ Neu x1 < x < 1 thì P x 1 n . q 1 x2P x . |Tn(x)| 1 n Tn(x) (3.10) (do x−xj > 0 và Tn(x) có dau không đoi trên (x1,1]). M t khác n Tn(x) = 2n−1 ∏(x xj) j=1 nên ta có n Tn ′ (x) = 2n−1 ∑ n j=1 (x − xj) n Tn(x) . (3.11) Ta có k=1 (x − xk) k=1 x−xk nên từ (3.10) và (3.11) suy ra |Tn ′(x)| = |Un(x)| ≤ n |Pn−1(x)| ≤ n vói moi x ∈ (x1;1]. Hoàn toàn tương tụ ta cũng có |Pn−1(x)| ≤ n vói moi x ∈ [−1,xn) Xét xn ≤ x ≤ x1. Khi đó ta có 1 2n Do suy ra x π 2n 2n n n n 1 |Pn−1(x)| ≤ 1 Bài toán đưoc chứng minh xong. = n. Bài toán 3.4.7 (Định lý Berstein-Markov). Cho đa thŕc Pn(x) = a0xn +a1xn−1 +...+an thóa mãn đieu ki n |Pn(x)| ≤ 1 với moi x ∈ [−1,1]. Chŕng minh rang khi đó |Pn ′(x)| ≤ n2 với moi x ∈ [−1,1] n j=1 j=1 ∏ | − n−1 (
  • 65. 62 Viết đề tài giá sinh viên – ZALO:0973.287.149-TEAMLUANVAN.COM . . n. .sin(α)Pn ′ (cos(α)). ≤ n kéo theo √ 1−x2 .Pn ′(x). ≤ 1. Lời giái. Đ t x = cosa. Khi đó theo giả thiet thì |Pn (cosa)| ≤ 1. Do Pn(cosa) có dạng n Pn (cosa) = ∑ (aj cos jα +bj sin jα). j=0 nên ta có the áp dụng ket quả của Bài toán 3.4.6, ta đưoc Cũng theo Bài toán 3.4.6, ta có Pn ′(x) ≤ . n . . n . V y |Pn ′(x)| ≤ n2 . Ta có đieu phải chứng minh.
  • 66. 63 Viết đề tài giá sinh viên – ZALO:0973.287.149-TEAMLUANVAN.COM Ket lu n 1 Nh ng ket qua đã đạt đư c Lu n văn “Đa thŕc trong các bài thi hoc sinh giói” đã đạt đưoc các ket quả sau: 1. Trình bày đưoc các tính chat của đa thức, các bài toán ve phép chia đa thức, ưóc - b i, nghi m và phương trình b c cao, đạo hàm và khai trien Taylor; 2. Lý thuyet và các bài toán ve các đa thức bat khả quy trên các vành (trưòng) so; 3. M t so chủ đe nâng cao như các đa thức nhieu bien, đa thức đoi xứng, phương trình hàm đa thức và đa thức Chebyshev. 2 Đe xuat m t so hưỚng nghiên c u tiep theo Đa thức đại so là m t trong những chủ đe quan trong và sâu sac của toán hoc. Sau những ket quả đã đạt đưoc trong lu n văn, chúng tôi hi vong và co gang sẽ tiep tục nghiên cứu các chủ đe liên quan, chȁng hạn: • Các phân thức hữu tý và các bài toán liên quan, • Bài toán bieu dien đa thức, sụ phân bo nghi m của đa thức và ứng dụng, • Các khía cạnh giải tích của đa thức, phương trình hàm đa thức. . .
  • 67. 64 Viết đề tài giá sinh viên – ZALO:0973.287.149-TEAMLUANVAN.COM Tài li u tham khao Tieng Vi t [1] Nguyen Hữu Đien (2006), Đa thŕc và ŕng dnng, NXB Giáo dục. [2] Nguyen Văn M u (1997), Phương trình hàm, NXB Giáo dục. [3] Nguyen Văn M u (2007), Chuyên đe Đa thŕc đại so và phân thŕc hru tý, NXB Giáo dục. [4] Nguyen Văn M u, Nguyen Văn Ngoc (2010), Chuyên đe Đa thŕc đoi xŕng và áp dnng, NXB Giáo dục. [5] Lê Thị Thanh Nhàn (2015), Lý thuyet đa thŕc, NXB ĐHQG Hà N i. [6] Lê Hoành Phò, Nguyen Văn Nho, Nguyen Tài Chung (2016), Chuyên kháo đa thŕc (tái bản lan thứ nhat), NXB ĐHQG Hà N i. Tieng Anh [7] Dusˇan Djukic´ (2007), Polynomials in One Variable, Olympiad Training Ma- terials (see www.imomath.com). [8] 51st International Mathematical Olympiad (2010), Shortlisted Problems with Solutions.